Exam 3 Test Bank

¡Supera tus tareas y exámenes ahora con Quizwiz!

Tinnitus., - is Ringing in ears

.

A nurse is preparing to perform a lung assessment on a patient and discovers through the nursing history the patient smokes. The nurse figures the pack-years for this patient who has smoked two and a half (2 1/2) packs a day for 20 years. Which value will the nurse record in the patient's medical record? Record answer as a whole number. _________ pack-years

50 Pack-years = Number of years smoking × Number of packs per day: 20 × 2.5 = 50.

A patient with hypertension is started on a new medication for treatment and is reporting a continuous dry cough. Which of the following medications do you suspect is causing this problem? a. Lisinopril b. Labetalol c. Losartan d. Hydrochlorothiazide

A

An experienced medical-surgical nurse chooses to work in obstetrics. Which level of proficiency is the nurse upon initial transition to the obstetrical floor? a. Novice b. Proficient c. Competent d. Advanced beginner

A A beginning nursing student or any nurse entering a situation in which there is no previous level of experience (e.g., an experienced operating room nurse chooses to now practice in home health) is an example of a novice nurse. A proficient nurse perceives a patient's clinical situation as a whole, is able to assess an entire situation, and can readily transfer knowledge gained from multiple previous experiences to a situation. A competent nurse understands the organization and specific care required by the type of patients (e.g., surgical, oncology, or orthopedic patients). This nurse is a competent practitioner who is able to anticipate nursing care and establish long-range goals. A nurse who has had some level of experience with the situation is an advanced beginner. This experience may only be observational in nature, but the nurse is able to identify meaningful aspects or principles of nursing care

During a routine physical examination of a 70-year-old patient, a blowing sound is auscultated over the carotid artery. Which assessment finding will the nurse report to the health care provider? a. Bruit b. Thrill c. Phlebitis d. Right-sided heart failure

A A bruit is the sound of turbulence of blood passing through a narrowed blood vessel & is auscultated as a blowing sound. A bruit can reflect cardiovascular disease in the carotid artery of middle-aged to older adults. Intensity or loudness is related to the rate of blood flow through the heart or the amount of blood regurgitated. A thrill is a continuous palpable sensation that resembles the purring of a cat. Jugular venous distention, not bruit, is a possible sign of right-sided heart failure. Some patients with heart disease have distended jugular veins when sitting. Phlebitis is an inflammation of a vein that occurs commonly after trauma to the vessel wall, infection, immobilization, & prolonged insertion of IV catheters. It affects predominantly peripheral veins.

Which assessment finding in a patient admitted with acute decompensated heart failure (ADHF) requires the most immediate action by the nurse? a. Oxygen saturation of 88% b. Weight gain of 1 kg (2.2 lb) c. Heart rate of 106 beats/minute d. Urine output of 50 mL over 2 hours

A A decrease in oxygen saturation to less than 92% indicates hypoxemia. The nurse should administer supplemental oxygen immediately to the patient. An increase in apical pulse rate, 1-kg weight gain, and decreases in urine output also indicate worsening heart failure and require nursing actions, but the low oxygen saturation rate requires the most immediate nursing action.

A nurse is caring for a group of patients. Which patient will the nurse see first? a. An adult with an S4 heart sound b. A young adult with an S3 heart sound c. An adult with vesicular lung sounds in the lung periphery d. A young adult with bronchovesicular breath sounds between the scapula posteriorly

A A fourth heart sound (S4) occurs when the atria contract to enhance ventricular filling. An S4 is often heard in healthy older adults, children, and athletes, but it is not normal in adults. Because S4 also indicates an abnormal condition, report it to a health care provider. An S3 is considered abnormal in adults over 31 years of age but can often be heard normally in children and young adults. Vesicular lungs sounds in the periphery and bronchovesicular lung sounds in between the scapula are normal findings.

The nurse is caring for a patient in the operating suite who is experiencing hypercarbia (carbon dioxide, CO2, retention), tachypnea, tachycardia, premature ventricular contractions, and muscle rigidity. Which condition does the nurse suspect the patient is experiencing? a. Malignant hyperthermia b. Fluid imbalance c. Hemorrhage d. Hypoxia

A A life-threatening, rare complication of anesthesia is malignant hyperthermia. Malignant hyperthermia causes hypercarbia, tachycardia, tachypnea, premature ventricular contractions, unstable blood pressure, cyanosis, skin mottling, and muscular rigidity. It often occurs during anesthesia induction. Hypoxia would manifest with decreased oxygen saturation as one of its signs and symptoms. Fluid imbalance would be assessed with intake and output and can manifest with tachycardia and blood pressure fluctuations but does not have muscle rigidity. Hemorrhage can manifest with tachycardia and decreased blood pressure, along with a thready pulse. Usually some sign or symptom of blood loss is noted (e.g., drains, incision, orifice, and abdomen).

Which statement by a patient scheduled for surgery is most important to report to the health care provider? a. I had a heart valve replacement last year. b. I had bacterial pneumonia 3 months ago. c. I have knee pain whenever I walk or jog. d. I have a strong family history of breast cancer.

A A patient with a history of valve replacement is at risk for endocarditis associated with invasive procedures and may need antibiotic prophylaxis. A current respiratory infection may affect whether the patient should have surgery, but a history of pneumonia is not a reason to postpone surgery. The patients knee pain is the likely reason for the surgery. A family history of breast cancer does not have any implications for the current surgery

SKIP The nurse is caring for a patient in the operating suite. Which outcome will be most appropriate for this patient at the end of the intra-operative phase? a. The patient will be free of burns at the grounding pad. b. The patient will be free of nausea and vomiting. c. The patient will be free of infection. d. The patient will be free of pain.

A A primary focus of intraoperative care is to prevent injury and complications related to anesthesia, surgery, positioning, and equipment use, including use of the electrical cautery grounding pad for the prevention of burns. The perioperative nurse is an advocate for the patient during surgery and protects the patient's dignity and rights at all times. Signs and symptoms of infection do not have the time to present during the intraoperative phase. During the intraoperative phase, the patient is anesthetized and unconscious and typically has an endotracheal tube that prevents conversation. Nausea, vomiting, and pain typically begin in the postoperative phase of the experience.

During a genitourinary examination of a 30-year-old male patient, the nurse identifies a small amount of a white, thick substance on the patient's uncircumcised glans penis. What is the nurse's next step? a. Record this as a normal finding. b. Avoid embarrassing questions about sexual activity. c. Notify the provider about a suspected sexually transmitted infection. d. Tell the patient to avoid doing self-examinations until symptoms clear.

A A small amount of thick, white smegma sometimes collects under the foreskin in the uncircumcised male and is considered normal. Penile pain or swelling, genital lesions, and urethral discharge are signs and symptoms that may indicate sexually transmitted infections (STI). All men 15 years and older need to perform a male-genital self-examination monthly. The nurse needs to assess a patient's sexual history and use of safe sex habits. Sexual history reveals risks for STI and HIV.

The standing orders for a patient include acetaminophen 650 mg every 4 hours prn for headache. After assessing the patient, the nurse identifies the need for headache relief and determines that the patient has not had acetaminophen in the past 4 hours. Which action will the nurse take next? a. Administer the acetaminophen. b. Notify the health care provider to obtain a verbal order. c. Direct the nursing assistive personnel to give the acetaminophen. d. Perform a pain assessment only after administering the acetaminophen.

A A standing order is a preprinted document containing orders for the conduct of routine therapies, monitoring guidelines, and/or diagnostic procedures for specific patients with identified clinical problems. The nurse will administer the medication. Notifying the health care provider is not necessary if a standing order exists. The nursing assistive personnel are not licensed to administer medications; therefore, medication administration should not be delegated to this person. A pain assessment should be performed before and after pain medication administration to assess the need for and effectiveness of the medication.

When caring for a patient the second postoperative day after abdominal surgery for removal of a large pancreatic cyst, the nurse obtains an oral temperature of 100.8 F. Which action should the nurse take first? a. Have the patient use the incentive spirometer. b. Assess the surgical incision for redness and swelling. c. Administer the ordered PRN acetaminophen (Tylenol). d. Ask the health care provider to prescribe a different antibiotic.

A A temperature of 100.8 F in the first 48 hours is usually caused by atelectasis, and the nurse should have the patient cough and deep breathe. This problem may be resolved by nursing intervention, and therefore notifying the health care provider is not necessary. Acetaminophen will reduce the temperature, but it will not resolve the underlying respiratory congestion. Because a wound infection does not usually occur before the third postoperative day, a wound infection is not a likely source of the elevated temperature

A patient arrives at the ambulatory surgery center for a scheduled laparoscopy procedure in outpatient surgery. Which information is of most concern to the nurse? a. The patient is planning to drive home after surgery. b. The patient had a sip of water 4 hours before arriving. c. The patients insurance does not cover outpatient surgery. d. The patient has not had surgery using general anesthesia before.

A After outpatient surgery, the patient should not drive home and will need assistance with transportation and home care. The patients experience with surgery is assessed, but it does not have as much application to the patients physiologic safety. The patients insurance coverage is important to establish, but this is not usually the nurses role or a priority in nursing care. Having clear liquids a few hours before surgery does not usually increase risk for aspiration.

The nurse is caring for a patient who refuses to bathe in the morning. When asked why, the patient says "I always bathe in the evening." Which action by the nurse is best? a. Defer the bath until evening and pass on the information to the next shift. b. Tell the patient that daily morning baths are the "normal" routine. c. Explain the importance of maintaining morning hygiene practices. d. Cancel hygiene for the day and attempt again in the morning.

A Allow the patient to follow normal hygiene practices; change the bath to evening. Patients have individual preferences about when to perform hygiene and grooming care. Knowing the patient's personal preferences promotes individualized care for the patient. Hygiene care is never routine. Maintaining individual personal preferences is important unless new hygiene practices are indicated by an illness or condition. Cancelling hygiene and trying again is not an option since the nurse already knows the reason for refusal. Adapting practices to meet individual needs is required.

A patient with sepsis as a result of long-term leukemia dies 25 hours after admission to the hospital. A full code was conducted without success. The patient had a urinary catheter, an intravenous line, an oxygen cannula, and a nasogastric tube. Which question is the priority for the nurse to ask the family before beginning postmortem care? a. "Is an autopsy going to be done?" b. "Which funeral home do you want to use?" c. "Would you like to assist in bathing your loved one?" d. "Do you want me to remove the lines and tubes before you see your loved one?"

A An autopsy or postmortem examination may be requested by the patient or the patient's family, as part of an institutional policy, or if required by law. Because the patient's death occurred as a result of long-term illness and not under suspicious circumstances, whether to conduct a postmortem examination would be decided by the family, & consent would have to be obtained from the family. The nurse needs to know if the lines can be removed or not depending upon the family's response to the question. Asking about bathing the deceased patient is a valid question but is not a priority, because the nurse needs to know the protocol to follow if an autopsy is to be done. Finding out which funeral home the deceased patient is to be transported to is valid but is not a priority, because other actions must be taken before the deceased patient is transported from the hospital. Asking about removing the lines may not be an option depending on the response of the family to an autopsy.

The patient has been brought to the emergency department following a motor vehicle accident. The patient is unresponsive. The driver's license states that glasses are needed to operate a motor vehicle, but no glasses were brought in with the patient. Which action should the nurse take next? a. Stand to the side of the patient's eye and observe the cornea. b. Conclude that the glasses were lost during the accident. c. Notify the ambulance personnel for missing glasses. d. Ask the patient where the glasses are.

A An important aspect of an eye examination is to determine if the patient wears contact lenses, especially in patients who are unresponsive. To determine whether a contact lens is present, stand to the side of the patient's eye and observe the cornea for the presence of a soft or rigid lens. It is also important to observe the sclera to detect the presence of a lens that has shifted off the cornea. An undetected lens causes severe corneal injury when left in place too long. Never assume that glasses were lost or were not worn. Contacting ambulance personnel takes time and cannot assume the glasses are missing. Asking the patient where the glasses are is inappropriate since the patient is unresponsive.

The nurse has just finished teaching a hypertensive patient about the newly prescribed ramipril (Altace). Which patient statement indicates that more teaching is needed? a. A little swelling around my lips and face is okay. b. The medication may not work as well if I take any aspirin. c. The doctor may order a blood potassium level occasionally. d. I will call the doctor if I notice that I have a frequent cough.

A Angioedema occurring with angiotensin-converting enzyme (ACE) inhibitor therapy is an indication that the ACE inhibitor should be discontinued. The patient should be taught that if any swelling of the face or oral mucosa occurs, the health care provider should be immediately notified because this could be life threatening. The other patient statements indicate that the patient has an accurate understanding of ACE inhibitor therapy.

Which action indicates a nurse is using critical thinking for implementation of nursing care to patients? a. Determines whether an intervention is correct and appropriate for the given situation b. Reads over the steps and performs a procedure despite lack of clinical competency c. Establishes goals for a particular patient without assessment d. Evaluates the effectiveness of interventions

A As you implement interventions, use critical thinking to confirm whether the interventions are correct and still appropriate for a patient's clinical situation. You are responsible for having the necessary knowledge and clinical competency to perform interventions for your patients safely and effectively. The nurse needs to recognize the safety hazards of performing an intervention without clinical competency and seek assistance from another nurse. The nurse cannot evaluate interventions until they are implemented. Patients need ongoing assessment before establishing goals because patient conditions can change very rapidly

The nurse is caring for a patient who has undergone surgery for a broken leg and has a cast in place. What should the nurse do to prevent skin impairment? a. Assess surfaces exposed to the edges of the cast for pressure areas. b. Keep the patient's blood pressure low to prevent overperfusion of tissue. c. Do not allow turning in bed because that may lead to redislocation of the leg. d. Restrict the patient's dietary intake to reduce the number of times on the bedpan.

A Assess surfaces exposed to casts, cloth restraints, bandages and dressings, tubing, or orthopedic devices. An external device applied to or around the skin exerts pressure or friction on the skin, leading to skin impairment. When restricted from moving, dependent body parts are exposed to pressure that reduces circulation to affected tissues, promoting pressure ulcers. Patients with limited caloric and protein intake develop impaired or delayed wound healing. Keeping the blood pressure artificially low may decrease arterial blood supply, leading to ischemia and breakdown.

The patient is diagnosed with athlete's foot (tinea pedis). The patient says that he is relieved because it is only athlete's foot, and it can be treated easily. Which information should the nurse consider when formulating a response to the patient? a. Contagious with frequent recurrences b. Helpful to air-dry feet after bathing c. Treated with salicylic acid d. Caused by lice

A Athlete's foot spreads to other body parts, especially the hands. It is contagious and frequently recurs. Drying feet well after bathing and applying powder help prevent infection. It is caused by a fungus, not lice, and is treated with applications of griseofulvin, miconazole, or tolnaftate. Plantar warts are treated with salicylic acid or electrodesiccation.

The nurse working in the dermatology clinic assesses a young adult female patient who is taking isotretinoin (Accutane) to treat severe cystic acne. Which assessment finding is most indicative of a need for further questioning of the patient? a. The patient recently had an intrauterine device removed. b. The patient already has some acne scarring on her forehead. c. The patient has also used topical antibiotics to treat the acne. d. The patient has a strong family history of rheumatoid arthritis.

A Because isotretinoin is teratogenic, contraception is required for women who are using this medication. The nurse will need to determine whether the patient is using other birth control methods. More information about the other patient data may also be needed, but the other data do not indicate contraindications to isotretinoin use.

A patient in the dermatology clinic has a thin, scaly erythematous plaque on the right cheek. Which action should the nurse take? a. Prepare the patient for a biopsy. b. Teach about the use of corticosteroid creams. c. Explain how to apply tretinoin (Retin-A) to the face. d. Discuss the need for topical application of antibiotics.

A Because the appearance of the lesion suggests actinic keratosis or possible squamous cell carcinoma (SCC), the appropriate treatment would be excision and biopsy. Over-the-counter (OTC) corticosteroids, topical antibiotics, and Retin-A would not be used for this lesion.

Which integumentary assessment data from an older patient admitted with bacterial pneumonia is of most concern for the nurse? a. Reports a history of allergic rashes b. Scattered macular brown areas on extremities c. Skin brown and wrinkled, skin tenting on forearm d. Longitudinal nail bed ridges noted; sparse scalp hair

A Because the patient will be receiving antibiotics to treat the pneumonia, the nurse should be most concerned about her history of allergic rashes. The nurse needs to do further assessment of possible causes of the allergic rashes and whether she has ever had allergic reactions to any drugs, especially antibiotics. The assessment data in the other response would be normal for an older patient.

A nurse performs cardiopulmonary resuscitation (CPR) on a 92-year-old with brittle bones and breaks a rib during the procedure, which then punctures a lung. The patient recovers completely without any residual problems and sues the nurse for pain and suffering and for malpractice. Which key point will the prosecution attempt to prove against the nurse? a. The CPR procedure was done incorrectly. b. The patient would have died if nothing was done. c. The patient was resuscitated according to the policy. d. The older patient with brittle bones might sustain fractures when chest compressions are done.

A Certain criteria are necessary to establish nursing malpractice. The prosecution would try to prove that a breach of duty had occurred (CPR done incorrectly), which had caused injury. The defense team, not the prosecution, would explain the correlation between brittle bones and rib fractures during CPR and that the patient was resuscitated according to policy. In this situation, although harm was caused, it was not because of failure of the nurse to perform a duty according to standards, the way other nurses would have performed in the same situation. The fact that the patient sustained injury as a result of age and physical status does not mean the nurse breached any duty to the patient. The nurse would need to make sure the defense attorney knew that the cardiopulmonary resuscitation (CPR) was done correctly. Without intervention, the patient most likely would not have survived.

A patient has scaling of the scalp. Which term will the nurse use to report this finding to the oncoming staff? a. Dandruff b. Alopecia c. Pediculosis d. Xerostomia

A Dandruff is scaling of the scalp that is accompanied by itching. Pediculosis (lice infestation) resides on scalp attached to hair strands; eggs look like oval particles, similar to dandruff. Alopecia is hair loss or balding. Xerostomia is dry mouth.

The nurse is caring for a patient in the postanesthesia care unit. The patient asks for a bedpan and states to the nurse, "I feel like I need to go to the bathroom, but I can't." Which nursing intervention will be most appropriate initially? a. Assess the patient for bladder distention. b. Encourage the patient to wait a minute and try again. c. Inform the patient that everyone feels this way after surgery. d. Call the health care provider to obtain an order for catheterization.

A Depending on the surgery, some patients do not regain voluntary control over urinary function for 6 to 8 hours after anesthesia. Palpate the lower abdomen just above the symphysis pubis for bladder distention. Another option is to use a bladder scan or ultrasound to assess bladder volume. The nurse must assess before deciding if the patient can try again. Not everyone feels as if they need to go but can't after surgery. Calling the health care provider is not the initial best action. The nurse needs to have data before calling the provider.

A nurse is conducting Weber's test. Which action will the nurse take? a. Place a vibrating tuning fork in the middle of patient's forehead. b. Place a vibrating tuning fork on the patient's mastoid process. c. Compare the number of seconds heard by bone versus air conduction. d. Compare the patient's degree of joint movement to the normal level.

A During Weber's test (lateralization of sound), the nurse places the vibrating tuning fork in the middle of the patient's forehead. During a Rinne test (comparison of air and bone conduction), the nurse places a vibrating tuning fork on the patient's mastoid process and compares the length of time air and bone conduction is heard. Comparing the patient's degree of joint movement to the normal level is a test for range of motion.

The nurse is caring for an older-adult patient with Alzheimer's disease who is ambulatory but requires total assistance with activities of daily living (ADLs). The nurse notices that the patient is edentulous. Which area should the nurse assess? a. Assess oral cavity. b. Assess room for drafts. c. Assess ankles for edema. d. Assess for reduced sensations.

A Edentulous means without teeth; therefore, the nurse needs to assess the oral cavity. While older adults may want the room warmer and drafts should be avoided, this does not help with being edentulous. Edentulous does not mean the patient has edema. While older-adult patients can have reduced sensations, this is not the meaning of edentulous.

The nurse is caring for a group of patients. Which patient will the nurse see first? a. A patient who had cataract surgery is coughing. b. A patient who had vascular repair of the right leg is not doing right leg exercises. c. A patient after knee surgery is wearing intermittent pneumatic compression devices and receiving heparin. d. A patient after surgery has vital signs taken every 15 minutes twice, every 30 minutes twice, hourly for 2 hours then every 4 hours.

A For patients who have had eye, intracranial, or spinal surgery, coughing may be contraindicated because of the potential increase in intraocular or intracranial pressure. The nurse will need to see this patient first to control the cough and intraocular pressure. All the rest are normal postoperative patients. Leg exercise should not be performed on the operative leg with vascular surgery. A patient after knee surgery should receive heparin and be wearing intermittent pneumatic compression devices; while the nurse will check on the patient, it does not have to be first. Monitoring vital signs after surgery is required and this is the standard schedule.

The debilitated patient is resisting attempts by the nurse to provide oral hygiene. Which action will the nurse take next? a. Insert an oral airway. b. Place the patient in a flat, supine position. c. Use undiluted hydrogen peroxide as a cleaner. d. Quickly proceed while not talking to the patient.

A If the patient is uncooperative, or is having difficulty keeping the mouth open, insert an oral airway. Insert it upside down, and then turn the airway sideways and over the tongue to keep the teeth apart. Do not use force. Position the patient on his or her side or turn the head to allow for drainage. Placing the patient in a flat, supine position could lead to aspiration. Hydrogen peroxide is irritating to mucosa. Even though the patient is debilitated, explain the steps of mouth care and the sensations that he or she will feel. Also tell the patient when the procedure is completed.

The nurse is bathing a patient and notices movement in the patient's hair. Which action will the nurse take? a. Use gloves to inspect the hair. b. Apply a lindane-based shampoo immediately. c. Shave the hair off of the patient's head. d. Ignore the movement and continue.

A In community health and home care settings, it is particularly important to inspect the hair for lice so appropriate hygienic treatment can be provided. If pediculosis capitis (head lice) is suspected, the nurse must protect self against self-infestations by handwashing and by using gloves or tongue blades to inspect the patient's hair. Suspicions cannot be ignored. Shaving hair off affected areas is the treatment for pediculosis pubis (crab lice) and is rarely used for head lice. Caution against use of products containing lindane because the ingredient is toxic and known to cause adverse reactions.

A 17-year-old patient, dying of heart failure, wants to have organs removed for transplantation after death. Which action by the nurse is correct? a. Instruct the patient to talk with parents about the desire to donate organs. b. Notify the health care provider about the patient's desire to donate organs. c. Prepare the organ donation form for the patient to sign while still oriented. d. Contact the United Network for Organ Sharing after talking with the patient.

A In this situation, the parents would need to sign the form because the teenager is under age 18. An individual who is at least 18 may sign the form allowing organ donation upon death. The nurse cannot allow the patient to sign the organ donation document because the patient is younger than age 18. The health care provider will be notified about the patient's wishes after the parents agree to donate the organs. The United Network for Organ Sharing (UNOS) has a contract with the federal government and sets policies and guidelines for the procurement of organs.

The nurse is caring for a patient in the postanesthesia care unit who has undergone a left total knee arthroplasty. The anesthesia provider has indicated that the patient received a left femoral peripheral nerve block. Which assessment will be an expected finding for this patient? a. Sensation decreased in the left leg b. Patient report of pain in the left foot c. Pulse decreased at the left posterior tibia d. Left toes cool to touch and slightly cyanotic

A Induction of regional anesthesia results in loss of sensation in an area of the body—in this case, the left leg. The peripheral nerve block influences the portions of sensory pathways that are anesthetized in the targeted area of the body. Decreased pulse, toes cool to touch, and cyanosis are indications of decreased blood flow and are not expected findings. Reports of pain in the left foot may indicate that the block is not working or is subsiding and is not an expected finding in the immediate postoperative period

When assessing a patient's feet, the nurse notices that the toenails are thick and separated from the nail bed. What does the nurse most likely suspect is the cause of this condition? a. Fungi b. Friction c. Nail polish d. Nail polish remover

A Inflammatory lesions and fungus of the nail bed cause thickened, horny nails that separate from the nail bed. Ask women whether they frequently polish their nails and use polish remover because chemicals in these products cause excessive nail dryness. Friction and pressure from ill-fitting or loose shoes causes keratosis (corns). It is seen mainly on or between toes, over bony prominences.

The nurse is completing a medication history for the surgical patient in preadmission testing. Which medication should the nurse instruct the patient to hold (discontinue) in preparation for surgery according to protocol? a. Warfarin b. Vitamin C c. Prednisone d. Acetaminophen

A Medications such as warfarin or aspirin alter normal clotting factors and thus increase the risk of hemorrhaging. Discontinue at least 48 hours before surgery. Acetaminophen is a pain reliever that has no special implications for surgery. Vitamin C actually assists in wound healing and has no special implications for surgery. Prednisone is a corticosteroid, and dosages are often temporarily increased rather than held.

The nurse is revising the care plan. In which order will the nurse perform the tasks, beginning with the first step? 1. Revise specific interventions. 2. Revise the assessment column. 3. Choose the evaluation method. 4. Delete irrelevant nursing diagnoses. a. 2, 4, 1, 3 b. 4, 2, 1, 3 c. 3, 4, 2, 1 d. 4, 2, 3, 1

A Modification of an existing written care plan includes four steps: 1. Revise data in the assessment column to reflect the patient's current status. Date any new data to inform other members of the health care team of the time that the change occurred. 2. Revise the nursing diagnoses. Delete nursing diagnoses that are no longer relevant and add and date any new diagnoses. Revise related factors and the patient's goals, outcomes, and priorities. Date any revisions. 3. Revise specific interventions that correspond to the new nursing diagnoses and goals. Be sure that revisions reflect the patient's present status. 4. Choose the method of evaluation for determining whether you achieved patient outcomes.

A patient in the intensive care unit with acute decompensated heart failure (ADHF) complains of severe dyspnea and is anxious, tachypneic, and tachycardic. All of the following medications have been ordered for the patient. The nurses priority action will be to a. give IV morphine sulfate 4 mg. b. give IV diazepam (Valium) 2.5 mg. c. increase nitroglycerin (Tridil) infusion by 5 mcg/min. d. increase dopamine (Intropin) infusion by 2 mcg/kg/min.

A Morphine improves alveolar gas exchange, improves cardiac output by reducing ventricular preload and afterload, decreases anxiety, and assists in reducing the subjective feeling of dyspnea. Diazepam may decrease patient anxiety, but it will not improve the cardiac output or gas exchange. Increasing the dopamine may improve cardiac output, but it will also increase the heart rate and myocardial oxygen consumption. Nitroglycerin will improve cardiac output and may be appropriate for this patient, but it will not directly reduce anxiety and will not act as quickly as morphine to decrease dyspnea.

A staff development nurse is providing an inservice for other nurses to educate them about the Nursing Interventions Classification (NIC) system. During the inservice, which statement made by one of the nurses in the room requires the staff development nurse to clarify the information provided? a. "This system can help medical students determine the cost of the care they provide to patients." b. "If the nursing department uses this system, communication among nurses who work throughout the hospital may be enhanced." c. "We could use this system to help organize orientation for new nursing employees because we can better explain the nursing interventions we use most frequently on our unit." d. "The NIC system provides one way to improve safe and effective documentation in the hospital's electronic health record."

A NIC does not help determine the cost of services provided by nurses. The staff development nurse would need to correct this misconception. Because this system is specific to nursing practice, it would not help medical students determine the costs of care. The NIC system developed by the University of Iowa differentiates nursing practice from that of other health care disciplines. All the other statements are true. Benefits of using NIC include enhancing communication among nursing staff and documentation, especially within health information systems such as an electronic documentation system. NIC also helps nurses identify the nursing interventions they implement most frequently. Units that identify routine nursing interventions can use this information to develop checklists for orientation.

Which action should the nurse include in the plan of care when caring for a patient admitted with acute decompensated heart failure (ADHF) who is receiving nesiritide (Natrecor)? a. Monitor blood pressure frequently. b. Encourage patient to ambulate in room. c. Titrate nesiritide slowly before stopping. d. Teach patient about home use of the drug.

A Nesiritide is a potent arterial and venous dilator, and the major adverse effect is hypotension. Because the patient is likely to have orthostatic hypotension, the patient should not be encouraged to ambulate. Nesiritide does not require titration and is used for ADHF but not in a home setting.

Which nurse most likely kept records on sanitation techniques and the effects on health? a. Florence Nightingale b. Mary Nutting c. Clara Barton d. Lillian Wald

A Nightingale was the first practicing nurse epidemiologist. Her statistical analyses connected poor sanitation with cholera and dysentery. Mary Nutting, Clara Barton, and Lillian Wald came after Nightingale, each contributing to the nursing profession in her own way. - Mary Nutting was instrumental in moving nursing education into universities. - Clara Barton founded the American Red Cross. - Lillian Wald helped open the Henry Street Settlement.

The nurse is preparing for a rectal examination of a nonambulatory male patient. In which position will the nurse place the patient? a. Sims' b. Knee-chest c. Dorsal recumbent d. Forward bending with flexed hips

A Nonambulatory patients are best examined in a side-lying Sims' position. Forward bending would require the patient to be able to stand upright. Knees to chest would be difficult to maintain in a nonambulatory male and is embarrassing and uncomfortable. Dorsal recumbent does not provide adequate access for a rectal examination and is used for abdominal assessment because it promotes relaxation of abdominal muscles.

Propranolol (Inderal) is prescribed for a patient diagnosed with hypertension. The nurse should consult with the health care provider before giving this medication when the patient reveals a history of a. asthma. b. daily alcohol use. c. peptic ulcer disease. d. myocardial infarction (MI).

A Nonselective b-blockers block b1 and b2-adrenergic receptors and can cause bronchospasm, especially in patients with a history of asthma. b-Blockers will have no effect on the patients peptic ulcer disease or alcohol use. b-Blocker therapy is recommended after MI.

The patient reports to the nurse of being afraid to speak up regarding a desire to end care for fear of upsetting spouse and children. Which principle in the nursing code of ethics ensures that the nurse will promote the patient's cause? a. Advocacy b. Responsibility c. Confidentiality d. Accountability

A Nurses advocate for patients when they support the patient's cause. A nurse's ability to adequately advocate for a patient is based on the unique relationship that develops and the opportunity to better understand the patient's point of view. Responsibility refers to respecting one's professional obligations and following through on promises. Confidentiality deals with privacy issues, and accountability refers to answering for one's actions.

The nurse is assessing a postoperative patient with a history of obstructive sleep apnea for airway obstruction. Which assessment finding will best alert the nurse to this complication? a. Drop in pulse oximetry readings b. Moaning with reports of pain c. Shallow respirations d. Disorientation

A One of the greatest concerns after general anesthesia is airway obstruction, especially in patients with obstructive sleep apnea. A drop in oxygen saturation by pulse oximetry is a sign of airway obstruction in patients with obstructive sleep apnea. Weak pharyngeal/laryngeal muscle tone from anesthetics; secretions in the pharynx, bronchial tree, or trachea; and laryngeal or subglottic edema also contribute to airway obstruction. In the postanesthetic patient, the tongue is a major cause of airway obstruction. Shallow respirations are indicative of respiratory depression. Moaning and reports of pain are common in all surgical patients and are an expected event. Disorientation is common when first awakening from anesthesia but can be a sign of hypoxia.

The nurse is providing preoperative teaching for the ambulatory surgery patient who will be having a cyst removed from the right arm. Which will be the best explanation for diet progression after surgery? a. "Start with clear liquids, soup, and crackers. Advance to a normal diet as tolerated." b. "Stay with ice chips for several hours. After that, you can have whatever you want." c. "Stay on clear liquids for 24 hours. Then you can progress to a normal diet." d. "Start with clear liquids for 2 hours and then full liquids for 2 hours. Then progress to a normal diet."

A Patients usually receive a normal diet the first evening after surgery unless they have undergone surgery on GI structures. Implement diet intake while judging the patient's response. For example, provide clear liquids such as water, apple juice, broth, or tea after nausea subsides. If the patient tolerates liquids without nausea, advance the diet as ordered. There is no need to stay on ice chips for several hours or clear liquids for 24 hours after this procedure. Putting a time frame on the progression is too prescriptive. Progression should be adjusted for the patient's needs.

The nurse is caring for a patient who has diabetes mellitus and circulatory insufficiency, with peripheral neuropathy and urinary incontinence. On which areas does the nurse focus care? a. Decreased pain sensation and increased risk of skin impairment b. Decreased caloric intake and accelerated wound healing c. High risk for skin infection and low saliva pH level d. High risk for impaired venous return and dementia

A Patients with paralysis, circulatory insufficiency, or peripheral neuropathy (nerve damage) are unable to sense an injury to the skin (decreased pain sensation). The presence of urinary incontinence, circulatory insufficiency, and neuropathy can combine to result in breakdown, so the patient has an increased risk of skin impairment. While the patient may have decreased caloric intake, the patient will not have accelerated wound healing with circulatory insufficiency, neuropathy, and incontinence. While the patient is at high risk for skin infection, the low salivary pH level is not an issue. While the patient may have a high risk for impaired venous return from the circulatory insufficiency, there is no indication the patient has dementia.

The nurse is preparing to assist the patient in using the incentive spirometer. Which nursing intervention should the nurse provide first? a. Perform hand hygiene. b. Explain use of the mouthpiece. c. Instruct the patient to inhale slowly. d. Place in the reverse Trendelenburg position.

A Performing hand hygiene reduces microorganisms and should be performed first. Placing the patient in the correct position such as high Fowler's for the typical postoperative patient or reverse Trendelenburg for the bariatric patient would be the next step in the process. Demonstration of use of the mouthpiece followed by the instruction to inhale slowly would be the last step in this scenario.

Which nursing assessment will indicate the patient is performing diaphragmatic breathing correctly? a. Hands placed on the border of the rib cage with fingers extended will touch as the chest wall contracts. b. Hands placed on the chest wall with fingers extended will separate as the chest wall contracts. c. The patient will feel upward movement of the diaphragm during inspiration. d. The patient will feel downward movement of the diaphragm during expiration.

A Positioning the hands along the borders of the rib cage allows the patient to feel movement of the chest and abdomen as the diaphragm descends and the lungs expand. As the patient takes a deep breath and slowly exhales, the middle fingers will touch while the chest wall contracts. The fingers will separate as the chest wall expands. The patient will feel normal downward movement of the diaphragm during inspiration and normal upward movement during expiration.

Which initial intervention is most appropriate for a patient who has a new onset of chest pain? a. Reassess the patient. b. Notify the health care provider. c. Administer a prn medication for pain. d. Call radiology for a portable chest x-ray.

A Preparation for implementation ensures efficient, safe, and effective nursing care; the first activity is reassessment. The cause of the patient's chest pain is unknown, so the patient needs to be reassessed before pain medication is administered or a chest x-ray is obtained. The nurse then notifies the patient's health care provider of the patient's current condition in anticipation of receiving further orders. The patient's chest pain could be due to muscular injury or a pulmonary issue. The nurse needs to reassess first.

A nurse identifies lice during a child's scalp assessment. The nurse teaches the parents about hair care. Which information from the parents indicates the nurse needs to follow up? a. We will use lindane-based shampoos. b. We will use the sink to wash hair. c. We will use a fine-toothed comb. d. We will use a vinegar hair rinse.

A Products containing lindane, a toxic ingredient, often cause adverse reactions; the nurse will need to follow up to correct the misconception. All the rest are correct. Instruct parents who have children with head lice to shampoo thoroughly with pediculicide (shampoo available at drugstores) in cold water at a basin or sink, comb thoroughly with a fine-toothed comb, and discard the comb. A dilute solution of vinegar and water helps loosen nits.

The patient is a 45-year-old African-American male who has come in for a routine annual physical. Which type of preventive screening does the nurse discuss with the patient? a. Digital rectal examination of the prostate b. Complete eye examination every year c. CA 125 blood test once a year d. Colonoscopy every 3 years

A Recommended preventive screenings include a digital rectal examination of the prostate and prostate-specific antigen test starting at age 50. CA 125 blood tests are indicated for women at high risk for ovarian cancer. Patients over the age of 65 need to have complete eye examinations yearly. Colonoscopy every 10 years is recommended in patients 50 years of age and older

Which action by the nurse indicates a safe and efficient use of social networks? a. Promotes support for a local health charity b. Posts a picture of a patient's infected foot c. Vents about a patient problem at work d. Friends a patient

A Social networks can be a supportive source of information about patient care or professional nursing activities. Even if you post an image of a patient without any obvious identifiers, the nature of shared media reposting can result in the image surfacing in a place where just the context of the image provides clues for friends or family to identify the patient. The ANA and NCSBN states, "Effective nurse-patient relationships are built on trust. Patients need to be confident that their most personal information and their basic dignity will be protected by the nurse." Becoming friends in online chat rooms, Facebook, or other public sites can interfere with your ability to maintain a therapeutic relationship.

The nurse is reviewing the surgical consent with the patient during preoperative education and finds the patient does not understand what procedure will be completed. What is the nurse's best next step? a. Notify the health care provider about the patient's question. b. Explain the procedure that will be completed. c. Continue with preoperative education. d. Ask the patient to sign the form.

A Surgery cannot be legally or ethically performed until the patient fully understands the need for a procedure and all the implications. It is the surgeon's responsibility to explain the procedure, associated risks, benefits, alternatives, and possible complications. It is important for the nurse to pause with preoperative education to notify the health care provider of the patient's questions. It is not within the nurse's scope to explain the procedure. The nurse can certainly reinforce what the health care provider has explained, but the information needs to come from the health care provider. It is not prudent to ask a patient to sign a form for a procedure that he/she does not understand.

The nurse is intervening for a patient that has a risk for a urinary infection. Which direct care nursing intervention is most appropriate? a. Teaches proper handwashing technique b. Properly cleans the patient's toilet c. Transports urine specimen to the lab d. Informs the oncoming nurse during hand-off

A Teaching proper handwashing technique is a direct care nursing intervention. All the rest are indirect nursing care: cleaning the toilet, transporting specimens, and performing hand-off reports.

The charge nurse observes a new registered nurse (RN) doing discharge teaching for a patient with hypertension who has a new prescription for enalapril (Vasotec). The charge nurse will need to intervene if the new RN tells the patient to a. increase the dietary intake of high-potassium foods. b. make an appointment with the dietitian for teaching. c. check the blood pressure (BP) with a home BP monitor at least once a day. d. move slowly when moving from lying to sitting to standing.

A The ACE inhibitors cause retention of potassium by the kidney, so hyperkalemia is a possible adverse effect. The other teaching by the new RN is appropriate for a patient with newly diagnosed hypertension who has just started therapy with enalapril.

The nurse assesses a patient who had a total abdominal hysterectomy 2 days ago. Which information about the patient is most important to communicate to the health care provider? a. The right calf is swollen, warm, and painful. b. The patients temperature is 100.3 F (37.9 C). c. The 24-hour oral intake is 600 mL greater than the total output. d. The patient complains of abdominal pain at level 6 (0 to 10 scale) when ambulating.

A The calf pain, swelling, and warmth suggest that the patient has a deep vein thrombosis, which will require the health care provider to order diagnostic tests and/or anticoagulants. Because the stress response causes fluid retention for the first 2 to 5 days postoperatively, the difference between intake and output is expected. A temperature elevation to 100.3 F on the second postoperative day suggests atelectasis, and the nurse should have the patient deep breathe and cough. Pain with ambulation is normal, and the nurse should administer the ordered analgesic before patient activities

A parent calls the school nurse with questions regarding the recent school vision screening. Snellen chart examination revealed 20/60 for both eyes. Which response by the nurse is the best regarding the eye examination results? a. Your child needs to see an ophthalmologist. b. Your child is suffering from strabismus. c. Your child may have presbyopia. d. Your child has cataracts.

A The child needs an eye examination with an ophthalmologist or optometrist. Normal vision is 20/20. The larger the denominator, the poorer the patient's visual acuity. For example, a value of 20/60 means that the patient, when standing 20 feet away, can read a line that a person with normal vision can read from 60 feet away. Strabismus is a (congenital) condition in which both eyes do not focus on an object simultaneously: The eyes appear crossed. Acuity may not be affected; Snellen test does not test for strabismus. Presbyopia is impaired near vision that occurs in middle-aged and older adults and is caused by loss of elasticity of the lens. Cataracts, a clouding of the lens, develop slowly and progressively after age 35 or suddenly after trauma.

A nurse is using a guide that provides principles of right and wrong to provide care to patients. Which guide is the nurse using? a. Code of ethics b. Standards of practice c. Standards of professional performance d. Quality and safety education for nurses

A The code of ethics is the philosophical ideals of right and wrong that define the principles you will use to provide care to your patients. The Standards of Practice describe a competent level of nursing care. The ANA Standards of Professional Performance describe a competent level of behavior in the professional role. Quality and safety education for nurses addresses the challenge to prepare nurses with the competencies needed to continuously improve the quality of care in their work environments.

There is one opening in the schedule at the dermatology clinic, and 4 patients are seeking appointments today. Which patient will the nurse schedule for the available opening? a. 38-year old with a 7-mm nevus on the face that has recently become darker b. 62-year-old with multiple small, soft, pedunculated papules in both axillary areas c. 42-year-old with complaints of itching after using topical fluorouracil on the nose d. 50-year-old with concerns about skin redness after having a chemical peel 3 days ago

A The description of the lesion is consistent with possible malignant melanoma. This patient should be assessed as soon as possible by the health care provider. Itching is common after using topical fluorouracil and redness is an expected finding a few days after a chemical peel. Skin tags are common, benign lesions after midlife.

The nurse is reviewing the laboratory test results for a patient who has recently been diagnosed with hypertension. Which result is most important to communicate to the health care provider? a. Serum creatinine of 2.8 mg/dL b. Serum potassium of 4.5 mEq/L c. Serum hemoglobin of 14.7 g/dL d. Blood glucose level of 96 mg/dL

A The elevated creatinine indicates renal damage caused by the hypertension. The other laboratory results are normal.

A postoperative patient has not voided for 8 hours after return to the clinical unit. Which action should the nurse take first? a. Perform a bladder scan. b. Encourage increased oral fluid intake. c. Assist the patient to ambulate to the bathroom. d. Insert a straight catheter as indicated on the PRN order.

A The initial action should be to assess the bladder for distention. If the bladder is distended, providing the patient with privacy (by walking with them to the bathroom) will be helpful. Because of the risk for urinary tract infection, catheterization should only be done after other measures have been tried without success. There is no indication to notify the surgeon about this common postoperative problem unless all measures to empty the bladder are unsuccessful

Which nursing action should the nurse take first in order to assist a patient with newly diagnosed stage 1 hypertension in making needed dietary changes? a. Collect a detailed diet history. b. Provide a list of low-sodium foods. c. Help the patient make an appointment with a dietitian. d. Teach the patient about foods that are high in potassium.

A The initial nursing action should be assessment of the patients baseline dietary intake through a thorough diet history. The other actions may be appropriate, but assessment of the patients baseline should occur first.

A male student comes to the college health clinic. He hesitantly describes that he found something wrong with his testis when taking a shower. Which assessment finding will alert the nurse to possible testicular cancer? a. Hard, pea-sized testicular lump b. Rubbery texture of testes c. Painful enlarged testis d. Prolonged diuretic use

A The most common symptoms of testicular cancer are a painless enlargement of one testis and the appearance of a palpable, small, hard lump, about the size of a pea, on the front or side of the testicle. Normally, the testes feel smooth, rubbery, and free of nodules. Use of diuretics, sedatives, or antihypertensives can lead to erection or ejaculation problems

The nurse has completed a preoperative assessment for a patient going to surgery and gathers assessment data. Which will be the most important next step for the nurse to take? a. Notify the operating suite that the patient has a latex allergy. b. Document that the patient had a bath at home this morning. c. Administer the ordered preoperative intravenous antibiotic. d. Ask the nursing assistive personnel to obtain vital signs.

A The most important step is notifying the operating suite of the patient's latex allergy. Many products that contain latex are used in the operating suite and the postanesthesia care unit (PACU). When preparing for a patient with this allergy, special considerations are required from preparation of the room to the types of tubes, gloves, drapes, and instruments utilized. Obtaining vital signs, documenting, and administering medications are all part of the process and should be done—with the latex allergy in mind. However, making sure that the patient has a safe environment is the first step.

A home health nurse notices that a patient's preschool children are often playing on the sidewalk and in the street unsupervised and repeatedly takes them back to the home and talks with the patient, but the situation continues. Which immediate action by the nurse is mandated by law? a. Contact the appropriate community child protection facility. b. Tell the parents that the authorities will be contacted shortly. c. Take pictures of the children to support the overt child abuse. d. Discuss with both parents about the safety needs of their children.

A The nurse has a duty to report this situation to protect the children. Any health care professional who does not report suspected child abuse or neglect may be liable for civil or criminal legal action. Talking with both parents is not mandated by law. There is no obligation to tell the parents that they will be reported to authorities. There is no obligation for the nurse to take pictures of the children.

The surgical unit nurse has just received a patient with a history of smoking from the post-anesthesia care unit. Which action is most important at this time? a. Auscultate for adventitious breath sounds. b. Obtain the patients blood pressure and temperature. c. Remind the patient about harmful effects of smoking. d. Ask the health care provider about prescribing a nicotine patch.

A The nurse should first ensure a patent airway and check for breathing and circulation (airway, breathing, and circulation [ABCs]). Circulation and temperature can be assessed after a patent airway and breathing have been established. The immediate postoperative period is not the optimal time for patient teaching about the harmful effects of surgery. Requesting a nicotine patch may be appropriate, but is not a priority at this time.

The nurse is caring for a postoperative patient who has had a minimally invasive carpel tunnel repair. The patient has a temperature of 97° F and is shivering. Which reason will the nurse most likely consider as the primary cause when planning care? a. Anesthesia lowers metabolism. b. Surgical suites have air currents. c. The patient is dressed only in a gown. d. The large open body cavity contributed to heat loss.

A The operating suite and recovery room environments are extremely cool. The patient's anesthetically depressed level of body function results in lowering of metabolism and a fall in body temperature. Although the patient is dressed in a gown and there are air currents in the operating room, these are not the primary reasons for the low temperature. Also, the patient in this type of case does not have a large open body cavity to contribute to heat loss.

The nurse working in the postanesthesia care unit (PACU) notes that a patient who has just been transported from the operating room is shivering and has a temperature of 96.5 F (35.8 C). Which action should the nurse take? a. Cover the patient with a warm blanket and put on socks. b. Notify the anesthesia care provider about the temperature. c. Avoid the use of opioid analgesics until the patient is warmer. d. Administer acetaminophen (Tylenol) 650 mg suppository rectally.

A The patient assessment indicates the need for active rewarming. There is no indication of a need for acetaminophen. Opioid analgesics may help reduce shivering. Because hypothermia is common in the immediate postoperative period, there is no need to notify the anesthesia care provider, unless the patient continues to be hypothermic after active rewarming.

Which information should the nurse include when teaching a patient who has just received a prescription for ciprofloxacin (Cipro) to treat a urinary tract infection? a. Use a sunscreen with a high SPF when exposed to the sun. b. Sun exposure may decrease the effectiveness of the medication. c. Photosensitivity may result in an artificial-looking tan appearance. d. Wear sunglasses to avoid eye damage while taking this medication.

A The patient should stay out of the sun. If that is not possible, teach them to wear sunscreen when taking medications that can cause photosensitivity. The other statements are not accurate.

After receiving change-of-shift report on a heart failure unit, which patient should the nurse assess first? a. A patient who is cool and clammy, with new-onset confusion and restlessness b. A patient who has crackles bilaterally in the lung bases and is receiving oxygen. c. A patient who had dizziness after receiving the first dose of captopril (Capoten) d. A patient who is receiving IV nesiritide (Natrecor) and has a blood pressure of 100/62

A The patient who has wet-cold clinical manifestations of heart failure is perfusing inadequately and needs rapid assessment and changes in management. The other patients also should be assessed as quickly as possible but do not have indications of severe decreases in tissue perfusion.

The nurse on the intermediate care unit received change-of-shift report on four patients with hypertension. Which patient should the nurse assess first? a. 43-year-old with a blood pressure (BP) of 160/92 who is complaining of chest pain b. 52-year-old with a BP of 212/90 who has intermittent claudication c. 50-year-old with a BP of 190/104 who has a creatinine of 1.7 mg/dL d. 48-year-old with a BP of 172/98 whose urine shows microalbuminuria

A The patient with chest pain may be experiencing acute myocardial infarction, and rapid assessment and intervention are needed. The symptoms of the other patients also show target organ damage but are not indicative of acute processes.

After receiving change-of-shift report about these postoperative patients, which patient should the nurse assess first? a. Obese patient who had abdominal surgery 3 days ago and whose wound edges are separating b. Patient who has 30 mL of sanguineous drainage in the wound drain 10 hours after hip replacement surgery c. Patient who has bibasilar crackles and a temperature of 100F (37.8C) on the first postoperative day after chest surgery d. Patient who continues to have incisional pain 15 minutes after hydrocodone and acetaminophen (Vicodin) administration

A The patients history and assessment suggests possible wound dehiscence, which should be reported immediately to the surgeon. Although the information about the other patients indicates a need for ongoing assessment and/or possible intervention, the data do not suggest any acute complications. Small amounts of red drainage are common in the first postoperative hours. Bibasilar crackles and a slightly elevated temperature are common after surgery, although the nurse will need to have the patient cough and deep breathe. Oral medications typically take more than 15 minutes for effective pain relief.

A patient scheduled for an elective hysterectomy tells the nurse, I am afraid that I will die in surgery like my mother did! Which response by the nurse is most appropriate? a. Tell me more about what happened to your mother. b. You will receive medications to reduce your anxiety. c. You should talk to the doctor again about the surgery. d. Surgical techniques have improved a lot in recent years.

A The patients statement may indicate an unusually high anxiety level or a family history of problems such as malignant hyperthermia, which will require precautions during surgery. The other statements may also address the patients concerns, but further assessment is needed first.

During a visit to a 78-year-old with chronic heart failure, the home care nurse finds that the patient has ankle edema, a 2-kg weight gain over the past 2 days, and complains of feeling too tired to get out of bed. Based on these data, the best nursing diagnosis for the patient is a. activity intolerance related to fatigue. b. disturbed body image related to weight gain. c. impaired skin integrity related to ankle edema. d. impaired gas exchange related to dyspnea on exertion.

A The patients statement supports the diagnosis of activity intolerance. There are no data to support the other diagnoses, although the nurse will need to assess for other patient problems.

A patient who had knee surgery received intramuscular ketorolac (Toradol) 30 minutes ago and continues to complain of pain at a level of 7 (0 to 10 scale). Which action is best for the nurse to take at this time? a. Administer the prescribed PRN IV morphine sulfate. b. Notify the health care provider about the ongoing knee pain. c. Reassure the patient that postoperative pain is expected after knee surgery. d. Teach the patient that the effects of ketorolac typically last about 6 to 8 hours

A The priority at this time is pain relief. Concomitant use of opioids and nonsteroidal antiinflammatory drugs (NSAIDs) improves pain control in postoperative patients. Patient teaching and reassurance are appropriate, but should be done after the patients pain is relieved. If the patient continues to have pain after the morphine is administered, the health care provider should be notified.

During preoperative assessment for a 7:30 AM (0730) surgery, the nurse finds the patient drank a cup of coffee this morning. The nurse reports this information to the anesthesia provider. Which action does the nurse anticipate next? a. A delay in or cancellation of surgery b. Questions regarding components of the coffee c. Additional questions about why the patient had coffee d. Instructions to determine what education was provided in the preoperative visit

A The recommendations before nonemergent procedures requiring general and regional anesthesia or sedation/analgesia include fasting from intake of clear liquids for 2 or more hours. A delay in or cancellation of surgery will be in order for this case. Questions regarding components of the coffee, asking why, and evaluating the preoperative education may all be items to be addressed, especially from a performance improvement perspective, but at this time in caring for this patient, a delay or cancellation is in order to prevent aspiration

The nurse demonstrates postoperative exercises for a patient. In which order will the nurse instruct the patient to perform the exercises? 1. Turning 2. Breathing 3. Coughing 4. Leg exercises a. 4, 1, 2, 3 b. 1, 2, 3, 4 c. 2, 3, 4, 1 d. 3, 1, 4, 2

A The sequence of exercises is leg exercises, turning, breathing, and coughing

The health care provider diagnoses impetigo in a patient who has crusty vesicopustular lesions on the lower face. Which instructions should the nurse include in the teaching plan? a. Clean the infected areas with soap and water. b. Apply alcohol-based cleansers on the lesions. c. Avoid use of antibiotic ointments on the lesions. d. Use petroleum jelly (Vaseline) to soften crusty areas.

A The treatment for impetigo includes softening of the crusts with warm saline soaks and then soap-and-water removal. Alcohol-based cleansers and use of petroleum jelly are not recommended for impetigo. Antibiotic ointments, such as mupirocin (Bactroban), may be applied to the lesions.

A patient who is scheduled for a therapeutic abortion tells the nurse, Having an abortion is not right. Which functional health pattern should the nurse further assess? a. Value-belief b. Cognitive-perceptual c. Sexuality-reproductive d. Coping-stress tolerance

A The value-belief pattern includes information about conflicts between a patients values and proposed medical care. In the cognitive-perceptual pattern, the nurse will ask questions about pain and sensory intactness. The sexuality-reproductive pattern includes data about the impact of the surgery on the patients sexuality. The coping-stress tolerance pattern assessment will elicit information about how the patient feels about the surgery.

A patient with atopic dermatitis has been using a high-potency topical corticosteroid ointment for several weeks. The nurse should assess for which adverse effect? a. Thinning of the affected skin b. Alopecia of the affected areas c. Reddish-brown discoloration of the skin d. Dryness and scaling in the areas of treatment

A Thinning of the skin indicates that atrophy, a possible adverse effect of topical corticosteroids, is occurring. The health care provider should be notified so that the medication can be changed or tapered. Alopecia, red- brown discoloration, and dryness/scaling of the skin are not adverse effects of topical corticosteroid use

The nurse is caring for a patient who has multiple ticks on lower legs and body. What should the nurse do to rid the patient of ticks? a. Use blunt tweezers and pull upward with steady pressure. b. Burn the ticks with a match or small lighter. c. Allow the ticks to drop off by themselves. d. Apply miconazole and cover with plastic.

A Using blunt tweezers, grasp the tick as close to the head as possible and pull upward with even, steady pressure. Hold until the tick pulls out, usually for about 3 to 4 minutes. Save the tick in a plastic bag, and put it in the freezer if necessary to identify the type of tick. Because ticks transmit several diseases to people, they must be removed. Allowing them to drop off by themselves is not an option. Do not burn ticks off with a match or lighter. Miconazole is used to treat athlete's foot; it is a fungal medication. Covering ticks with plastic does not remove ticks

A febrile preschool-aged child presents to the after-hours clinic. Varicella (chickenpox) is diagnosed on the basis of the illness history and the presence of small, circumscribed skin lesions filled with serous fluid. Which type of skin lesion will the nurse report? a. Vesicles b. Wheals c. Papules d. Pustules

A Vesicles are circumscribed, elevated skin lesions filled with serous fluid that measure less than 1 cm. - Wheals are irregularly shaped, elevated areas of superficial localized edema that vary in size. They are common with mosquito bites and hives. - Papules are palpable, circumscribed, solid elevations in the skin that are smaller than 1 cm. - Pustules are elevations of skin similar to vesicles, but they are filled with pus and vary in size like acne.

The nurse finds it difficult to care for a patient whose advance directive states that no extraordinary resuscitation measures should be taken. Which step may help the nurse to find resolution in this assignment? a. Scrutinize personal values. b. Call for an ethical committee consult. c. Decline the assignment on religious grounds. d. Convince the family to challenge the directive.

A Clarifying values—your own, your patients', your co-workers'—is an important and effective part of ethical discourse. Calling for a consult, declining the assignment, and convincing the family to challenge the patient's directive are not ideal resolutions because they do not address the reason for the nurse's discomfort, which is the conflict between the nurse's values and those of the patient. The nurse should value the patient's decisions over the nurse's personal values.

The patient is reporting an inability to clear nasal passages. Which action will the nurse take? a. Use gentle suction to prevent tissue damage. b. Instruct patient to blow nose forcefully to clear the passage. c. Place a dry washcloth under the nose to absorb secretions. d. Insert a cotton-tipped applicator to the back of the nose.

A Excessive nasal secretions can be removed using gentle suctioning. However, patients usually remove secretions from the nose by gentle blowing into a soft tissue. Caution the patient against harsh blowing that creates pressure capable of injuring the eardrum, the nasal mucosa, and even sensitive eye structures. If the patient is unable to remove nasal secretions, assist by using a wet washcloth or a cotton-tipped applicator moistened in water or saline. Never insert the applicator beyond the length of the cotton tip.

Four patients in labor all request epidural analgesia to manage their pain at the same time. Which ethical principle is most compromised when only one nurse anesthetist is on call? a. Justice b. Fidelity c. Beneficence d. Nonmaleficence

A Justice refers to fairness and is used frequently in discussion regarding access to health care resources. Here the just distribution of resources, in this case pain management, cannot be justly apportioned. Nonmaleficence refers to avoidance of harm; beneficence refers to taking positive actions to help others. Fidelity refers to the agreement to keep promises. Each of these principles is partially expressed in the question; however, justice is most comprised because not all laboring patients have equal access to pain management owing to lack of personnel resources.

A nurse is providing oral care to a patient with stomatitis. Which technique will the nurse use? a. Avoid commercial mouthwashes. b. Avoid normal saline rinses. c. Brush with a hard toothbrush. d. Brush with an alcohol-based toothpaste.

A Stomatitis causes burning, pain, and change in food and fluid tolerance. Advise patients to avoid alcohol and commercial mouthwash and stop smoking. When caring for patients with stomatitis, brush with a soft toothbrush and floss gently to prevent bleeding of the gums. In some cases, flossing needs to be temporarily omitted from oral care. Normal saline rinses (approximately 30 mL) on awaking in the morning, after each meal, and at bedtime help clean the oral cavity.

The nurse hears a health care provider say to the charge nurse that a certain nurse cannot care for patients because the nurse is stupid and won't follow orders. The health care provider also writes in the patient's medical records that the same nurse, by name, is not to care for any of the patients because of incompetence. Which torts has the health care provider committed? (Select all that apply.) a. Libel b. Slander c. Assault d. Battery e. Invasion of privacy

A, B Slander occurred when the health care provider spoke falsely about the nurse, and libel occurred when the health care provider wrote false information in the chart. Both of these situations could cause problems for the nurse's reputation. Invasion of privacy is the release of a patient's medical information to an unauthorized person such as a member of the press, the patient's employer, or the patient's family. Assault is any action that places a person in reasonable fear of harmful, imminent, or unwelcome contact. No actual contact is required for an assault to occur. Battery is any intentional touching without consent.

On your unit there are two RNs: one is a new RN while the other is an experienced RN. In addition, there are three LPNs and two nursing assistants. Which tasks delegated to one of the nursing assistants by the new RN needs to be re-evaluated? (Select all that apply.) A. Apply hydrocortisone cream to eczema on skin after giving the patient a bath. B. Assist the patient with administering a Fleet Enema. C. Empty an ostomy bag. D. Collect and record patient's blood pressure, heart rate, temperature, oxygen saturation, respirations, and pain rating. E. Assist a patient with ambulating.

A, B Option A is a task for an LPN or RN...hydrocortisone cream is a medication and the nursing assistant can't administer medications. Option B: is a task for an LPN or RN....it is a procedure. Option C, D, and E are all delegated tasks a nursing assistant can perform.

On your unit there are two RNs: one is a new RN while the other is an experienced RN. In addition, there are three LPNs and two nursing assistants. Which tasks delegated to one of the nursing assistants by the new RN needs to be re-evaluated? (Select all that apply.) A. Apply hydrocortisone cream to eczema on skin after giving the patient a bath. B. Assist the patient with administering a Fleet Enema. C. Empty an ostomy bag. D. Collect and record patient's blood pressure, heart rate, temperature, oxygen saturation, respirations, and pain rating. E. Assist a patient with ambulating.

A, B Option A is a task for an LPN or RN...hydrocortisone cream is a medication and the nursing assistant can't administer medications. Option B: is a task for an LPN or RN....it is a procedure. Option C, D, and E are all delegated tasks a nursing assistant can perform.

The nurse is participating in a "time-out." In which activities will the nurse be involved? (Select all that apply.) a. Verify the correct site. b. Verify the correct patient. c. Verify the correct procedure. d. Perform "time-out" after surgery. e. Perform the actual marking of the operative site.

A, B, C A time-out is performed just before starting the procedure for final verification of the correct patient, procedure, site, and any implants. The marking and time-out most commonly occur in the holding area, just before the patient enters the OR. The individual performing surgery and who is accountable for it must personally mark the site, and the patient must be involved if possible.

A nurse is providing hygiene care to a bariatric patient using chlorhexidine gluconate (CHG) wipes. Which actions will the nurse take? (Select all that apply.) a. Do not rinse. b. Clean under breasts. c. Inform that the skin will feel sticky. d. Dry thoroughly between skin folds. e. Use two wipes for each area of the body.

A, B, C CHG wipes are easy to use and accessible for older patients and bariatric patients, offering a no- rinse or -drying procedure. For a bariatric patient or a patient who is diaphoretic, provide special attention to body areas such as beneath the woman's breasts, in the groin, skin folds, and perineal area, where moisture collects and irritates skin surfaces. Use wipes as directed on package—one wipe per each area of the body. CHG can leave the skin feeling sticky. If patients complain about its use, you need to explain their vulnerability to infection and how CHG helps reduce occurrence of health care-associated infection

A nurse is preparing a teaching session about contemporary influences on nursing. Which examples should the nurse include? (Select all that apply.) a. Human rights b. Affordable Care Act c. Demographic changes d. Medically underserved e. Decreasing health care costs

A, B, C, D Multiple external forces affect nursing, including the need for nurses' self-care, Affordable Care Act (ACA) and rising (not decreasing) health care costs, demographic changes of the population, human rights, and increasing numbers of medically underserved.

A nurse is providing nursing care to a group of patients. Which actions are direct care interventions? (Select all that apply.) a. Ambulating a patient b. Inserting a feeding tube c. Performing resuscitation d. Documenting wound care e. Teaching about medications

A, B, C, E All of the interventions listed (ambulating, inserting a feeding tube, performing resuscitation, and teaching) are direct care interventions involving patient and nurse interaction, except documenting wound care. Documenting wound care is an example of an indirect intervention.

Which interventions are appropriate for a patient with diabetes and poor wound healing? (Select all that apply.) a. Perform dressing changes twice a day as ordered. b. Teach the patient about signs and symptoms of infection. c. Instruct the family about how to perform dressing changes. d. Gently refocus patient from discussing body image changes. e. Administer medications to control the patient's blood sugar as ordered.

A, B, C, E Nursing priorities include interventions directed at enhancing wound healing. Teaching the patient about signs and symptoms of infection will help the patient identify signs of appropriate wound healing and know when the need for calling the health care provider arises. Performing dressing changes, controlling blood sugars through administration of medications, and instructing the family in dressing changes all contribute to wound healing. As long as a patient is stable and alert, it is appropriate to allow family to assist with care. The patient should be allowed to discuss body image changes.

A nurse is preparing to carry out interventions. Which resources will the nurse make sure are available? (Select all that apply.) a. Equipment b. Safe environment c. Confidence d. Assistive personnel e. Creativity

A, B, D A nurse will organize time and resources in preparation for implementing nursing care. Most nursing procedures require some equipment or supplies. Before performing an intervention, decide which supplies you need and determine their availability. Patient care staff (assistive personnel) work together as patients' needs demand it. A patient's care environment needs to be safe and conducive to implementing therapies. Confidence and creativity are needed to provide safe and effective patient care; however, these are critical thinking attitudes, not resources.

The nurse is caring for a patient who has peripheral neuropathy. Which clinical manifestations does the nurse expect to find upon assessment? (Select all that apply.) a. Abnormal gait b. Foot deformities c. Absent or decreased pedal pulses d. Muscle wasting of lower extremities e. Decreased hair growth on legs and feet

A, B, D A patient with peripheral neuropathy has muscle wasting of lower extremities, foot deformities, and abnormal gait. A patient with vascular insufficiency will have decreased hair growth on legs and feet, absent or decreased pulses, and thickened nails.

A patient has approximately 6 months to live and asks about a do not resuscitate (DNR) order. Which statements by the nurse give the patient correct information? (Select all that apply.) a. "You will be resuscitated unless there is a DNR order in the chart." b. "If you want certain procedures or actions taken or not taken, and you might not be able to tell anyone at the time, you need to complete documents ahead of time that give your health care provider this information." c. "You will be resuscitated at any time to allow you the longest length of survival." d. "If you decide you want a DNR order, you will need to talk to your health care provider." e. "If you travel to another state, your living will should cover your wishes."

A, B, D Health care providers perform CPR on an appropriate patient unless a do not resuscitate (DNR) order has been placed in the patient's chart. The statutes assume that all patients will be resuscitated unless a written DNR order is found in the chart. Legally competent adult patients can consent to a DNR order verbally or in writing after receiving appropriate information from the health care provider. A health care proxy or durable power of attorney for health care (DPAHC) is a legal document that designates a person or persons of one's choosing to make health care decisions when the patient is no longer able to make decisions on his or her own behalf. This agent makes health care treatment decisions based on the patient's wishes, like a DNR. Resuscitation is performed anytime (not just for the longest length of survival) unless a DNR is written in the chart. Differences among the states have been noted regarding advance directives, so the patient should check state laws to see if a state will honor an advance directive that was originated in another state

Which patients will the nurse determine are in need of perineal care? (Select all that apply.) a. A patient with rectal and genital surgical dressings b. A patient with urinary and fecal incontinence c. A circumcised male who is ambulatory d. A patient who has an indwelling catheter e. A bariatric patient

A, B, D, E Patients most in need of perineal care include those at greatest risk for acquiring an infection (e.g., uncircumcised males, patients who have indwelling urinary catheters, or those who are recovering from rectal or genital surgery or childbirth). A patient with urinary and bowel incontinence needs perineal cleaning with each episode of soiling. Bariatric patients need special attention to body areas such as skin folds and the perineal area. In addition, women who are having a menstrual period require perineal care. Circumcised males are not at high risk for acquiring infection, and ambulatory patients can usually provide perineal self-care

A nurse is a member of the ethics committee. Which purposes will the nurse fulfill in this committee? (Select all that apply.) a. Education b. Case consultation c. Purchasing power d. Direct patient care e. Policy recommendation

A, B, E An ethics committee devoted to the teaching and processing of ethical issues and dilemmas exists in most health care facilities. It is generally multidisciplinary and it serves several purposes: education, policy recommendation, and case consultation. It does not have purchasing power or provide direct patient care.

A nurse is teaching a patient with contact dermatitis of the arms and legs about ways to decrease pruritus. Which information should the nurse include in the teaching plan (select all that apply)? a. Cool, wet cloths or dressings can be used to reduce itching. b. Take cool or tepid baths several times daily to decrease itching. c. Add oil to your bath water to aid in moisturizing the affected skin. d. Rub yourself dry with a towel after bathing to prevent skin maceration. e. Use of an over-the-counter (OTC) antihistamine can reduce scratching.

A, B, E Cool or tepid baths, cool dressings, and OTC antihistamines all help reduce pruritus and scratching. Adding oil to bath water is not recommended because of the increased risk for falls. The patient should use the towel to pat (not rub) the skin dry.

Which activities can the nurse working in the outpatient clinic delegate to a licensed practical/vocational nurse (LPN/LVN) (select all that apply)? a. Administer patch testing to a patient with allergic dermatitis. b. Interview a new patient about chronic health problems and allergies. c. Apply a sterile dressing after the health care provider excises a mole. d. Teach a patient about site care after a punch biopsy of an upper arm lesion. e. Explain potassium hydroxide testing to a patient with a superficial skin infection.

A, C Skills such as administration of patch testing and sterile dressing technique are included in LPN/LVN education and scope of practice. Obtaining a health history and patient education require more critical thinking and registered nurse (RN) level education and scope of practice.

While ambulating in the room, a patient complains of feeling dizzy. In what order will the nurse accomplish the following activities? a. Have the patient sit down in a chair. b. Give the patient something to drink. c. Take the patients blood pressure (BP). d. Notify the patients health care provider.

A, C, B, D The first priority for the patient with syncope is to prevent a fall, so the patient should be assisted to a chair. Assessment of the BP will determine whether the dizziness is due to orthostatic hypotension, which occurs because of hypovolemia. Increasing the fluid intake will help prevent orthostatic dizziness. Because this is a common postoperative problem that is usually resolved through nursing measures such as increasing fluid intake and making position changes more slowly, there is no urgent need to notify the health care provider.

A nurse is assessing several patients. Which assessment findings will cause the nurse to follow up? (Select all that apply.) a. Orthopnea b. Nonpalpable lymph nodes c. Pleural friction rub present d. Crackles in lower lung lobes e. Grade 5 muscle function level f. A 160-degree angle between nail plate and nail

A, C, D Abnormal findings will cause a nurse to follow up. Orthopnea is abnormal and indicates cardiovascular or respiratory problems. Pleural friction rub is abnormal and indicated an inflamed pleura. Crackles are adventitious breath sounds and indicate random, sudden re-inflation of groups of alveoli, indicating disruptive passage of air through small airways. Lymph nodes should be nonpalpable; palpable lymph nodes are abnormal. Grade 5 muscle function is normal. A 160-degree angle between nail plate and nail is normal; a larger degree angle is abnormal and indicates clubbing.

Based on the Joint Commission Core Measures for patients with heart failure, which topics should the nurse include in the discharge teaching plan for a patient who has been hospitalized with chronic heart failure (select all that apply)? a. How to take and record daily weight b. Importance of limiting aerobic exercise c. Date and time of follow-up appointment d. Symptoms indicating worsening heart failure e. Actions and side effects of prescribed medications

A, C, D, E The Joint Commission Core Measures state that patients should be taught about prescribed medications, follow-up appointments, weight monitoring, and actions to take for worsening symptoms. Patients with heart failure are encouraged to begin or continue aerobic exercises such as walking, while self-monitoring to avoid excessive fatigue.

The nurse is preparing for a patient who will be going to surgery. The nurse screens for risk factors that can increase a person's risks in surgery. What risk factors are included in the nurse's screening? (Select all that apply.) a. Age b. Race c. Obesity d. Nutrition e. Pregnancy f. Ambulatory surgery

A, C, D, E Very young and old patients are at risk during surgery because of immature or declining physiological status. Normal tissue repair and resistance to infection depend on adequate nutrients. Obesity increases surgical risk by reducing respiratory and cardiac function. During pregnancy, the concern is for the mother and the developing fetus. Because all major systems of the mother are affected during pregnancy, risks for operative complications are increased. Race and ambulatory surgery are not risks associated with a surgical procedure.

As the registered nurse, which tasks below should you NOT delegate to the LPN? (Select all that apply.) A. Performing an assessment on a new admission B. Collecting a urine sample from an indwelling Foley catheter C. Developing a plan of care for a patient who is admitted with Guillain-Barré Syndrome D. Educating a patient about how to monitor for side effects associated with Warfarin E. Auscultating lung and bowel sounds F. Starting a blood transfusion G. Administering IV Morphine 2 mg for pain H. Providing wound care to a stage 3 pressure injury

A, C, D, F, G these are all out of the scope of practice for an LPN. Remember anything that deals with assessments, educating, evaluating, developing a plan of care, IV medications, unstable patients, or invasive/complex procedures where there is unpredictability the RN is responsible for doing it, & these tasks can't be delegated. An LPN can perform a focused assessment by listening to lung or bowel sounds & report the findings to the RN but a comprehensive assessment is done by the RN. In addition, the LPN can perform standard procedures that are predictable on stable patients like wound care for a pressure injury, Foley catheter insertion, obtaining an EKG, obtaining blood glucose level etc.

As the registered nurse, which tasks below should you NOT delegate to the LPN? (Select all that apply.) A. Performing an assessment on a new admission B. Collecting a urine sample from an indwelling Foley catheter C. Developing a plan of care for a patient who is admitted with Guillain-Barré Syndrome D. Educating a patient about how to monitor for side effects associated with Warfarin E. Auscultating lung and bowel sounds F. Starting a blood transfusion G. Administering IV Morphine 2 mg for pain H. Providing wound care to a stage 3 pressure injury

A, C, D, F, G these are all out of the scope of practice for an LPN. Remember anything that deals with assessments, educating, evaluating, developing a plan of care, IV medications, unstable patients, or invasive/complex procedures where there is unpredictability the RN is responsible for doing it, & these tasks can't be delegated. An LPN can perform a focused assessment by listening to lung or bowel sounds & report the findings to the RN but a comprehensive assessment is done by the RN. In addition, the LPN can perform standard procedures that are predictable on stable patients like wound care for a pressure injury, Foley catheter insertion, obtaining an EKG, obtaining blood glucose level etc.

A nurse is assessing a patient's cranial nerve IX. Which items does the nurse gather before conducting the assessment? (Select all that apply.) a. Vial of sugar b. Snellen chart c. Tongue blade d. Ophthalmoscope e. Lemon applicator

A, C, E Cranial nerve IX is the glossopharyngeal, which controls taste and ability to swallow. The nurse asks the patient to identify sour (lemon) or sweet (sugar) tastes on the back of the tongue and uses a tongue blade to elicit a gag reflex. Ophthalmoscopes are used for vision. A Snellen chart is used to test cranial nerve II (optic).

The nurse manager from the oncology unit has had two callouts; the orthopedic unit has had multiple discharges and probably will have to cancel one or two of its nurses. The orthopedic unit has agreed to "float" two of its nurses to the oncology unit if oncology can "float" a nursing assistant to the orthopedic unit to help with obtaining vital signs. Which concepts does this situation entail? (Select all that apply.) a. Autonomy b. Informatics c. Accountability d. Political activism e. Teamwork and collaboration

A, C, E Staffing is an independent nursing intervention and is an example of autonomy. Along with increased autonomy comes accountability or responsibility for outcomes of an action. When nurses work together this is teamwork and collaboration. Informatics is the use of information and technology to communicate, manage knowledge, mitigate error, and support decision making. Political activism usually involves more than day-to-day activities such as unit staffing.

Which of the following patients are MOST at risk for developing heart failure? Select-all-that-apply: A. A 69 year old male with a history of alcohol abuse and is recovering from a myocardial infarction. B. A 55 year old female with a health history of asthma and hypoparathyroidism. C. A 30 year old male with a history of endocarditis and has severe mitral stenosis. D. A 45 year old female with lung cancer stage 2. E. A 58 year old female with uncontrolled hypertension and is being treated for influenza.

A, C, E These patients are at most risk for heart failure. Remember risks factor for developing heart failure include: remember the mnemonic FAILURE: Faulty heart valves ( Option C mitral stenosis in this case), Arrhythmias, Infarction (Option A), Lineage, Uncontrolled hypertension (Option E), Recreational drug usage, Evaders (Option E with influenza)

The nurse calculates the medication dose for an infant on the pediatric unit and determines that the dose is twice what it should be based upon the drug book's information. The pediatrician is contacted and says to administer the medication as ordered. Which actions should the nurse take next? (Select all that apply.) a. Notify the nursing supervisor. b. Administer the medication as ordered. c. Give the amount listed in the drug book. d. Ask the mother to give the drug to her child. e. Check the chain of command policy for such situations.

A, E If the health care provider confirms an order and the nurse still believes that it is inappropriate, the nurse should inform the supervising nurse and follow the established chain of command. Nurses follow health care providers' orders unless they believe the orders are in error or may harm patients. Therefore, the nurse needs to assess all orders. If an order seems to be erroneous or harmful, further clarification from the health care provider is necessary. The supervising nurse should be able to help resolve the questionable order, but only the health care provider who wrote the order or a health care provider covering for the one who wrote the order can change the order. Harm to the infant could occur if the medication is given as ordered. The nurse cannot change an order by giving the amount listed in the drug book. Asking the mother to give the drug is inappropriate.

The nurse is caring for a postoperative patient with an incision. Which actions will the nurse take to decrease wound infections? (Select all that apply.) a. Maintain normoglycemia. b. Use a straight razor to remove hair. c. Provide bath and linen change daily. d. Perform first dressing change 2 days postoperatively. e. Perform hand hygiene before and after contact with the patient. f. Administer antibiotics within 60 minutes before surgical incision.

A, E Performing hand hygiene before and after contact with the patient helps to decrease the number of microorganisms and break the chain of infection. Maintaining blood glucose levels at less than 150 mg/dL has resulted in decreased wound infection. Removing unwanted hair by clipping instead of shaving decreases the numbers of nicks and cuts caused by a razor and the potential for the introduction of microbes. The patient is postoperative; administration of an antibiotic 60 minutes before the surgical incision supports the defense against infection preoperatively. Providing a bath and linen change daily is positive but is not necessarily important for infection control. Many surgeons prefer to change surgical dressings the first time so they can inspect the incisional area, but this is done before 2 days postoperatively.

A patient states they are having intense itching between their fingers with brown linear lines presenting. Based on your nursing knowledge this best describes what condition? a. Scabies b. Psorasis c. Ecemza d. Ringworm

A. Intense itching and brown linear lines between the finger is a hallmark sign of scabies.

A patient with impetigo would have which of the following psychosocial issues? a. Social isolation with restrictions with physical activity b. Decreased personal hygiene c. Increased risk for radiation exposure d. None of the options are correct

A. Patient with impetigo are placed in contact isolation for the bacterial infection is highly contagious. Therefore, the patient would have to be isolated from people and activities and social contact will impact them psychosocially.

A patient with herpes zoster is newly admitted. Based on your nursing knowledge of this disease, which statment is correct? a. "If a person has not had chickenpox they could contract herpes zoster." b. "The virus is located in the basal nerve root ganglion." c. "Herpes zoster can only be diagnosed by skin stains." d. "Herpes zoster can appear in a healthy person at anytime."

A. Shingles is located in the DORSAL (not basal) nerve root. In addition, skin cultures and antinuclear antibody test can diagnose shingles. Lastly, it tends to appear in the IMMUNOCOMPROMISED who have had a history of chickenpox....rarely a healthy person.

Which statement by a patient with scabies causes concern and that they should be re-educated by the nurse? a. "I'm going on vacation next week with my bowling team." b. "I will apply anti-scabies medication thickly to my face and scalp. c. "I will wash my clothing only in cold water with bleach." d. "I will avoid using soap and water while using anti-scabies treatment."

A. The face and scalp are not affected by scabies. The patients bedding and clothes should be wash in HOT water. Also, the patient should remove medication prior to reapplication of medication with soap and water.

The doctor orders a skin culture and antiobiotic therapy on your newly admitted patient. Which of the following sequence of nursing interventions is correct? a. Obtain skin culture and then administer antibiotic therapy. b. Administer antibiotic therapy and then in 1 hour obtain skin culture. c. Obtain skin culture and then administer antibiotic therapy 6 hours from skin culture collection. d. Start antibiotic therapy and then immediately collect culture.

A. The nurse should always collect the skin culture first and then start the antibiotic therapy.

You are developing a care plan for a patient with psoriasis. What would you include in your nursing interventions? a. Administer daily soaks with tepid, wet compresses to affected area of the skin. b. Keep blisters intact and protected. c. Apply acetic acid compresses as prescribed. d. Place the patient in contact isolation.

A. The only option that is correct is: "Administer daily soaks with tepid, wet compresses to affected are of the skin"...this allow for the scales to be removed. All of the other options are nursing interventions for a patient with shingles.

A patient recently had a malignant melanoma removed from her back. Which statement by the patient ensures she understood your instructions about preventing skin cancer and further skin care? a. "I will avoid sun exposure between 11 am to 3 pm." b. "I will wear a SPF of 15 sunscreen when going out in the boat with my bikini." c. "I will make an appointment every 3 years for a skin cancer check." d. "Melanoma rarely metastasizes only squamous cell."

A. The patient should avoid the sun from 11 am to 3 pm (when the sun rays are the strongest), wear layers to cover the skin (not just a bikini), have regular skin check-up every 6 months to a year. In addition, melanoma is most likely to metastasize when compared to squamos cell.

A patient with acne vulargis is taking Accutane. Which statement by the patient is correct? a. "I stopped taking my vitamin A supplement before I took my 1st dose of Accutane." b. "I can't wait to start seeing results next week." c. "I love using oil-based cosmetics...it hides my pimples better." d. "I scrub by face three times a day with over-the-counter cleansers."

A. The patient should stopped taking any vitamin A supplement because this interacts with Accutane. In addition, the patient should be instructed that results will take 4 to 6 weeks to notice and that WATER based cosmetic should be used, and to avoid scrub the face with any type of OTC cleansers.

During a full body skin assessment of a patient you see the following lesion with these characteristics: waxy border, red papule with a central crater. This describes what type of lesion? a. Basal cell b. Squamous cell c. Melonoma d. Normal, benign mole

A. These are the charateritics of a basal cell lesion....all of the other options are incorrect.

A patient presents with a skin infection caused by STREPTOCOCCUS PYOGENES. The patient's lower leg is infected from the deep dermis to the subcutaneous fat. What skin disorder does this describe? a. Cellulitis b. Erysipelas c. Psoriasis d. Impetigo

A. This describes cellulitis. This key words in this question is STREPTOCOCCUS PYOGENES and the lower leg, deep dermis to the subcutaneous fat which is a hallmark of cellulitis.

A client calls the Ask a Nurse line and describes the symptoms of frostbite. How would you assist the caller? a. Instruct the patient to rewarm the affected part with warm water for 15 to 20 minutes until the skin flushes. b. Instruct the patient to attempt to debride the blisters so new skin can form. c. Instruct the patient to immediately cover the exposed area and let it air dry. d. Instruct the patient to massage the affected area to increase blood flow.

A. When treating frostbite you would never instruct the patient to debride the area or massage it...for this causes tissue damage. The only correct option in the question is: Instruct the patient to rewarm the affected part with warm water for 15 to 20 minutes until the skin flushes.

A patient is being discharged from having a skin biopsy of a lesion on the right thigh. What would you include in the discharge instructions? a. Keep dressing in place for 8 hours and then clean daily. b. Avoid antibiotic ointments usage for 72 hours. c. Remove the dressing every 1 to 2 hours and let air dry. d. If redness and yellow exudate present this is normal for 7 days.

A. After a skin biopsy the dressing should be kept in place for 8 hours and then clean daily. Antibiotic ointment is usually prescribed by the doctor and should be used as directed. All the other options are incorrect.

You are observing your patient use the incentive spirometry. What demonstration by the patient lets you know the patient understands how to use the device properly? A. The patient inhales slowly on the device and maintains the flow indicator between 600 to 900 level B. The patient blows on the mouthpiece rapidly. C. The patient uses the incentive spirometry once a day D. The patient rapidly inhales on the devices and exhales

A. All of the options are wrong expect for "The patient inhales slowly on the device and maintains the flow indicator between 600 to 900 level". The other options do not demonstrate how to properly use the incentive spirometry.

When assessing your patient who is post-opt, you notice that the patient's right calf vein feels hard, cord-like, and is tender to the touch. The patient reports it is aching and painful. What would NOT be an appropriate nursing intervention for this patient? A. Allow the patient to dangle the legs to help increase circulation and alleviate pain B. Instruct the patient to not sit in one position for a long period of time C. Elevate the extremity 30 degrees without allowing any pressure on affected area D. Administer anticoagulants as ordered by MD

A. All options are correct except for "Allow the patient to dangle the legs to help increase circulation and alleviate pain". The patient should NOT dangle the legs because this causes blood to pool in the lower extremities which will put the patient at risk for another blood clot formation.

As a nurse, which statement is incorrect regarding an informed consent signed by a patient? A. The nurse is responsible for obtaining the consent for surgery B. Patients under 18 years of age may need a parent or legal guardian to sign a consent form C. The nurse can witness the client signing the consent form D. It is the nurse's responsibility to ensure the patient has been educated by the physician about the procedure before informed consent is obtained

A. All statements are correct except that it's the nurse's responsibility for obtaining the consent for surgery. It is the surgeon's responsibility.

After surgery your patient starts to shiver uncontrollably. What nursing intervention would you do FIRST? A. Apply warm blankets & continue oxygen as prescribed B. Take the patient's rectal temperature C. Page the doctor for further orders D. Adjust the thermostat in the room

A. Shivering is an early sign that the patient is starting to experience hypothermia. Immediately, the nurse would need to control the shivering by applying warm blankets and continue oxygen. When the patient starts to experience hypothermia, vital organs are not receiving as much oxygenated blood due to the vasoconstriction. Therefore, oxygen would need to be continued. Then the nurse would take the patient's temperature.

A patient is 6 days post-opt from abdominal surgery. The patient is to be discharged later today. The patient uses the call light and asks you to come to his room and look at his surgical site. On arrival, you see that approximately 2 inches of internal organs are protruding through the incision. What intervention would you NOT do? A. Put the patient in prone position with knees extended to put pressure on the site B. Cover the wound with sterile normal saline dressing C. Monitor for signs of shock D. Notify the MD and administer as prescribed antiemetic to prevent vomiting

A. The patient is experiencing wound evisceration. This is an emergent situation. The patient should be placed in low Fowler's position with the knees bent to prevent abdominal tension.

Non-pharmacological techniques can help lower blood pressure. Which of the following is not considered one of these types of techniques? a. Dietary changes b. Multivitamins c. Smoking cessation d. Limiting caffeine

B

Which of the following patients does not have a risk factor for hypertension? a. A 25 year old male with a BMI of 35. b. A 35 year old female with a total cholesterol level of 100. c. A 68 year old male who reports smoking 2 packs of cigarettes a day. d. A 40 year old female with a family history of hypertension and diabetes.

B

Which of the following patients is not a candidate for a beta blocker medication? a. A 45 year old male with angina. b. A 39 year old female with asthma. c. A 25 year old female with migraines. d. A 55 year old male with a history of two heart attacks.

B

A teen patient is tearful and reports locating lumps in her breasts. Other history obtained is that she is currently menstruating. Physical examination reveals soft and movable cysts in both breasts that are painful to palpation. The nurse also notes that the patient's nipples are erect, but the areola is wrinkled. Which action will the nurse take after talking with the health care provider? a. Reassure patient that her symptoms are normal. b. Discuss the possibility of fibrocystic disease as the probable cause. c. Consult a breast surgeon because of the abnormal nipples and areola. d. Tell the patient that the symptoms may get worse when her period ends.

B A common benign condition of the breast is benign (fibrocystic) breast disease. This patient has symptoms of fibrocystic disease, which include bilateral lumpy, painful breasts sometimes accompanied by nipple discharge. Symptoms are more apparent during the menstrual period. When palpated, the cysts (lumps) are soft, well differentiated, and movable. Deep cysts feel hard. Although a common condition, benign breast disease is not normal; therefore, the nurse does not tell the patient that this is a normal finding. During examination of the nipples and areolae, the nipple sometimes becomes erect with wrinkling of the areola. Therefore, consulting a breast surgeon to treat her nipples and areolae is not appropriate.

A nurse prepares the budget and policies for an intensive care unit. Which role is the nurse implementing? a. Educator b. Manager c. Advocate d. Caregiver

B A manager coordinates the activities of members of the nursing staff in delivering nursing care and has personnel, policy, and budgetary responsibility for a specific nursing unit or facility. As an educator, you explain concepts and facts about health, describe the reason for routine care activities, demonstrate procedures such as self-care activities, reinforce learning or patient behavior, and evaluate the patient's progress in learning. As a patient advocate, you protect your patient's human and legal rights and provide assistance in asserting these rights if the need arises. As a caregiver, you help patients maintain and regain health, manage disease and symptoms, and attain a maximal level function and independence through the healing process.

On admission of a patient to the postanesthesia care unit (PACU), the blood pressure (BP) is 122/72. Thirty minutes after admission, the BP falls to 114/62, with a pulse of 74 and warm, dry skin. Which action by the nurse is most appropriate? a. Increase the IV fluid rate. b. Continue to take vital signs every 15 minutes. c. Administer oxygen therapy at 100% per mask. d. Notify the anesthesia care provider (ACP) immediately.

B A slight drop in postoperative BP with a normal pulse and warm, dry skin indicates normal response to the residual effects of anesthesia and requires only ongoing monitoring. Hypotension with tachycardia and/or cool, clammy skin would suggest hypovolemic or hemorrhagic shock and the need for notification of the ACP, increased fluids, and high-concentration oxygen administration.

A teen female patient reports intermittent abdominal pain for 12 hours. No dysuria is present. Which action will the nurse take when performing an abdominal assessment? a. Assess the area that is most tender first. b. Ask the patient about the color of her stools. c. Recommend that the patient take more laxatives. d. Avoid sexual references such as possible pregnancy.

B Abdominal pain can be related to bowels. If stools are black or tarry (melena), this may indicate gastrointestinal alteration. The nurse should caution patients about the dangers of excessive use of laxatives or enemas. There is not enough information about the abdominal pain to recommend laxatives. Determine if the patient is pregnant, and note her last menstrual period. Pregnancy causes changes in abdominal shape and contour. Assess painful areas last to minimize discomfort and anxiety.

While providing care to a patient, the nurse is responsible, both professionally and legally. Which concept does this describe? a. Autonomy b. Accountability c. Patient advocacy d. Patient education

B Accountability means that the nurse is responsible, professionally and legally, for the type and quality of nursing care provided. Autonomy is an essential element of professional nursing that involves the initiation of independent nursing interventions without medical orders. As a patient advocate, the nurse protects the patient's human and legal rights and provides assistance in asserting these rights if the need arises. As an educator, the nurse explains concepts and facts about health, describes the reasons for routine care activities, demonstrates procedures such as self-care activities, reinforces learning or patient behavior, and evaluates the patient's progress in learning.

The patient is being fitted with a hearing aid. In teaching the patient how to care for the hearing aid, which instructions will the nurse provide? a. Change the battery every day or as needed. b. Adjust the volume for a talking distance of 1 yard. c. Wear the hearing aid 24 hours per day except when sleeping. d. Avoid the use of hairspray, but aerosol perfumes are allowed.

B Adjust volume to a comfortable level for talking at a distance of 1 yard. Initially, wear a hearing aid for 15 to 20 minutes; then gradually increase wear time to 10 to 12 hours per day. Batteries last 1 week with daily wearing of 10 to 12 hours. Avoid the use of hairspray and perfume while wearing hearing aids. Residue from the spray can cause the aid to become oily and greasy.

The nurse is encouraging the postoperative patient to utilize diaphragmatic breathing. Which priority goal is the nurse trying to achieve? a. Manage pain b. Prevent atelectasis c. Reduce healing time d. Decrease thrombus formation

B After surgery, patients may have reduced lung volume and may require greater effort to cough and deep breathe; inadequate lung expansion can lead to atelectasis and pneumonia. Purposely utilizing diaphragmatic breathing can decrease this risk. During general anesthesia, the lungs are not fully inflated during surgery and the cough reflex is suppressed, so mucus collects within airway passages. Diaphragmatic breathing does not manage pain; in some cases, if splinting and pain medications are not given, it can cause pain. Diaphragmatic breathing does not reduce healing time or decrease thrombus formation. Better, more effective interventions are available for these situations.

SKIP The nurse is concerned about the skin integrity of the patient in the intraoperative phase of surgery. Which action will the nurse take to minimize skin breakdown? a. Encouraging the patient to bathe before surgery b. Securing attachments to the operating table with foam padding c. Periodically adjusting the patient during the surgical procedure d. Measuring the time a patient is in one position during surgery

B Although it may be necessary to place a patient in an unusual position, try to maintain correct alignment and protect the patient from pressure, abrasion, and other injuries. Special mattresses, use of foam padding, and attachments to the operating suite table provide protection for the extremities and bony prominences. Bathing before surgery helps to decrease the number of microbes on the skin. Periodically adjusting the patient during the surgical procedure is impractical and can present a safety issue with regard to maintaining sterility of the field and maintaining an airway. Measuring the time the patient is in one position may help with monitoring the situation but does not prevent skin breakdown

A nursing student has been written up several times for being late with providing patient care and for omitting aspects of patient care and not knowing basic procedures that were taught in the skills course one term earlier. The nursing student says, "I don't understand what the big deal is. As my instructor, you are there to protect me and make sure I don't make mistakes." What is the best response from the nursing instructor? a. "You are practicing under the license of the hospital's insurance." b. "You are expected to perform at the level of a professional nurse." c. "You are expected to perform at the level of a prudent nursing student." d. "You are practicing under the license of the nurse assigned to the patient."

B Although nursing students are not employees of the health care facility where they are having their clinical experience, they are expected to perform as professional nurses would in providing safe patient care. Different levels of standards do not apply. No standard is used for nursing students other than that they must meet the standards of a professional nurse. Student nurses do not practice under anybody's license; nursing students are liable if their actions exceed their scope of practice or cause harm to patients.

A nurse assists a patient on the first postoperative day to ambulate, cough, deep breathe, and turn. Which action by the nurse is most helpful? a. Teach the patient to fully exhale into the incentive spirometer. b. Administer ordered analgesic medications before these activities. c. Ask the patient to state two possible complications of immobility. d. Encourage the patient to state the purpose of splinting the incision.

B An important nursing action to encourage these postoperative activities is administration of adequate analgesia to allow the patient to accomplish the activities with minimal pain. Even with motivation provided by proper teaching, positive reinforcement, and concern about complications, patients will have difficulty if there is a great deal of pain involved with these activities. When using an incentive spirometer, the patient should be taught to inhale deeply, rather than exhale into the spirometer to promote lung expansion and prevent atelectasis.

A recent immigrant who does not speak English is alert and requires hospitalization. What is the initial action that the nurse must take to enable informed consent to be obtained? a. Ask a family member to translate what the nurse is saying. b. Request an official interpreter to explain the terms of consent. c. Notify the nursing manager that the patient doesn't speak English. d. Use hand gestures and medical equipment while explaining in English.

B An official interpreter must be present to explain the terms of consent if a patient speaks only a foreign language. A family member or acquaintance who speaks a patient's language should not interpret health information. Family members can tell those caring for the patient what the patient is saying, but privacy regarding the patient's condition, assessment, etc., must be protected. A nurse can take care of requesting an interpreter, and the nurse manager is not needed. Using hand gestures and medical equipment is inappropriate when communicating with a patient who does not understand the language spoken. Certain hand gestures may be acceptable in one culture and not appropriate in another. The medical equipment may be unknown and frightening to the patient, and the patient still doesn't understand what is being said.

The nurse explains the pain relief measures available after surgery during preoperative teaching for a surgical patient. Which comment from the patient indicates the need for additional education on this topic? a. "I will be asked to rate my pain on a pain scale." b. "I will have minimal pain because of the anesthesia." c. "I will take the pain medication as the provider prescribes it." d. "I will take my pain medications before doing postoperative exercises."

B Anesthesia will be provided during the procedure itself, and the patient should not experience pain during the procedure; however, this will not minimize the pain after surgery. Pain management is utilized after the postoperative phase. Inform the patient of interventions available for pain relief, including medication, relaxation, and distraction. The patient needs to know and understand how to take the medications that the health care provider will prescribe postoperatively. During the stay in the facility, the level of pain is frequently assessed by the nurses. Coordinating pain medication with postoperative exercises helps to minimize discomfort and allows the exercises to be more effective.

While recovering from a severe illness, a hospitalized patient wants to change a living will, which was signed 9 months ago. Which response by the nurse is most appropriate? a. "Check with your admitting health care provider whether a copy is on your chart." b. "Let me check with someone here in the hospital who can assist you." c. "You are not allowed to ever change a living will after signing it." d. "Your living will can be changed only once each calendar year."

B As long as the patient is not declared legally incompetent or lacks the capacity to make decisions, living wills can be changed. It is the nurse's responsibility to find an appropriate person in the facility to assist the patient. Checking with the health care provider about the presence of a living will on the chart has nothing to do with the patient's desire to change the living will. The question states that the patient wants to change a living will. A living will can be changed whenever the patient decides to change it, as long as the patient is competent.

A nurse assesses a patient's fluid status and decides that the patient needs to drink more fluids. The nurse then encourages the patient to drink more fluids. Which concept is the nurse demonstrating? a. Licensure b. Autonomy c. Certification d. Accountability

B Autonomy is an essential element of professional nursing that involves the initiation of independent nursing interventions without medical orders. To obtain licensure in the United States, the RN candidate must pass the NCLEX-RN. Beyond the NCLEX-RN, the nurse may choose to work toward certification in a specific area of nursing practice. Accountability means that you are responsible, professionally and legally, for the type and quality of nursing care provided.

A 74 year old female presents to the ER with complaints of dyspnea, persistent cough, and unable to sleep at night due to difficulty breathing. On assessment, you note crackles throughout the lung fields, respiratory rate of 25, and an oxygen saturation of 90% on room air. Which of the following lab results confirm your suspicions of heart failure? A. K+ 5.6 B. BNP 820 C. BUN 9 D. Troponin <0.02

B BNP (b-type natriuretic peptide) is a biomarker released by the ventricles when there is excessive pressure in the heart due to heart failure. <100 no failure, 100-300 present, >300 pg/mL mild, >600 pg/mL >moderate, 900 pg/mL severe

A 38-year-old female patient states that she is using topical fluorouracil to treat actinic keratoses on her face. Which additional assessment information will be most important for the nurse to obtain? a. History of sun exposure by the patient b. Method of birth control used by the patient c. Length of time the patient has used fluorouracil d. Appearance of the treated areas on the patients face

B Because fluorouracil is teratogenic, it is essential that the patient use a reliable method of birth control. The other information is also important for the nurse to obtain, but lack of reliable birth control has the most potential for serious adverse medication effects.

A patient with hypertension who has just started taking atenolol (Tenormin) returns to the health clinic after 2 weeks for a follow-up visit. The blood pressure (BP) is unchanged from the previous visit. Which action should the nurse take first? a. Inform the patient about the reasons for a possible change in drug dosage. b. Question the patient about whether the medication is actually being taken. c. Inform the patient that multiple drugs are often needed to treat hypertension. d. Question the patient regarding any lifestyle changes made to help control BP.

B Because noncompliance with antihypertensive therapy is common, the nurses initial action should be to determine whether the patient is taking the atenolol as prescribed. The other actions also may be implemented, but these would be done after assessing patient compliance with the prescribed therapy.

The nurse reviews the laboratory results for a patient on the first postoperative day after a hiatal hernia repair. Which finding would indicate to the nurse that the patient is at increased risk for poor wound healing? a. Potassium 3.5 mEq/L b. Albumin level 2.2 g/dL c. Hemoglobin 11.2 g/dL d. White blood cells 11,900/L

B Because proteins are needed for an appropriate inflammatory response and wound healing, the low serum albumin level (normal level 3.5 to 5.0 g/dL) indicates a risk for poor wound healing. The potassium level is normal. Because a small amount of blood loss is expected with surgery, the hemoglobin level is not indicative of an increased risk for wound healing. WBC count is expected to increase after surgery as a part of the normal inflammatory response.

A patient who has begun to awaken after 30 minutes in the postanesthesia care unit (PACU) is restless and shouting at the nurse. The patients oxygen saturation is 96%, and recent laboratory results are all normal. Which action by the nurse is most appropriate? a. Increase the IV fluid rate. b. Assess for bladder distention. c. Notify the anesthesia care provider (ACP). d. Demonstrate the use of the nurse call bell button.

B Because the patients assessment indicates physiologic stability, the most likely cause of the patients agitation is emergence delirium, which will resolve as the patient wakes up more fully. The nurse should look for a cause such as bladder distention. Although hypoxemia is the most common cause, the patients oxygen saturation is 96%. Emergence delirium is common in patients recovering from anesthesia, so there is no need to notify the ACP. Orientation of the patient to bed controls is needed, but is not likely to be effective until the effects of anesthesia have resolved more completely.

The postanesthesia care unit (PACU) nurse transports the inpatient surgical patient to the medical-surgical floor. Before leaving the floor, the medical-surgical nurse obtains a complete set of vital signs. What is the rationale for this nursing action? a. This is done to complete the first action in a head-to-toe assessment. b. This is done to compare and monitor for vital sign variation during transport. c. This is done to ensure that the medical-surgical nurse checks on the postoperative patient. d. This is done to follow hospital policy and procedure for care of the surgical patient.

B Before the PACU nurse leaves the acute care area, the staff nurse assuming care for the patient takes a complete set of vital signs to compare with PACU findings. Minor vital sign variations normally occur after transporting the patient. The PACU nurse reviews the patient's information with the medical-surgical nurse, including the surgical and PACU course, physician orders, and the patient's condition. While vital signs may or may not be the first action in a head-to-toe assessment, this is not the rationale for this situation. While following policy or ascertaining that the floor nurse checks on the patient are good reasons for safe care, they are not the best rationale for obtaining vital signs.

A nurse is teaching the staff about Benner's levels of proficiency. In which order should the nurse place the levels from beginning level to ending level? 1. Expert 2. Novice 3. Proficient 4. Competent 5. Advanced beginner a. 2, 4, 5, 1, 3 b. 2, 5, 4, 3, 1 c. 4, 2, 5, 3, 1 d. 4, 5, 2, 1, 3

B Benner's levels of proficiency are as follows: novice, advanced beginner, competent, proficient, and expert

A nurse is providing AM care to patients. Which action will the nurse take? a. Soaks feet of patient with peripheral vascular disease b. Applies CHG solution to wash perineum of patient with a stroke c. Cleanses eye from outer canthus to inner canthus of patient with diabetes d. Uses long, firm stroke to wash legs of patient with blood-clotting disorder

B CHG is safe to use on the perineum and external mucosa. If patient has diabetes or peripheral vascular disease with impaired circulation and/or sensation, do not soak feet. Maceration of skin may predispose to infection. Do not use long, firm strokes to wash the lower extremities of patients with history of deep vein thrombosis or blood-clotting disorders. Use short, light strokes instead. Eye should be cleansed from the inner to outer canthus on all patients.

A patient who has never had any prior surgeries tells the nurse doing the preoperative assessment about an allergy to bananas and avocados. Which action is most important for the nurse to take? a. Notify the dietitian about the food allergies. b. Alert the surgery center about a possible latex allergy. c. Reassure the patient that all allergies are noted on the medical record. d. Ask whether the patient uses antihistamines to reduce allergic reactions.

B Certain food allergies (e.g., eggs, avocados, bananas, chestnuts, potatoes, peaches) are related to latex allergies. When a patient is allergic to latex, special nonlatex materials are used during surgical procedures, and the staff will need to know about the allergy in advance to obtain appropriate nonlatex materials and have them available during surgery. The other actions also may be appropriate, but prevention of allergic reaction during surgery is the most important action.

Which information in the preoperative patients medication history is most important to communicate to the health care provider? a. The patient uses acetaminophen (Tylenol) occasionally for aches and pains. b. The patient takes garlic capsules daily but did not take any on the surgical day. c. The patient has a history of cocaine use but quit using the drug over 10 years ago. d. The patient took a sedative medication the previous night to assist in falling asleep.

B Chronic use of garlic may predispose to intraoperative and postoperative bleeding. The use of a sedative the previous night, occasional acetaminophen use, and a distant history of cocaine use will not usually affect the surgical outcome.

A nurse has compassion fatigue. What is the nurse experiencing? a. Lateral violence and intrapersonal conflict b. Burnout and secondary traumatic stress c. Short-term grief and single stressor d. Physical and mental exhaustion

B Compassion fatigue is a term used to describe a state of burnout and secondary traumatic stress. Compassion fatigue may contribute to what is described as lateral violence (nurse-nurse interactions, not intrapersonal). Frequent, intense, or prolonged exposure to grief and loss places nurses at risk for developing compassion fatigue. Stressors, not a single stressor, contribute to compassion fatigue. Physical and mental exhaustion describes burnout only.

A graduate of a baccalaureate degree program is ready to start working as an RN in the emergency department. Which action must the nurse take first? a. Obtain certification for an emergency nurse. b. Pass the National Council Licensure Examination. c. Take a course on genomics to provide competent emergency care. d. Complete the Hospital Consumer Assessment of Healthcare Providers Systems.

B Currently, in the United States, the most common way to become a registered nurse (RN) is through completion of an associate's degree or baccalaureate degree program. Graduates of both programs are eligible to take the National Council Licensure Examination for Registered Nurses (NCLEX-RN) to become registered nurses in the state in which they will practice. Certification can be obtained after passing the NCLEX and working for the specified amount of time. Genomics is a newer term that describes the study of all the genes in a person and interactions of these genes with one another and with that person's environment. Consumers can also access Hospital Consumer Assessment of Healthcare Providers Systems (HCAHPS) to obtain information about patients' perspectives on hospital care.

The nurse is caring for a postoperative patient with an abdominal incision. The nurse provides a pillow to use during coughing. Which activity is the nurse promoting? a. Pain relief b. Splinting c. Distraction d. Anxiety reduction

B Deep breathing and coughing exercises place additional stress on the suture line and cause discomfort. Splinting incisions with hands and a pillow provides firm support and reduces incisional pull. Providing a pillow during coughing does not provide distraction or reduce anxiety. Providing a pillow does not provide pain relief. Coughing can increase anxiety because it can cause pain. Analgesics provide pain relief.

A patient who is just waking up after having hip replacement surgery is agitated and confused. Which action should the nurse take first? a. Administer the ordered opioid. b. Check the oxygen (O2) saturation. c. Take the blood pressure and pulse. d. Apply wrist restraints to secure IV lines.

B Emergence delirium may be caused by a variety of factors. However, the nurse should first assess for hypoxemia. The other actions also may be appropriate, but are not the best initial action.

A nurse is assessing a patient's skin. Which patient is most at risk for impaired skin integrity? a. A patient who is afebrile b. A patient who is diaphoretic c. A patient with strong pedal pulses d. A patient with adequate skin turgor

B Excessive moisture (diaphoretic) on the surface of the skin serves as a medium for bacterial growth and causes irritation, softens epidermal cells, and leads to skin maceration. A patient who is afebrile is not a high risk; however, a patient who is febrile (fever) is prone to skin breakdown. A patient with strong pedal pulses is not a high risk; however, a patient with vascular insufficiency is. A patient with adequate skin turgor is not a high risk; however, a patient with poor skin turgor is.

The nurse is intervening for a family member with role strain. Which direct care nursing intervention is most appropriate? a. Assisting with activities of daily living b. Counseling about respite care options c. Teaching range-of-motion exercises d. Consulting with a social worker

B Family caregivers need assistance in adjusting to the physical and emotional demands of caregiving. Sometimes they need respite (i.e., a break from providing care). Counseling is an example of a direct care nursing intervention. The other options do not address the identified problem of role strain (activities of daily living and range-of-motion exercises). Consulting is an indirect care nursing intervention.

A patient's hygiene schedule of bathing and brushing teeth is largely influenced by family customs. For which age group is the nurse most likely providing care? a. Adolescent b. Preschooler c. Older adult d. Adult

B Family customs play a major role during childhood in determining hygiene practices such as the frequency of bathing, the time of day bathing is performed, and even whether certain hygiene practices such as brushing of the teeth or flossing are performed. As children enter adolescence, peer groups and media often influence hygiene practices. During the adult years involvement with friends and work groups shape the expectations that people have about personal appearance. Some older adults' hygiene practices change because of changes in living conditions and available resources.

A nurse is performing a mental status examination and asks an adult patient what the statement "Don't cry over spilled milk" means. Which area is the nurse assessing? a. Long-term memory b. Abstract thinking c. Recent memory d. Knowledge

B For an individual to explain common phrases such as "A stitch in time saves nine" or "Don't cry over spilled milk" requires a higher level of intellectual function or abstract thinking. Knowledge-based assessment is factual. Assess knowledge by asking how much the patient knows about the illness or the reason for seeking health care. To assess past (long-term) memory, ask the patient to recall the maiden name of the patient's mother, a birthday, or a special date in history. It is best to ask open- ended questions rather than simple yes/no questions. Patients demonstrate immediate recall (recent memory) by repeating a series of numbers in the order in which they are presented or in reverse order.

A new nurse is working in a unit that uses interdisciplinary collaboration. Which action will the nurse take? a. Act as a leader of the health care team. b. Develop good communication skills. c. Work solely with nurses. d. Avoid conflict.

B Good communication between other health care providers builds trust and is related to the acceptance of your role in the health care team. As a beginning nurse, you will not be considered a leader of the health care team, but your input as an interdisciplinary team member is critical. Interdisciplinary involves other health care providers, not just nurses. Organizational culture includes leadership, communication processes, shared beliefs about the quality of clinical guidelines, and conflict resolution.

The nurse is assessing an adult patient's patellar reflex. Which finding will the nurse record as normal? a. 1+ b. 2+ c. 3+ d. 4+

B Grade reflexes as follows: 0: No response; 1+: Sluggish or diminished; 2+: Active or expected response; 3+: More brisk than expected, slightly hyperactive; and 4+: Brisk and hyperactive with intermittent or transient clonus.

The nurse is providing oral care to an unconscious patient and notes that the patient has extremely bad breath. Which term will the nurse use when reporting to the oncoming shift? a. Cheilitis b. Halitosis c. Glossitis d. Dental caries

B Halitosis is the term for "bad breath." Cheilitis is the term for cracked lips. Dental caries are cavities in the teeth and could be a cause of the halitosis. Glossitis is the term for inflamed tongue.

Which assessment finding for a patient who is receiving IV furosemide (Lasix) to treat stage 2 hypertension is most important to report to the health care provider? a. Blood glucose level of 175 mg/dL b. Blood potassium level of 3.0 mEq/L c. Most recent blood pressure (BP) reading of 168/94 mm Hg d. Orthostatic systolic BP decrease of 12 mm Hg

B Hypokalemia is a frequent adverse effect of the loop diuretics and can cause life-threatening dysrhythmias. The health care provider should be notified of the potassium level immediately and administration of potassium supplements initiated. The elevated blood glucose and BP also indicate a need for collaborative interventions but will not require action as urgently as the hypokalemia. An orthostatic drop of 12 mm Hg is common and will require intervention only if the patient is symptomatic.

The nurse is prescreening a surgical patient in the preadmission testing unit. The medication history indicates that the patient is currently taking an anticoagulant. Which action should the nurse take when consulting with the health care provider? a. Ask for a radiological examination of the chest. b. Ask for an international normalized ratio (INR). c. Ask for a blood urea nitrogen (BUN). d. Ask for a serum sodium (Na).

B INR, PT (prothrombin time), APTT (activated partial thromboplastin time), and platelet counts reveal the clotting ability of the blood. Anticoagulants can be utilized for different conditions, but its action is to increase the time it takes for the blood to clot. This action can put the surgical patient at risk for bleeding tendencies. Typically, if at all possible, this medication is held several days before a surgical procedure to decrease this risk. Chest x-ray, BUN, and Na are diagnostic screening tools for surgery but are not specific to anticoagulants.

The nurse assesses a circular, flat, reddened lesion about 5 cm in diameter on a middle-aged patients ankle. How should the nurse determine if the lesion is related to intra-dermal bleeding? a. Elevate the patients leg. b. Press firmly on the lesion. c. Check the temperature of the skin around the lesion. d. Palpate the dorsalis pedis and posterior tibial pulses.

B If the lesion is caused by intradermal or subcutaneous bleeding or a nonvascular cause, the discoloration will remain when direct pressure is applied to the lesion. If the lesion is caused by blood vessel dilation, blanching will occur with direct pressure. The other assessments will assess circulation to the leg, but will not be helpful in determining the etiology of the lesion.

The nurse is caring for a preoperative patient. The nurse teaches the principles and demonstrates leg exercises for the patient. The patient is unable to perform leg exercises correctly. What is the nurse's best next step? a. Encourage the patient to practice at a later date. b. Assess for the presence of anxiety, pain, or fatigue. c. Ask the patient why exercises are not being done. d. Evaluate the educational methods used to educate the patient.

B If the patient is unable to perform leg exercises, the nurse should look for circumstances that may be impacting the patient's ability to learn. In this case, the patient can be anticipating the upcoming surgery and may be experiencing anxiety. The patient may also be in pain or may be fatigued; both of these can affect the ability to learn. Evaluation of educational methods may be needed, but in this case, principles and demonstrations are being utilized. Asking anyone "why" can cause defensiveness and may not help in attaining the answer. The nurse is aware that the patient is unable to do the exercises. Moving forward without ascertaining that learning has occurred will not help the patient in meeting goals

When providing hygiene for an older-adult patient, the nurse closely assesses the skin. What is the rationale for the nurse's action? a. Outer skin layer becomes more resilient. b. Less frequent bathing may be required. c. Skin becomes less subject to bruising. d. Sweat glands become more active.

B In older adults, daily bathing as well as bathing with water that is too hot or soap that is harsh causes the skin to become excessively dry. As the patient ages, the skin thins and loses its resiliency and moisture, and lubricating skin glands become less active, making the skin fragile and prone to bruising and breaking.

Conjoined twins are in the neonatal department of the community hospital until transfer to the closest medical center. A photographer from the local newspaper gets off the elevator on the neonatal floor and wants to take pictures of the infants. Which initial action should the nurse take? a. Escort the cameraman to the neonatal unit while a few pictures are taken quietly. b. Tell the cameraman where the hospital's public relations department is located. c. Have the cameraman wait for permission from the health care provider. d. Ask the cameraman how the pictures are to be used in the newspaper.

B In some cases, information about a scientific discovery or a major medical breakthrough or an unusual situation is newsworthy. In this case, anyone seeking information needs to contact the hospital's public relations department to ensure that invasion of privacy does not occur. It is not the nurse's responsibility to decide independently the legality of disclosing information. The nurse does not have the right to allow the cameraman access to the neonatal unit. This would constitute invasion of privacy. The health care provider has no responsibility regarding this situation and cannot allow the cameraman on the unit. It is not the nurse's responsibility to find out how the pictures are to be used. This is a task for the public relations department.

A patient visiting with family members in the waiting area tells the nurse "I don't feel good, especially in the stomach." What should the nurse do? a. Request that the family leave, so the patient can rest. b. Ask the patient to return to the room, so the nurse can inspect the abdomen. c. Ask the patient when the last bowel movement was and to lie down on the sofa. d. Tell the patient that the dinner tray will be ready in 15 minutes and that may help the stomach feel better.

B In this case, the environment needs to be conducive to completing a thorough assessment. A patient's care environment needs to be safe and conducive to implementing therapies. When you need to expose a patient's body parts, do so privately by closing room doors or curtains because the patient will then be more relaxed; the patient needs to return to the room for an abdominal assessment for privacy and comfort. The family can remain in the waiting area while the nurse assists the patient back to the room. Beginning the assessment in the waiting area (lie down on the sofa) in the presence of family and other visitors does not promote privacy and patient comfort. Telling the patient that the dinner tray is almost ready is making an assumption that the abdominal discomfort is due to not eating. The nurse needs to perform an assessment first.

A 53-year-old patient with Stage D heart failure and type 2 diabetes asks the nurse whether heart transplant is a possible therapy. Which response by the nurse is most appropriate? a. Because you have diabetes, you would not be a candidate for a heart transplant. b. The choice of a patient for a heart transplant depends on many different factors. c. Your heart failure has not reached the stage in which heart transplants are needed. d. People who have heart transplants are at risk for multiple complications after surgery.

B Indications for a heart transplant include end-stage heart failure (Stage D), but other factors such as coping skills, family support, and patient motivation to follow the rigorous posttransplant regimen are also considered. Diabetic patients who have well-controlled blood glucose levels may be candidates for heart transplant. Although heart transplants can be associated with many complications, this response does not address the patients question.

A patient reports chronic itching of the ankles and continuously scratches the area. Which assessment finding will the nurse expect? a. Hypertrophied scars on both ankles b. Thickening of the skin around the ankles c. Yellowish-brown skin around both ankles d. Complete absence of melanin in both ankles

B Lichenification is likely to occur in areas where the patient scratches the skin frequently. Lichenification results in thickening of the skin with accentuated normal skin markings. Vitiligo is the complete absence of melanin in the skin. Keloids are hypertrophied scars. Yellowish-brown skin indicates jaundice. Vitiligo, keloids, and jaundice do not usually occur as a result of scratching the skin.

The nurse is examining a female with vaginal discharge. Which position will the nurse place the patient for proper examination? a. Sitting b. Lithotomy c. Knee-chest d. Dorsal recumbent

B Lithotomy is the position for examination of female genitalia. The lithotomy position provides for the maximum exposure of genitalia and allows the insertion of a vaginal speculum. Sitting does not allow adequate access for speculum insertion and is better used to visualize upper body parts. Dorsal recumbent is used to examine the head and neck, anterior thorax and lungs, breasts, axillae, heart, and abdomen. Knee-chest provides maximal exposure of the rectal area but is embarrassing and uncomfortable

A nurse works full time on the oncology unit at the hospital and works part time on weekends giving immunizations at the local pharmacy. While giving an injection on a weekend, the nurse caused injury to the patient's arm and is now being sued. How will the hospital's malpractice insurance provide coverage for this nurse? a. It will provide coverage as long as the nurse followed all procedures, protocols, and policies correctly. b. The hospital's malpractice insurance covers this nurse only during the time the nurse is working at the hospital. c. As long as the nurse has never been sued before this incident, the hospital's malpractice insurance will cover the nurse. d. The hospital's malpractice insurance will provide approximately 50% of the coverage the nurse will need.

B Malpractice insurance provided by the employing institution covers nurses only while they are working within the scope of their employment. It is always wise to find out if malpractice insurance is provided by a secondary place of employment, in this case, the pharmacy, or the nurse should carry an individual malpractice policy to cover situations such as this. The hospital policy would not provide coverage even if the nurse followed all procedures and policies or had never been sued. It will not provide 50% of coverage.

The patient has presented to the ambulatory surgery center to have a colonoscopy. The patient is scheduled to receive moderate sedation (conscious sedation) during the procedure. How will the nurse interpret this information? a. The procedure results in loss of sensation in an area of the body. b. The procedure requires a depressed level of consciousness. c. The procedure will be performed on an outpatient basis. d. The procedure necessitates the patient to be immobile.

B Moderate sedation (conscious sedation) is used routinely for procedures that do not require complete anesthesia but rather a depressed level of consciousness. Not all patients who are treated on an outpatient basis receive moderate sedation. Regional anesthesia such as local anesthesia provides loss of sensation in an area of the body. General anesthesia is used for patients who need to be immobile and to not remember the surgical procedure

The nurse is interviewing a patient with contact dermatitis. Which finding indicates a need for patient teaching? a. The patient applies corticosteroid cream to pruritic areas. b. The patient uses Neosporin ointment on minor cuts or abrasions. c. The patient adds oilated oatmeal (Aveeno) to the bath water every day. d. The patient takes diphenhydramine (Benadryl) at night if itching occurs.

B Neosporin can cause contact dermatitis. The other medications are being used appropriately by the patient.

A nurse is preparing to provide hygiene care. Which principle should the nurse consider when planning hygiene care? a. Hygiene care is always routine and expected. b. No two individuals perform hygiene in the same manner. c. It is important to standardize a patient's hygienic practices. d. During hygiene care do not take the time to learn about patient needs.

B No two individuals perform hygiene in the same manner; it is important to individualize the patient's care based on knowing about the patient's unique hygiene practices and preferences. Hygiene care is never routine; this care requires intimate contact with the patient and communication skills to promote the therapeutic relationship. In addition, during hygiene, the nurse should take time to learn about the patient's health promotion practices and needs, emotional needs, and health care education needs.

The nurse has a goal of becoming a certified registered nurse anesthetist (CRNA). Which activity is appropriate for a CRNA? a. Manages gynecological services such as PAP smears b. Works under the guidance of an anesthesiologist c. Obtains a PhD degree in anesthesiology d. Coordinates acute medical conditions

B Nurse anesthetists provide surgical anesthesia under the guidance and supervision of an anesthesiologist, who is a physician (health care provider) with advanced knowledge of surgical anesthesia. Nurse practitioners, not CRNAs, manage self-limiting acute and chronic stable medical conditions; certified nurse-midwives provide gynecological services such as routine Papanicolaou (Pap) smears. The CRNA is an RN with an advanced education in a nurse anesthesia accredited program. A PhD is not a requirement.

A nurse is a preceptor for a nurse who just graduated from nursing school. When caring for a patient, the new graduate nurse begins to explain to the patient the purpose of completing a physical assessment. Which statement made by the new graduate nurse requires the preceptor to intervene? a. "I will use the information from my assessment to figure out if your antihypertensive medication is working effectively." b. "Nursing assessment data are used only to provide information about the effectiveness of your medical care." c. "Nurses use data from their patient's physical assessment to determine a patient's educational needs." d. "Information gained from physical assessment helps nurses better understand their patients' emotional needs."

B Nursing assessment data are used to evaluate the effectiveness of all aspects of a patient's care, not just the patient's medical care. Assessment data help to evaluate the effectiveness of medications and to determine a patient's health care needs, including the need for patient education. Nurses also use assessment data to identify patients' psychosocial and cultural needs.

The nurse establishes trust and talks with a school-aged patient before administering an injection. Which type of implementation skill is the nurse using? a. Cognitive b. Interpersonal c. Psychomotor d. Judgmental

B Nursing practice includes cognitive, interpersonal, and psychomotor skills. Interpersonal skills involve developing trusting relationships with patients, conveying caring and compassion, and communicating clearly. Cognitive skills include critical thinking and decision-making skills. Psychomotor skill requires the integration of cognitive and motor abilities, such as administering the injection. Being judgmental is not appropriate in nursing; nurses are nonjudgmental.

An older-adult patient is taking aminoglycoside for a severe infection. Which assessment is the priority? a. Eyes b. Ears c. Skin d. Reflexes

B Older adults are especially at risk for hearing loss caused by ototoxicity (injury to auditory nerve) resulting from high maintenance doses of antibiotics (e.g., aminoglycosides). While eyes and skin are important, they are not the priority. Reflexes are expected to be diminished in older adults.

The nurse has administered a preoperative medication to the patient going to surgery. Which action will the nurse take next? a. Notify the operating suite that the medication has been given. b. Instruct the patient to call for help to go to the restroom. c. Waste any unused medication according to policy. d. Ask the patient to sign the consent for surgery.

B Once a preoperative medication has been administered, instruct the patient to call for help when getting out of bed to prevent falls. For patient safety, explain the purpose of a preoperative medication and its effects. Notifying the operating suite that the medication has been given may be part of a facilities procedure but is not the best next step. It is important to have the patient sign consents before the patient has received medication that may make him/her drowsy. Wasting unused medication according to policy is important but is not the best next step.

The nurse is caring for a patient who will undergo a removal of a lung lobe. Which level of care will the patient require immediately post procedure? a. Acute care—medical-surgical unit b. Acute care—intensive care unit c. Ambulatory surgery d. Ambulatory surgery—extended stay

B Patients undergoing extensive surgery and requiring anesthesia of long duration recover slowly. If a patient is undergoing major surgery such as a procedure on the lung, a stay in the hospital and specifically in the intensive care unit is required to monitor for potential risks to well-being. This patient would require more care than can be provided on a medical-surgical unit. It is not appropriate for this type of patient to go home after the procedure or to stay in an extended stay area of an ambulatory surgery area because of the complexity and associated risks.

Patients with heart failure can experience episodes of exacerbation. All of the patients below have a history of heart failure. Which of the following patients are at MOST risk for heart failure exacerbation? A. A 55 year old female who limits sodium and fluid intake regularly. B. A 73 year old male who reports not taking Amiodarone for one month and is experiencing atrial fibrillation. C. A 67 year old female who is being discharged home from heart valve replacement surgery. D. A 78 year old male who has a health history of eczema and cystic fibrosis.

B Patients who are in an arrhythmia (especially a-fib) are at risk for developing heart failure because the heart is not contracting properly and blood is pooling in the chambers.

A teenaged male patient who wrestles in high school is examined by the nurse in the clinic. Which assessment finding would prompt the nurse to teach the patient about the importance of not sharing headgear to prevent the spread of pediculosis? a. Ringlike rashes with red, scaly borders over the entire scalp b. Papular, wheal-like lesions with white deposits on the hair shaft c. Patchy areas of alopecia with small vesicles and excoriated areas d. Red, hivelike papules and plaques with sharply circumscribed borders

B Pediculosis is characterized by wheal-like lesions with parasites that attach eggs to the base of the hair shaft. The other descriptions are more characteristic of other types of skin disorders.

A patient who has diabetes and uses insulin to control blood glucose has been NPO since midnight before having a knee replacement surgery. Which action should the nurse take? a. Withhold the usual scheduled insulin dose because the patient is NPO. b. Obtain a blood glucose measurement before any insulin administration. c. Give the patient the usual insulin dose because stress will increase the blood glucose. d. Administer a lower dose of insulin because there will be no oral intake before surgery.

B Preoperative insulin administration is individualized to the patient, and the current blood glucose will provide the most reliable information about insulin needs. It is not possible to predict whether the patient will require no insulin, a lower dose, or a higher dose without blood glucose monitoring.

A nurse is teaching a patient and family about quality of life. Which information should the nurse include in the teaching session about quality of life? a. It is deeply social. b. It is hard to define. c. It is an observed measurement for most people. d. It is consistent and stable over the course of one's lifetime.

B Quality of life remains deeply individual (not social) and difficult to predict. Quality of life is not just a measurable entity but a shared responsibility. Quality of life measures may take into account the age of the patient, the patient's ability to live independently, his or her ability to contribute to society in a gainful way, and other nuanced measures of quality.

The nurse is caring for a patient who is scheduled to undergo a surgical procedure. The nurse is completing an assessment and reviews the patient's laboratory tests and allergies and prepares the patient for surgery. In which perioperative nursing phase is the nurse working? a. Perioperative b. Preoperative c. Intraoperative d. Postoperative

B Reviewing the patient's laboratory tests and allergies is done before surgery in the preoperative phase. Perioperative means before, during, and after surgery. Intraoperative means during the surgical procedure in the operating suite; postoperative means after the surgery and could occur in the postanesthesia care unit, in the ambulatory surgical area, or on the hospital unit.

Which of the following is a common side effect of Spironolactone? A. Renal failure B. Hyperkalemia C. Hypokalemia D. Dry cough

B Spironolactone is potassium-sparing. Therefore, it can increase the potassium level (hyperkalemia).

A patient with a history of hypertension treated with a diuretic and an ACE (angiotensin-converting enzyme) inhibitor arrives in the emergency department complaining of a severe headache and nausea and has a blood pressure (BP) of 238/118 mm Hg. Which question should the nurse ask first? a. Did you take any acetaminophen (Tylenol) today? b. Have you been consistently taking your medications? c. Have there been any recent stressful events in your life? d. Have you recently taken any antihistamine medications?

B Sudden withdrawal of antihypertensive medications can cause rebound hypertension and hypertensive crisis. Although many over-the-counter medications can cause hypertension, antihistamines and acetaminophen do not increase BP. Stressful events will increase BP but not usually to the level seen in this patient.

A pediatric oncology nurse floats to an orthopedic trauma unit. Which action should the nurse manager of the orthopedic unit take to enable safe care to be given by this nurse? a. Provide a complete orientation to the functioning of the entire unit. b. Determine patient acuity and care the nurse can safely provide. c. Allow the nurse to choose which mealtime works best. d. Assign nursing assistive personnel to assist with care.

B Supervisors are liable if they give staff nurses an assignment that they cannot safely handle. Nurses who float must inform the supervisor of any lack of experience in caring for the types of patients on the nursing unit. They should request and receive an orientation to the unit. A basic orientation is needed, whereas a complete orientation of the functioning of the entire unit would take a period of time that would exceed what the nurse has to spend on orientation. Allowing nurses to choose which mealtime they would like is a nice gesture of thanks for the nurse, but it does not enable safe care. Having nursing assistive personnel may help the nurse complete basic tasks such as hygiene and turning, but it does not enable safe nursing care that the nurse and manager are ultimately responsible for.

You are assisting a patient up from the bed to the bathroom. The patient has swelling in the feet and legs. The patient is receiving treatment for heart failure and is taking Hydralazine and Isordil. Which of the following is a nursing priority for this patient while assisting them to the bathroom? A. Measure and record the urine voided. B. Assist the patient up slowing and gradually. C. Place the call light in the patient's reach while in the bathroom. D. Provide privacy for the patient.

B The best answer for this particular question is option B. All the options are important for the nurse to perform. However, Hydralazine (vasodilator) and Isordil (nitrate) can cause orthostatic hypotension. The patient should transfer slowly and gradually to decrease dizziness and the risk of falling.

A new nurse notes that the health care unit keeps a listing of patient names in a closed book behind the front desk of the nursing station so patients can be located easily. Which action is most appropriate for the nurse to take? a. Talk with the nurse manager about the listing being a violation of the Health Insurance Portability and Accountability Act (HIPAA). b. Use the book as needed while keeping it away from individuals not involved in patient care. c. Move the book to the upper ledge of the nursing station for easier access. d. Ask the nurse manager to move the book to a more secluded area.

B The book is located where only staff would have access so the nurse can use the book as needed. The privacy section of the HIPAA provides standards regarding accountability in the health care setting. These rules include patient rights to consent to the use and disclosure of their protected health information, to inspect and copy their medical record, and to amend mistaken or incomplete information. It is not the responsibility of the new nurse to move items used by others on the patient unit. The listing is protected as long as it is used appropriately as needed to provide care. There is no need to move the book to a more secluded area.

SKIP The circulating nurse is caring for a patient intraoperatively. Which primary role of the circulating nurse will be implemented? a. Suturing the surgical incision in the OR suite b. Managing patient care activities in the OR suite c. Assisting with applying sterile drapes in the OR suite d. Handing sterile instruments and supplies to the surgeon in the OR suite

B The circulating nurse is an RN who remains unscrubbed and uses the nursing process in the management of patient care activities in the OR suite. The circulating nurse also manages patient positioning, antimicrobial skin preparation, medications, implants, placement and function of intermittent pneumatic compression (IPC) devices, specimens, warming devices and surgical counts of instruments, and dressings. The RN first assistant collaborates with the surgeon by handling and cutting tissue, using instruments and medical devices, providing exposure of the surgical area and hemostasis, and suturing. The scrub nurse, who can be a registered nurse, a licensed practical nurse, or a surgical technologist, maintains the sterile field, assists with applying the sterile drapes, and hands sterile instruments and supplies to the surgeon

An older-adult patient is being seen for chronic entropion. Which condition will the nurse assess for in this patient? a. Ptosis b. Infection c. Borborygmi d. Exophthalmos

B The diagnosis of entropion can lead to lashes of the lids irritating the conjunctiva and cornea. Irritation can lead to infection. Exophthalmos is a bulging of the eyes and usually indicates hyperthyroidism. An abnormal drooping of the lid over the pupil is called ptosis. In the older adult, ptosis results from a loss of elasticity that accompanies aging. Hyperactive sounds are loud, "growling" sounds called borborygmi, which indicate increased GI motility.

Which blood pressure (BP) finding by the nurse indicates that no changes in therapy are needed for a patient with stage 1 hypertension who has a history of diabetes mellitus? a. 102/60 mm Hg b. 128/76 mm Hg c. 139/90 mm Hg d. 136/82 mm Hg

B The goal for antihypertensive therapy for a patient with hypertension and diabetes mellitus is a BP <130/80 mm Hg. The BP of 102/60 may indicate overtreatment of the hypertension and an increased risk for adverse drug effects. The other two blood pressures indicate a need for modifications in the patients treatment.

The nurse is preparing a patient for surgery. Which goal is a priority for assessing the patient before surgery? a. Plan for care after the procedure. b. Establish a patient's baseline of normal function. c. Educate the patient and family about the procedure. d. Gather appropriate equipment for the patient's needs.

B The goal of the preoperative assessment is to identify a patient's normal preoperative function and the presence of any risks to recognize, prevent, and minimize possible postoperative complications. Gathering appropriate equipment, planning care, and educating the patient and family are all important interventions that must be provided for the surgical patient; they are part of the nursing process but are not the priority reason/goal for completing an assessment of the surgical patient.

The nurse questions a health care provider's decision to not tell the patient about a cancer diagnosis. Which ethical principle is the nurse trying to uphold for the patient? a. Consequentialism b. Autonomy c. Fidelity d. Justice

B The nurse is upholding autonomy. Autonomy refers to the freedom to make decisions free of external control. Respect for patient autonomy refers to the commitment to include patients in decisions about all aspects of care. Consequentialism is focused on the outcome and is a philosophical approach. Justice refers to fairness and is most often used in discussions about access to health care resources. Fidelity refers to the agreement to keep promises.

A nurse teaches a group of nursing students about nurse practice acts. Which information is most important to include in the teaching session about nurse practice acts? a. Protects the nurse b. Protects the public c. Protects the provider d. Protects the hospital

B The nurse practice acts regulate the scope of nursing practice and protect public health, safety, and welfare. They do not protect the nurse, provider, or hospital

The nurse is providing a complete bed bath to a patient using a commercial bath cleansing pack (bag bath). What should the nurse do? a. Rinse thoroughly. b. Allow the skin to air-dry. c. Do not use a bath towel. d. Dry the skin with a towel.

B The nurse should allow the skin to air-dry for 30 seconds. Drying the skin with a towel removes the emollient that is left behind after the water/cleanser solution evaporates. It is permissible to lightly cover the patient with a bath blanket or towel to prevent chilling. Do not rinse when using a bag bath

An older patient is being discharged from the ambulatory surgical unit following left eye surgery. The patient tells the nurse, I do not know if I can take care of myself with this patch over my eye. Which action by the nurse is most appropriate? A. Refer the patient for home health care services. B. Discuss the specific concerns regarding self-care. C. Give the patient written instructions regarding care. D. Assess the patients support system for care at home.

B The nurses initial action should be to assess exactly the patients concerns about self-care. referral to home health care & assessment of the patients support system may be appropriate actions but will be based on further assessment of the patients concerns. Written instructions should be given to the patient, but these are unlikely to address the patients stated concern about self-care.

An older patient who had knee replacement surgery 2 days ago can only tolerate being out of bed with physical therapy twice a day. Which collaborative problem should the nurse identify as a priority for this patient? a. Potential complication: hypovolemic shock b. Potential complication: venous thromboembolism c. Potential complication: fluid and electrolyte imbalance d. Potential complication: impaired surgical wound healing

B The patient is older and relatively immobile, which are two risk factors for development of deep vein thrombosis. The other potential complications are possible postoperative problems, but they are not supported by the data about this patient.

Which action will the nurse in the hypertension clinic take in order to obtain an accurate baseline blood pressure (BP) for a new patient? a. Deflate the BP cuff at a rate of 5 to 10 mm Hg per second. b. Have the patient sit in a chair with the feet flat on the floor. c. Assist the patient to the supine position for BP measurements. d. Obtain two BP readings in the dominant arm and average the results.

B The patient should be seated with the feet flat on the floor. The BP is obtained in both arms, and the results of the two arms are not averaged. The patient does not need to be in the supine position. The cuff should be deflated at 2 to 3 mm Hg per second.

Five minutes after receiving the ordered preoperative midazolam (Versed) by IV injection, the patient asks to get up to go to the bathroom to urinate. Which action by the nurse is most appropriate? a. Assist the patient to the bathroom and stay with the patient to prevent falls. b. Offer a urinal or bedpan and position the patient in bed to promote voiding. c. Allow the patient up to the bathroom because medication onset is 10 minutes. d. Ask the patient to wait because catheterization is performed just before the surgery

B The patient will be at risk for a fall after receiving the sedative, so the best nursing action is to have the patient use a bedpan or urinal. Having the patient get up either with assistance or independently increases the risk for a fall. The patient will be uncomfortable and risk involuntary incontinence if the bladder is full during transport to the operating room

In the postanesthesia care unit (PACU), a patients vital signs are blood pressure 116/72, pulse 74, respirations 12, and SpO2 91%. The patient is sleepy but awakens easily. Which action should the nurse take first? a. Place the patient in a side-lying position. b. Encourage the patient to take deep breaths. c. Prepare to transfer the patient to a clinical unit. d. Increase the rate of the postoperative IV fluids.

B The patients borderline SpO2 and sleepiness indicate hypoventilation. The nurse should stimulate the patient and remind the patient to take deep breaths. Placing the patient in a lateral position is needed when the patient first arrives in the PACU and is unconscious. The stable blood pressure and pulse indicate that no changes in fluid intake are required. The patient is not fully awake and has a low SpO2, indicating that transfer from the PACU to a clinical unit is not appropriate.

The nurse is assessing a patient who has been admitted to the intensive care unit (ICU) with a hypertensive emergency. Which finding is most important to report to the health care provider? a. Urine output over 8 hours is 250 mL less than the fluid intake. b. The patient cannot move the left arm and leg when asked to do so. c. Tremors are noted in the fingers when the patient extends the arms. d. The patient complains of a headache with pain at level 8/10 (0 to 10 scale).

B The patients inability to move the left arm and leg indicates that a hemorrhagic stroke may be occurring and will require immediate action to prevent further neurologic damage. The other clinical manifestations are also likely caused by the hypertension and will require rapid nursing actions, but they do not require action as urgently as the neurologic changes.

After receiving change-of-shift report on a heart failure unit, which patient should the nurse assess first? a. Patient who is taking carvedilol (Coreg) and has a heart rate of 58 b. Patient who is taking digoxin and has a potassium level of 3.1 mEq/L c. Patient who is taking isosorbide dinitrate/hydralazine (BiDil) and has a headache d. Patient who is taking captopril (Capoten) and has a frequent nonproductive cough

B The patients low potassium level increases the risk for digoxin toxicity and potentially fatal dysrhythmias. The nurse should assess the patient for other signs of digoxin toxicity and then notify the health care provider about the potassium level. The other patients also have side effects of their medications, but their symptoms do not indicate potentially life-threatening complications.

The patient requires routine gynecological services after giving birth to her son, and while seeing the nurse-midwife, the patient asks for a referral to a pediatrician for the newborn. Which action should the nurse-midwife take initially? a. Provide the referral as requested. b. Offer to provide the newborn care. c. Refer the patient to the supervising provider. d. Tell the patient that is not allowed to make referrals.

B The practice of nurse-midwifery involves providing independent care for women during normal pregnancy, labor, and delivery, as well as care for the newborn. After being apprised of the midwifery role, if the patient insists on seeing a pediatrician, the nurse-midwife should provide the referral. The supervising provider is an obstetric provider, not a pediatrician. A nurse-midwife can make referrals.

The nurse obtains the following information from a patient newly diagnosed with prehypertension. Which finding is most important to address with the patient? a. Low dietary fiber intake b. No regular aerobic exercise c. Weight 5 pounds above ideal weight d. Drinks a beer with dinner on most nights

B The recommendations for preventing hypertension include exercising aerobically for 30 minutes most days of the week. A weight that is 5 pounds over the ideal body weight is not a risk factor for hypertension. The Dietary Approaches to Stop Hypertension (DASH) diet is high in fiber, but increasing fiber alone will not prevent hypertension from developing. The patients alcohol intake is within guidelines and will not increase the hypertension risk.

Which action could the post-anesthesia care unit (PACU) nurse delegate to unlicensed assistive personnel (UAP) who help with the transfer of a patient to the clinical unit? a. Clarify the postoperative orders with the surgeon. b. Help with the transfer of the patient onto a stretcher. c. Document the appearance of the patients incision in the chart. d. Provide hand off communication to the surgical unit charge nurse.

B The scope of practice of UAP includes repositioning and moving patients under the supervision of a nurse. Providing report to another nurse, assessing and documenting the wound appearance, and clarifying physician orders with another nurse require registered-nurse (RN) level education and scope of practice

A nurse is providing a bath. In which order will the nurse clean the body, beginning with the first area? 1. Face 2. Eyes 3. Perineum 4. Arm and chest 5. Hands and nails 6. Back and buttocks 7. Abdomen and legs a. 1, 2, 5, 4, 7, 6, 3 b. 2, 1, 4, 5, 7, 3, 6 c. 2, 1, 5, 4, 6, 7, 3 d. 1, 2, 4, 5, 3, 7, 6

B The sequence for giving a bath is as follows: eyes, face, both arms, chest, hands/nails, abdomen, both legs, perineal hygiene, back, and buttocks/anus.

While assessing the skin of an 82-year-old patient, a nurse discovers nonpainful, ruby red papules on the patient's trunk. What is the nurse's next action? a. Explain that the patient has basal cell carcinoma and should watch for spread. b. Document cherry angiomas as a normal older adult skin finding. c. Tell the patient that this is a benign squamous cell carcinoma. d. Record the presence of petechiae.

B The skin is normally free of lesions, except for common freckles or age-related changes such as skin tags, senile keratosis (thickening of skin), cherry angiomas (ruby red papules), and atrophic warts. Basal cell carcinoma is most common in sun-exposed areas and frequently occurs in a background of sun-damaged skin; it almost never spreads to other parts of the body. Squamous cell carcinoma is more serious than basal cell and develops on the outer layers of sun-exposed skin; these cells may travel to lymph nodes and throughout the body. Report abnormal lesions to the health care provider for further examination. Petechiae are nonblanching, pinpoint-size, red or purple spots on the skin caused by small hemorrhages in the skin layers.

The nurse is preparing to witness the patient signing the operative consent form when the patient says, I do not really understand what the doctor said. Which action is best for the nurse to take? a. Provide an explanation of the planned surgical procedure. b. Notify the surgeon that the informed consent process is not complete. c. Administer the prescribed preoperative antibiotics and withhold any ordered sedative medications. d. Notify the operating room staff that the surgeon needs to give a more complete explanation of the procedure.

B The surgeon is responsible for explaining the surgery to the patient, and the nurse should wait until the surgeon has clarified the surgery before having the patient sign the consent form. The nurse should communicate directly with the surgeon about the consent form rather than asking other staff to pass on the message. It is not within the nurses legal scope of practice to explain the surgical procedure. No preoperative medications should be administered until the patient understands the surgical procedure and signs the consent form.

A 38-year-old female is admitted for an elective surgical procedure. Which information obtained by the nurse during the preoperative assessment is most important to report to the anesthesiologist before surgery? a. The patients lack of knowledge about postoperative pain control measures b. The patients statement that her last menstrual period was 8 weeks previously c. The patients history of a postoperative infection following a prior cholecystectomy d. The patients concern that she will be unable to care for her children postoperatively

B This statement suggests that the patient may be pregnant, and pregnancy testing is needed before administration of anesthetic agents. Although the other data may also be communicated with the surgeon and anesthesiologist, they will affect postoperative care and do not indicate a need for further assessment before surgery.

The nurse is teaching a patient about contact lens care. Which instructions will the nurse include in the teaching session? a. Use tap water to clean soft lenses. b. Wash and rinse lens storage case daily. c. Reuse storage solution for up to a week. d. Keep the lenses is a cool dry place when not being used.

B Thoroughly wash and rinse lens storage case on a daily basis. Clean periodically with soap or liquid detergent, rinse thoroughly with warm water, and air-dry. Do not use tap water to clean soft lenses. Lenses should be kept moist or wet when not worn. Use fresh solution daily when storing and disinfecting lenses.

The nurse is preparing to provide a complete bed bath to an unconscious patient. The nurse decides to use a bag bath. In which order will the nurse clean the body, starting with the first area? 1. Neck, shoulders, and chest 2. Abdomen and groin/perineum 3. Legs, feet, and web spaces 4. Back of neck, back, and then buttocks 5. Both arms, both hands, web spaces, and axilla a. 5, 1, 2, 3, 4 b. 1, 5, 2, 3, 4 c. 1, 5, 2, 4, 3 d. 5, 1, 2, 4, 3

B Use all six chlorhexidene gluconate (CHG) cloths in the following order: 1. Cloth 1: Neck, shoulders, and chest 2. Cloth 2: Both arms, both hands, web spaces, and axilla 3. Cloth 3: Abdomen and then groin/perineum 4. Cloth 4: Right leg, right foot, and web spaces 5. Cloth 5: Left leg, left foot, and web spaces 6. Cloth 6: Back of neck, back, and then buttocks

During a severe respiratory epidemic, the local health care organizations decide to give health care workers priority access to ventilators over other members of the community who also need that resource. Which philosophy would give the strongest support for this decision? a. Deontology b. Utilitarianism c. Ethics of care d. Feminist ethics

B Utilitarianism focuses on the greatest good for the most people; the organizations decide to ensure that as many health care workers as possible will survive to care for other members of the community. Deontology defines actions as right or wrong based on their "right-making characteristics" such as fidelity to promises, truthfulness, and justice. Feminist ethics looks to the nature of relationships to guide participants in making difficult decisions, especially relationships in which power is unequal or in which a point of view has become ignored or invisible. The ethics of care and feminist ethics are closely related, but ethics of care emphasizes the role of feelings.

When professionals work together to solve ethical dilemmas, nurses must examine their own values. What is the best rationale for this step? a. So fact is separated from opinion b. So different perspectives are respected c. So judgmental attitudes can be provoked d. So the group identifies the one correct solution

B Values are personal beliefs that influence behavior. To negotiate differences of value, it is important to be clear about your own values: what you value, why, and how you respect your own values even as you try to respect those of others whose values differ from yours. Ethical dilemmas are a problem in that no one right solution exists. It is not to separate fact from opinion. Judgmental attitudes are not to be used, much less provoked.

A patient uses an in-the-canal hearing aid. Which assessment is a priority? a. Eyeglass usage b. Cerumen buildup c. Type of physical exercise d. Excessive moisture problems

B With this type of model (in-the-canal), cerumen tends to plug this model more than others. There are three popular types of hearing aids. An in-the-canal (ITC) aid is the newest, smallest, and least visible and fits entirely in the ear canal. It has cosmetic appeal, is easy to manipulate and place in the ear, and does not interfere with wearing eyeglasses or using the telephone, and the patient can wear it during most physical exercise. An in-the-ear aid (ITE, or intra-aural) is more noticeable than the ITC aid and is not for people with moisture or skin problems in the ear canal. The larger size of this type of aid (behind-the-ear, BTE, or post-aural) can make use of eyeglasses and phones difficult; it is more difficult to keep in place during physical exercise.

The nurse is teaching a new nurse about protocols. Which information from the new nurse indicates a correct understanding of the teaching? a. Protocols are guidelines to follow that replace the nursing care plan. b. Protocols assist the clinician in making decisions and choosing interventions for specific health care problems or conditions. c. Protocols are policies designating each nurse's duty according to standards of care and a code of ethics. d. Protocols are prescriptive order forms that help individualize the plan of care.

B A clinical practice guideline or protocol is a systematically developed set of statements that helps nurses, physicians, and other health care providers make decisions about appropriate health care for specific clinical situations. This guideline establishes interventions for specific health care problems or conditions. The protocol does not replace the nursing care plan. Evidence-based guidelines from protocols can be incorporated into an individualized plan of care. A clinical guideline is not the same as a hospital policy. Standing orders contain orders for the care of a specific group of patients. A protocol is not a prescriptive order form like a standing order

The nurse is teaching the patient about flossing and oral hygiene. Which instruction will the nurse include in the teaching session? a. Using waxed floss prevents bleeding. b. Flossing removes plaque and tartar from the teeth. c. Performing flossing at least 3 times a day is beneficial. d. Applying toothpaste to the teeth before flossing is harmful.

B Dental flossing removes plaque and tartar between teeth. To prevent bleeding, the patient should use unwaxed floss. Flossing once a day is sufficient. If toothpaste is applied to the teeth before flossing, fluoride will come in direct contact with tooth surfaces, aiding in cavity prevention

A nurse is experiencing an ethical dilemma with a patient. Which information indicates the nurse has a correct understanding of the primary cause of ethical dilemmas? a. Unequal power b. Presence of conflicting values c. Judgmental perceptions of patients d. Poor communication with the patient

B Ethical dilemmas almost always occur in the presence of conflicting values. While unequal power, judgmental perceptions, and poor communication can contribute to the dilemma, these are not causes of a dilemma. Without clarification of values, the nurse may not be able to distinguish fact from opinion or value, and this can lead to judgmental attitudes.

The nurse has become aware of missing narcotics in the patient care area. Which ethical principle obligates the nurse to report the missing medications? a. Advocacy b. Responsibility c. Confidentiality d. Accountability

B Responsibility refers to one's willingness to respect and adhere to one's professional obligations. It is the nurse's responsibility to report missing narcotics. Accountability refers to the ability to answer for one's actions. Advocacy refers to the support of a particular cause. The concept of confidentiality is very important in health care and involves protecting patients' personal health information.

The nurse has been working in the clinical setting for several years as an advanced practice nurse. However, the nurse has a strong desire to pursue research and theory development. To fulfill this desire, which program should the nurse attend? a. Doctor of Nursing Science degree (DNSc) b. Doctor of Philosophy degree (PhD) c. Doctor of Nursing Practice degree (DNP) d. Doctor in the Science of Nursing degree (DSN)

B Some doctoral programs prepare nurses for more rigorous research and theory development and award the research-oriented Doctor of Philosophy (PhD) in nursing. Professional doctoral programs in nursing (DSN or DNSc) prepare graduates to apply research findings to clinical nursing. The DNP is a practice doctorate that prepares advanced practice nurses such as nurse practitioners.

The nurse is caring for a patient with cognitive impairments. Which actions will the nurse take during AM care? (Select all that apply.) a. Administer ordered analgesic 1 hour before bath time. b. Increase the frequency of skin assessment. c. Reduce triggers in the environment. d. Keep the room temperature cool. e. Be as quick as possible.

B, C If a patient is physically dependent or cognitively impaired, increase the frequency of skin assessment. Adapt your bathing procedures and the environment to reduce the triggers. For example, administer any ordered analgesic 30 minutes before a bath and be gentle in your approach. Keep the patient's body as warm as possible with warm towels and be sure the room temperature is comfortable.

Which patients below are best assigned to the LPN? (Select all that apply.) A. A 30-year-old male patient with active GI bleeding that requires multiple blood transfusions. B. A 78-year-old female with osteoporosis who needs assistance performing range of motion exercises and ambulating with a walker. C. A 29-year-old male patient who is post-op day 6 from a colostomy placement that is on a clear liquid diet. D. A 55-year-old male patient who reports chest pain and has ST segment elevation on his EKG.

B, C LPNs should be assigned STABLE patients with predicable outcomes and cases that don't require critical thinking or complex analysis. The patients in options A and D are unstable and require constant care with decisions being based on how to interpret patient findings.

Select all the correct statements about educating the patient with heart failure: A. It is important patients with heart failure notify their physician if they gain more than 6 pounds in a day or 10 pounds in a week. B. Patients with heart failure should receive an annual influenza vaccine and be up-to-date with the pneumonia vaccine." C. Heart failure patients should limit sodium intake to 2-3 grams per day. D. Heart failure is exacerbated by illness, too much fluid or sodium intake, and arrhythmias. E. Patients with heart failure should limit exercise because of the risks.

B, C, D Option A is wrong because heart failure patients should notify their doctor if they gain 2-3 pounds in a day or 5 pounds in a week, and option E is wrong because exercise is important for heart failure patients to help strengthen the heart muscle...so they should exercise as tolerated.

When delegating you know that as an RN you must follow the 5 Rights of Delegation to make sure you are delegating properly. Select all of the 5 Rights of Delegation: A. Right Credentials B. Right Direction/Communication C. Right Supervision D. Right Experience E. Right Task F. Right Person G. Right Patient H. Right Circumstance I. Right Time J. Right Order

B, C, E, F, H The 5 Rights of Delegation are: Right Task, Right Circumstance, Right Person, Right Direction/Communication, and Right Supervision.

When delegating you know that as an RN you must follow the 5 Rights of Delegation to make sure you are delegating properly. Select all the 5 Rights of Delegation: (Select all that apply.) A. Right Credentials B. Right Direction/Communication C. Right Supervision D. Right Experience E. Right Task F. Right Person G. Right Patient H. Right Circumstance I. Right Time J. Right Order

B, C, E, F, H The 5 Rights of Delegation are: Right Task, Right Circumstance, Right Person, Right Direction/Communication, and Right Supervision.

Which patients below are best assigned to the LPN? (Select all that apply.) A. A 30-year-old male patient with active GI bleeding that requires multiple blood transfusions. B. A 78-year-old female with osteoporosis who needs assistance performing range of motion exercises and ambulating with a walker. C. A 29-year-old male patient who is post-op day 6 from a colostomy placement that is on a clear liquid diet. D. A 55-year-old male patient who reports chest pain and has ST segment elevation on his EKG.

B, C. LPNs should be assigned STABLE patients with predicable outcomes and cases that don't require critical thinking or complex analysis. The patients in options A and D are unstable and require constant care with decisions being based on how to interpret patient findings.

Select all the correct statements about the pharmacodynamics of Beta-blockers for the treatment of heart failure: A. These drugs produce a negative inotropic effect on the heart by increasing myocardial contraction. B. A side effect of these drugs include bradycardia. C. These drugs are most commonly prescribed for patients with heart failure who have COPD. D. Beta-blockers are prescribed with ACE or ARBs to treat heart failure.

B, D

A nurse is teaching the staff about professional negligence or malpractice. Which criteria to establish negligence will the nurse include in the teaching session? (Select all that apply.) a. Injury did not occur. b. That duty was breached. c. Nurse carried out the duty. d. Duty of care was owed to the patient. e. Patient understands benefits and risks of a procedure.

B, D Certain criteria are necessary to establish nursing malpractice: (1) the nurse (defendant) owed a duty of care to the patient (plaintiff), (2) the nurse did not carry out or breached that duty, (3) the patient was injured, and (4) the nurse's failure to carry out the duty caused the injury. If an injury did not occur and the nurse carried out the duty, no malpractice occurred. When a patient understands benefits and risks of the procedure, that is informed consent, not malpractice.

After licensure, the nurse wants to stay current in knowledge and skills. Which programs are the most common ways nurses can do this? (Select all that apply.) a. Master's degree b. Inservice education c. Doctoral preparation d. Continuing education e. National Council Licensure Examination retakes

B, D Continuing education programs help nurses maintain current nursing skills, gain new knowledge and theory, and obtain new skills reflecting the changes in the health care delivery system. Inservice education programs are provided by a health care facility to increase the knowledge, skills, & competencies of nurses employed by the institution. Both can help the nurse stay current. Master's degree programs are valuable for those in the role of nurse educator, nurse administrator, or advanced practice nurse. Professional doctoral programs in nursing (DSN or DNSc) prepare graduates to apply research findings to clinical nursing. National Council Licensure Examination retakes are not to keep current; this test is taken to enter RN practice.

The nurse is using a forced air warmer for a surgical patient preoperatively. Which goals is the nurse trying to achieve? (Select all that apply.) a. Induce shivering. b. Reduce blood loss. c. Induce pressure ulcers. d. Reduce cardiac arrests. e. Reduce surgical site infection.

B, D, E Evidence suggests that pre-warming for a minimum of 30 minutes may reduce the occurrence of hypothermia. Prevention of hypothermia (core temperature < 36° C) helps to reduce complications such as shivering, cardiac arrest, blood loss, SSI, pressure ulcers, and mortality.

The nurse is providing preoperative education and reviews with the patient what it will be like to be in the surgical environment. Which points should the nurse include in the teaching session? (Select all that apply.) a. The operative suite will be very dark. b. The family is not allowed in the operating suite. c. The operating table or bed will be comfortable and soft. d. The nurses will be there to assist you through this process. e. The surgical staff will be dressed in special clothing with hats and masks.

B, D, E The surgical staff is dressed in special clothing, hats, and masks—all for infection control. Families are not allowed in the operating suite for several reasons, which include infection control and sterility. The nurse is there as the coordinator and patient advocate during a surgical procedure. The rooms are very bright so everyone can see, and the operating table is very uncomfortable for the patient.

Which of the following are NOT typical signs and symptoms of right-sided heart failure? Select-all-that-apply: A. Jugular venous distention B. Persistent cough C. Weight gain D. Crackles E. Nocturia F. Orthopnea

B, D, F Persistent cough, crackles (also called rales), and orthopnea are signs and symptoms of LEFT-sided heart failure...not right-sided heart failure.

What is a potential postoperative concern regarding a patient who has already resumed a solid diet? A. Failure to pass stool within 12 hours of eating solid foods B. Failure to pass stool within 48 hours of eating solid foods C. Passage of excessive flatus D. Patient reports a decreased appetite

B. After a patient resumes solid food, they should have a bowel movement within 48 hours. The patient may be experiencing constipation and appropriate interventions must be followed.

The nurse is monitoring the patient who is 24 hours post-opt from surgery. Which finding requires intervention? A. BP 100/80 B. 24-hour urine output of 300 ml C. Pain rating of 4 on 1-10 scale D. Temperature of 99.3' F

B. The nurse needs to watch the patient's urinary output closely. Urinary output within a 24-hour period should be at least 30 ml/hr. In this case, the patient is only urinating 12.5 ml/hr.

As the nurse you are getting the patient ready for surgery. You are completing the preoperative checklist. Which of the following is not part of the preoperative checklist? A. Assess for allergies B. Conducting the Time Out C. Informed consent is signed D. Ensuring that the history and physical examination has been completed

B. The time out is conducted by the OR nurse prior to surgery. All of the other options are conducted by the nurse getting the patient ready for surgery.

A patient is recovering from surgery. The patient is very restless, heart rate is 120 bpm and blood pressure is 70/53, skin is cool/clammy. As the nurse you would? A. Continue to monitor the patient B. Notify the MD C. Obtain an EKG D. Check the patient's blood glucose

B. This is an emergency situation. The patient is more than likely experiencing a hemorrhage of some type. Notifying the MD would be the first line of action and then you could check the patient's blood glucose and obtain an EKG. This patient is probably going to need a surgical intervention.

Assault., - is when the Nurse states that she will wrap a bandage over patient's mouth if he won't be quiet.

Battery., - is when the Nurse applies abdominal bandage after refusal.

A patients T-tube is draining dark green fluid after gallbladder surgery. What action by the nurse is the most appropriate? a. Notify the patients surgeon. b. Place the patient on bed rest. c. Document the color and amount of drainage. d. Irrigate the T-tube with sterile normal saline.

C A T-tube normally drains dark green to bright yellow drainage, so no action other than to document the amount and color of the drainage is needed. The other actions are not necessary

The nurse completed assessments on several patients. Which assessment finding will the nurse record as normal? a. Pulse strength 3 b. 1+ pitting edema c. Constricting pupils when directly illuminated d. Hyperactive bowel sounds in all four quadrants

C A normal finding is pupils constricting when directly illuminated with a penlight. A pulse strength of 3 indicates a full or increased pulse; 2 is normal. 1+ pitting edema is abnormal; there should be no edema for a normal finding. Hyperactive bowel sounds are abnormal and indicate increased GI motility; normal bowel sounds are active

A self-sufficient bedridden patient is unable to reach all body parts. Which type of bath will the nurse assign to the nursing assistive personnel? a. Bag bath b. Sponge bath c. Partial bed bath d. Complete bed bath

C A partial bath consists of washing body parts that the patient cannot reach, including the back, and providing a backrub. Dependent patients in need of partial hygiene or self-sufficient bedridden patients who are unable to reach all body parts receive a partial bed bath. Complete bed baths are administered to totally dependent patients in bed. The bag bath contains several soft, nonwoven cotton cloths that are premoistened in a solution of no-rinse surfactant cleanser and emollient. The sponge bath involves bathing from a bath basin or a sink with the patient sitting in a chair

The paramedics transport an adult involved in a motor vehicle accident to the emergency department. On physical examination, the patient's level of consciousness is reported as opening eyes to pain and responding with inappropriate words and flexion withdrawal to painful stimuli. Which value will the nurse report for the patient's Glasgow Coma Scale score? a. 5 b. 7 c. 9 d. 11

C According to the guidelines of the Glasgow Coma Scale, the patient has a score of 9. Opening eyes to pain is 2 points; inappropriate word use is 3 points; and flexion withdrawal is 4 points. The total for this patient is 2 + 3 + 4 = 9.

The nurse is monitoring a patient in the postanesthesia care unit (PACU) for postoperative fluid and electrolyte imbalance. Which action will be most appropriate for the nurse to take? a. Encourage copious amounts of water. b. Start an additional intravenous (IV) line. c. Measure and record all intake and output. d. Weigh the patient and compare with preoperative weight.

C Accurate recording of intake and output assesses renal and circulatory function. Measure and record all sources of intake and output. Encouraging copious amounts of water in a postoperative patient might encourage nausea and vomiting. In the PACU, it is impractical to weigh the patient while waking from surgery, but in the days afterward, it is a good assessment parameter for fluid imbalance. Starting an additional IV is not necessary and is not important at this juncture.

The nurse is caring for a postoperative patient on the medical-surgical floor. Which activity will the nurse encourage to prevent venous stasis and the formation of thrombus? a. Diaphragmatic breathing b. Incentive spirometry c. Leg exercises d. Coughing

C After general anesthesia, circulation slows, and when the rate of blood slows, a greater tendency for clot formation is noted. Immobilization results in decreased muscular contractions in the lower extremities; these promote venous stasis. Coughing, diaphragmatic breathing, and incentive spirometry are utilized to decrease atelectasis and pneumonia.

The nurse is caring for a patient who requires a complex dressing change. While in the patient's room, the nurse decides to change the dressing. Which action will the nurse take just before changing the dressing? a. Gathers and organizes needed supplies b. Decides on goals and outcomes for the patient c. Assesses the patient's readiness for the procedure d. Calls for assistance from another nursing staff member

C Always be sure a patient is physically and psychologically ready for any interventions or procedures. After determining the patient's readiness for the dressing change, the nurse gathers needed supplies. The nurse establishes goals and outcomes before intervening. The nurse needs to ask another staff member to help if necessary after determining readiness of the patient.

The nasogastric (NG) tube is removed on the second postoperative day, and the patient is placed on a clear liquid diet. Four hours later, the patient complains of sharp, cramping gas pains. What action by the nurse is the most appropriate? a. Reinsert the NG tube. b. Give the PRN IV opioid. c. Assist the patient to ambulate. d. Place the patient on NPO status.

C Ambulation encourages peristalsis and the passing of flatus, which will relieve the patients discomfort. If distention persists, the patient may need to be placed on NPO status, but usually this is not necessary. Morphine administration will further decrease intestinal motility. Gas pains are usually caused by trapping of flatus in the colon, and reinsertion of the NG tube will not relieve the pains.

The nurse is caring for a patient in preadmission testing. The patient has been assigned a physical status classification by the American Society of Anesthesiologists of ASA III. Which assessment will support this classification? a. Normal, healthy patient b. Denial of any major illnesses or conditions c. Poorly controlled hypertension with implanted pacemaker d. Moribund patient not expected to survive without the operation

C An ASA III rating is a patient with a severe systemic disease, such as poorly controlled hypertension with an implanted pacemaker. - ASA I is a normal healthy patient with no major illnesses or conditions. - ASA II is a patient with mild systemic disease. - ASA V is a moribund patient who is not expected to survive without the operation and includes patients with ruptured abdominal/thoracic aneurysm or massive trauma

A patient who is receiving dobutamine (Dobutrex) for the treatment of acute decompensated heart failure (ADHF) has the following nursing interventions included in the plan of care. Which action will be most appropriate for the registered nurse (RN) to delegate to an experienced licensed practical/vocational nurse (LPN/LVN)? a. Assess the IV insertion site for signs of extravasation. b. Teach the patient the reasons for remaining on bed rest. c. Monitor the patients blood pressure and heart rate every hour. d. Titrate the rate to keep the systolic blood pressure >90 mm Hg.

C An experienced LPN/LVN would be able to monitor BP and heart rate and would know to report significant changes to the RN. Teaching patients, making adjustments to the drip rate for vasoactive medications, and monitoring for serious complications such as extravasation require RN level education and scope of practice.

A patient in the dermatology clinic is scheduled for removal of a 15-mm multicolored and irregular mole from the upper back. The nurse should prepare the patient for which type of biopsy? a. Shave biopsy b. Punch biopsy c. Incisional biopsy d. Excisional biopsy

C An incisional biopsy would remove the entire mole and the tissue borders. The appearance of the mole indicates that it may be malignant. A shave biopsy would not remove the entire mole. The mole is too large to be removed with punch biopsy. Excisional biopsies are done for smaller lesions and where a good cosmetic effect is desired, such as on the face.

Which diagnostic test will be most useful to the nurse in determining whether a patient admitted with acute shortness of breath has heart failure? a. Serum troponin b. Arterial blood gases c. B-type natriuretic peptide d. 12-lead electrocardiogram

C B-type natriuretic peptide (BNP) is secreted when ventricular pressures increase, as they do with heart failure. Elevated BNP indicates a probable or very probable diagnosis of heart failure. A twelve-lead electrocardiogram, arterial blood gases, and troponin may also be used in determining the causes or effects of heart failure but are not as clearly diagnostic of heart failure as BNP.

The nurse is preparing a patient for a surgical procedure on the right great toe. Which action will be most important to include in this patient's preparation? a. Place the patient in a clean surgical gown. b. Ask the patient to remove all hairpins and cosmetics. c. Ascertain that the surgical site has been correctly marked. d. Determine where the family will be located during the procedure.

C Because errors have occurred in the past with patients undergoing the wrong surgery on the wrong site, the universal protocol guidelines have been implemented and are used with all invasive procedures. Part of this protocol includes marking the operative site with indelible ink. Knowing where the family is during a procedure, placing the patient in a clean gown, and asking the patient to remove all hairpins and cosmetics are important but are not most important in this list of items.

A patient is transferred from the post-anesthesia care unit (PACU) to the clinical unit. Which action by the nurse on the clinical unit should be performed first? a. Assess the patients pain. b. Orient the patient to the unit. c. Take the patients vital signs. d. Read the postoperative orders.

C Because the priority concerns after surgery are airway, breathing, and circulation, the vital signs are assessed first. The other actions should take place after the vital signs are obtained and compared with the vital signs before transfer.

A newly admitted patient who is morbidly obese asks the nurse for assistance to the bathroom for the first time. Which action should the nurse take initially? a. Ask for at least two other assistive personnel to come to the room. b. Medicate the patient to alleviate discomfort while ambulating. c. Review the patient's activity orders. d. Offer the patient a walker.

C Before beginning care, review the plan to determine the need for assistance and the type required. Before intervening, the nurse must check the patient's orders. For example, if the patient is on bed rest, the nurse will need to explain the use of a bedpan rather than helping the patient get out of bed to go to the bathroom. Asking for assistive personnel is appropriate after making sure the patient can get out of bed. If the patient is obese, the nurse will likely need assistance in getting the patient to the bathroom. Medicating the patient before checking the orders is not advised in this situation. Before medicating for pain, the nurse needs to perform a pain assessment. Offering the patient a walker is a premature intervention until the orders are verified.

The nurse is caring for a patient who is immobile. The nurse frequently checks the patient for impaired skin integrity. What is the rationale for the nurse's action? a. Inadequate blood flow leads to decreased tissue ischemia. b. Patients with limited caloric intake develop thicker skin. c. Pressure reduces circulation to affected tissue. d. Verbalization of skin care needs is decreased.

C Body parts exposed to pressure have reduced circulation to affected tissue. Patients with limited caloric and protein intake develop thinner, less elastic skin with loss of subcutaneous tissue. Inadequate blood flow causes ischemia and breakdown. Verbalization is affected when altered cognition occurs from dementia, psychological disorders, or temporary delirium, not from immobility.

A patient with heart failure has a new order for captopril (Capoten) 12.5 mg PO. After administering the first dose and teaching the patient about the drug, which statement by the patient indicates that teaching has been effective? a. I will be sure to take the medication with food. b. I will need to eat more potassium-rich foods in my diet. c. I will call for help when I need to get up to use the bathroom. d. I will expect to feel more short of breath for the next few days.

C Captopril can cause hypotension, especially after the initial dose, so it is important that the patient not get up out of bed without assistance until the nurse has had a chance to evaluate the effect of the first dose. The angiotensin-converting enzyme (ACE) inhibitors are potassium sparing, and the nurse should not teach the patient to purposely increase sources of dietary potassium. Increased shortness of breath is expected with the initiation of b-adrenergic blocker therapy for heart failure, not for ACE inhibitor therapy. ACE inhibitors are best absorbed when taken an hour before eating.

Vital signs for a patient reveal a high blood pressure of 187/100. Orders state to notify the health care provider for diastolic blood pressure greater than 90. What is the nurse's first action? a. Follow the clinical protocol for a stroke. b. Review the most recent lab results for the patient's potassium level. c. Assess the patient for other symptoms or problems, and then notify the health care provider. d. Administer an antihypertensive medication from the stock supply, and then notify the health care provider.

C Communication to other health care professionals must be timely, accurate, and relevant to a patient's clinical situation. The best answer is to reassess the patient for other symptoms or problems, and then notify the health care provider according to the orders. Reviewing the potassium level does not address the problem of high blood pressure. The nurse does not follow the protocol since the order says to notify the health care provider. The orders read to notify the health care provider, not administer medications.

A nurse attends a workshop on current nursing issues provided by the American Nurses Association. Which type of education did the nurse receive? a. Graduate education b. Inservice education c. Continuing education d. Registered nurse education

C Continuing education involves formal, organized educational programs offered by universities, hospitals, state nurses associations, professional nursing organizations, and educational and health care institutions. After obtaining a baccalaureate degree in nursing, you can pursue graduate education leading to a master's or doctoral degree in any number of graduate fields, including nursing. Inservice education programs are instruction or training provided by a health care facility or institution. Registered nurse education is the education preparation for an individual intending to be an RN.

The nurse instructs a patient about application of corticosteroid cream to an area of contact dermatitis on the right leg. Which patient action indicates that further teaching is needed? a. The patient takes a tepid bath before applying the cream. b. The patient spreads the cream using a downward motion. c. The patient applies a thick layer of the cream to the affected skin. d. The patient covers the area with a dressing after applying the cream.

C Creams and ointments should be applied in a thin layer to avoid wasting the medication. The other actions by the patient indicate that the teaching has been successful.

A dark-skinned patient has been admitted to the hospital with chronic heart failure. How would the nurse best assess this patient for cyanosis? a. Assess the skin color of the earlobes. b. Apply pressure to the palms of the hands. c. Check the lips and oral mucous membranes. d. Examine capillary refill time of the nail beds.

C Cyanosis in dark-skinned individuals is more easily seen in the mucous membranes. Earlobe color may change in light-skinned individuals, but this change in skin color is difficult to detect on darker skin. Application of pressure to the palms of the hands and nail bed assessment would check for adequate circulation but not for skin color.

A patient with left-sided heart failure is having difficulty breathing. Which of the following is the most appropriate nursing intervention? A. Encourage the patient to cough and deep breathe. B. Place the patient in Semi-Fowler's position. C. Assist the patient into High Fowler's position. D. Perform chest percussion therapy.

C Due to the patient being in fluid overload (especially with left-sided heart failure...remember the lungs are majorly affected in this type of heart failure), it is most appropriate to place the patient in High Fowler's position to help make breathing easier.

The nurse considers several new female patients to receive additional teaching on the need for more frequent Pap test and gynecological examinations. Which assessment findings reveal the patient at highest risk for cervical cancer and having the greatest need for patient education? a. 13 years old, nonsmoker, not sexually active b. 15 years old, social smoker, celibate c. 22 years old, smokes 1 pack of cigarettes per day, has multiple sexual partners d. 50 years old, stopped smoking 30 years ago, has history of multiple pregnancies

C Females considered to be at higher risk include those who smoke, have multiple sex partners, and have a history of sexually transmitted infections. Of all the assessment findings listed, the 22-year- old smoker with multiple sexual partners has the greatest number of risk factors for cervical cancer. The other patients are at lower risk: not sexually active, celibate, and do not smoke

The nurse prepares to obtain a culture from a patient who has a possible fungal infection on the foot. Which items should the nurse gather for this procedure? a. Sterile gloves b. Patch test instruments c. Cotton-tipped applicators d. Local anesthetic, syringe, and intradermal needle

C Fungal cultures are obtained by swabbing the affected area of the skin with cotton-tipped applicators. Sterile gloves are not needed because it is not a sterile procedure. Local injection is not needed because the swabbing is not usually painful. The patch test is done to determine whether a patient is allergic to specific testing material, not for obtaining fungal specimens.

An older patient has been diagnosed with possible white coat hypertension. Which action will the nurse plan to take next? a. Schedule the patient for regular blood pressure (BP) checks in the clinic. b. Instruct the patient about the need to decrease stress levels. c. Tell the patient how to self-monitor and record BPs at home. d. Inform the patient that ambulatory blood pressure monitoring will be needed.

C Having the patient self-monitor BPs at home will provide a reliable indication about whether the patient has hypertension. Regular BP checks in the clinic are likely to be high in a patient with white coat hypertension. Ambulatory blood pressure monitoring may be used if the data from self-monitoring are unclear. Although elevated stress levels may contribute to hypertension, instructing the patient about this is unlikely to reduce BP.

The nurse is teaching the parents of a child who has head lice (pediculosis capitis). Which information will the nurse include in the teaching session? a. Treatment is use of regular shampoo. b. Products containing lindane are most effective. c. Head lice may spread to furniture and other people. d. Manual removal is not a realistic option as treatment.

C Head lice are difficult to remove and spread to furniture and other people if not treated. Caution against use of products containing lindane because the ingredient is toxic and is known to cause adverse reactions. Treatments use medicated shampoo for eliminating lice. Manual removal is the best option when treatment has failed.

During a sexually transmitted illness presentation to high-school students, the nurse recommends the human papillomavirus (HPV) vaccine series. Which condition is the nurse trying to prevent? a. Breast cancer b. Ovarian cancer c. Cervical cancer d. Testicular cancer

C Human papillomavirus (HPV) infection increases the person's risk for cervical cancer. HPV vaccine is recommended for females aged 11 to 12 years but can be given to females ages 12 through 26; males can also receive the vaccine. HPV is not a risk factor for breast, ovarian, and testicular cancer.

A patient has recently started on digoxin (Lanoxin) in addition to furosemide (Lasix) and captopril (Capoten) for the management of heart failure. Which assessment finding by the home health nurse is a priority to communicate to the health care provider? a. Presence of 1 to 2+ edema in the feet and ankles b. Palpable liver edge 2 cm below the ribs on the right side c. Serum potassium level 3.0 mEq/L after 1 week of therapy d. Weight increase from 120 pounds to 122 pounds over 3 days

C Hypokalemia can predispose the patient to life-threatening dysrhythmias (e.g., premature ventricular contractions), and potentiate the actions of digoxin and increase the risk for digoxin toxicity, which can also cause life-threatening dysrhythmias. The other data indicate that the patients heart failure requires more effective therapies, but they do not require nursing action as rapidly as the low serum potassium level.

The female nurse is caring for a male patient who is uncircumcised but not ambulatory and has full function of all extremities. The nurse is providing the patient with a partial bed bath. How should perineal care be performed for this patient? a. Should be postponed because it may cause embarrassment b. Should be unnecessary because the patient is uncircumcised c. Should be done by the patient d. Should be done by the nurse

C If a patient is able to perform perineal self-care, encourage this independence. Patients most in need of perineal care are those at greatest risk for acquiring an infection such as uncircumcised males; perineal care is necessary. Embarrassment should not cause the nurse to overlook the patient's hygiene needs. The nurse should provide this care only if the patient is unable to do so.

A nurse is providing nursing care to patients after completing a care plan from nursing diagnoses. In which step of the nursing process is the nurse? a. Assessment b. Planning c. Implementation d. Evaluation

C Implementation, the fourth step of the nursing process, formally begins after a nurse develops a plan of care. With a care plan based on clear and relevant nursing diagnoses, a nurse initiates interventions that are designed to assist the patient in achieving the goals and expected outcomes needed to support or improve the patient's health status. The nurse gathers data during the assessment phase and mutually sets goals and prioritizes care during the planning phase. During the evaluation phase, the nurse determines the achievement of goals and effectiveness of interventions.

The nurse prescribes strategies and alternatives to reach an expected outcome. Which standard of nursing practice is the nurse following? a. Assessment b. Diagnosis c. Planning d. Implementation

C In planning, the registered nurse develops a plan that prescribes strategies and alternatives to attain expected outcomes. During assessment, the registered nurse collects comprehensive data pertinent to the patient's health and/or the situation. In diagnosis, the registered nurse analyzes the assessment data to determine the diagnoses or issues. During implementation, the registered nurse implements (carries out) the identified plan.

The nurse is making a preoperative education appointment with a patient. The patient asks if a family member should come to the appointment. Which is the best response by the nurse? a. "There is no need for an additional person at the appointment." b. "Your family can come and wait with you in the waiting room." c. "We recommend including family members at this appointment." d. "It is required that you have a family member at this appointment."

C Including family members in perioperative education is advisable. Often a family member is a coach for postoperative exercises when the patient returns from surgery. If anxious relatives do not understand routine postoperative events, it is likely that their anxiety will heighten the patient's fears and concerns. Preoperative preparation of family members before surgery helps to minimize anxiety and misunderstanding. An additional person is needed at the appointment if at all possible, and he or she needs to be involved in the process, not just waiting in the waiting room; however, it is certainly not a requirement for actually completing the surgery that someone comes to this appointment.

The registered nurse (RN) is caring for a patient with a hypertensive crisis who is receiving sodium nitroprusside (Nipride). Which nursing action can the nurse delegate to an experienced licensed practical/vocational nurse (LPN/LVN)? a. Titrate nitroprusside to decrease mean arterial pressure (MAP) to 115 mm Hg. b. Evaluate effectiveness of nitroprusside therapy on blood pressure (BP). c. Set up the automatic blood pressure machine to take BP every 15 minutes. d. Assess the patients environment for adverse stimuli that might increase BP.

C LPN/LVN education and scope of practice include the correct use of common equipment such as automatic blood pressure machines. The other actions require advanced nursing judgment and education, and should be done by RNs.

Which action should the nurse take when administering the initial dose of oral labetalol (Normodyne) to a patient with hypertension? a. Encourage the use of hard candy to prevent dry mouth. b. Instruct the patient to ask for help if heart palpitations occur. c. Ask the patient to request assistance when getting out of bed. d. Teach the patient that headaches may occur with this medication.

C Labetalol decreases sympathetic nervous system activity by blocking both - and b-adrenergic receptors, leading to vasodilation and a decrease in heart rate, which can cause severe orthostatic hypotension. Heart palpitations, dry mouth, dehydration, and headaches are possible side effects of other antihypertensives.

A patient with heart failure is taking Losartan and Spironolactone. The patient is having EKG changes that presents with tall peaked T-waves and flat p-waves. Which of the following lab results confirms these findings? A. Na+ 135 B. BNP 560 C. K+ 8.0 D. K+ 1.5

C Losartan and Spironolactone can both cause an increased potassium level (hyperkalemia). Losartan is an ARB and Spironolactone is a potassium-sparing diuretic. Therefore, the EKG changes are a sign of a high potassium level (normal potassium level is 3.5-5.1).

The nurse plans discharge teaching for a patient with chronic heart failure who has prescriptions for digoxin (Lanoxin) and hydrochlorothiazide (HydroDIURIL). Appropriate instructions for the patient include a. limit dietary sources of potassium. b. take the hydrochlorothiazide before bedtime. c. notify the health care provider if nausea develops. d. skip the digoxin if the pulse is below 60 beats/minute.

C Nausea is an indication of digoxin toxicity and should be reported so that the provider can assess the patient for toxicity and adjust the digoxin dose, if necessary. The patient will need to include potassium-containing foods in the diet to avoid hypokalemia. Patients should be taught to check their pulse daily before taking the digoxin and if the pulse is less than 60, to call their provider before taking the digoxin. Diuretics should be taken early in the day to avoid sleep disruption.

The nurse is caring for a patient the first postoperative day following a laparotomy for a small bowel obstruction. The nurse notices new bright-red drainage about 5 cm in diameter on the dressing. Which action should the nurse take first? a. Reinforce the dressing. b. Apply an abdominal binder. c. Take the patients vital signs. d. Recheck the dressing in 1 hour for increased drainage.

C New bright-red drainage may indicate hemorrhage, and the nurse should initially assess the patients vital signs for tachycardia and hypotension. The surgeon should then be notified of the drainage and the vital signs. The dressing may be changed or reinforced, based on the surgeons orders or institutional policy. The nurse should not wait an hour to recheck the dressing.

The nurse inserts an intravenous (IV) catheter using the correct technique and following the recommended steps according to standards of care and hospital policy. Which type of implementation skill is the nurse using? a. Cognitive b. Interpersonal c. Psychomotor d. Judgmental

C Nursing practice includes cognitive, interpersonal, and psychomotor skills. Psychomotor skill requires the integration of cognitive and motor abilities. The nurse in this example displayed the psychomotor skill of inserting an intravenous catheter while following standards of care and integrating knowledge of anatomy and physiology. Cognitive involves the application of critical thinking and use of good judgment in making sound clinical decisions. Interpersonal skills involve developing trusting relationships with patients, conveying caring and compassion, and communicating clearly.

A postoperative patient has a nursing diagnosis of ineffective airway clearance. The nurse determines that interventions for this nursing diagnosis have been successful if which is observed? a. Patient drinks 2 to 3 L of fluid in 24 hours. b. Patient uses the spirometer 10 times every hour. c. Patients breath sounds are clear to auscultation. d. Patients temperature is less than 100.4 F orally.

C One characteristic of ineffective airway clearance is the presence of adventitious breath sounds such as rhonchi or crackles, so clear breath sounds are an indication of resolution of the problem. Spirometer use and increased fluid intake are interventions for ineffective airway clearance but may not improve breath sounds in all patients. Elevated temperature may occur with atelectasis, but a normal or near-normal temperature does not always indicate resolution of respiratory problems.

The nurse is providing oral care to a patient. In which order will the nurse clean the oral cavity, starting with the first area? 1. Roof of mouth, gums, and inside cheek 2. Chewing and inner tooth surfaces 3. Outer tooth surfaces 4. Tongue a. 4, 1, 3, 2 b. 3, 2, 4, 1 c. 2, 3, 1, 4 d. 1, 4, 2, 3

C Oral care is provided in the following sequence: Clean chewing and inner tooth surfaces first. Clean outer tooth surfaces. Moisten brush with chlorhexidine rinse to rinse. Use toothette to clean roof of mouth, gums, and inside cheeks. Gently brush tongue but avoid stimulating gag reflex. Rinse.

During a school physical examination, the nurse reviews the patient's current medical history. The nurse discovers the patient has allergies. Which assessment finding is consistent with allergies? a. Clubbing b. Yellow discharge c. Pale nasal mucosa d. Puffiness of nasal mucosa

C Pale nasal mucosa with clear discharge indicates allergy. Clubbing is due to insufficient oxygenation at the periphery resulting from conditions such as chronic emphysema and congenital heart disease; it is noted in the nails. A sinus infection results in yellowish or greenish discharge. Habitual use of intranasal cocaine and opioids causes puffiness and increased vascularity of the nasal mucosa.

A patient who has chronic heart failure tells the nurse, I was fine when I went to bed, but I woke up in the middle of the night feeling like I was suffocating! The nurse will document this assessment finding as a. orthopnea. b. pulsus alternans. c. paroxysmal nocturnal dyspnea. d. acute bilateral pleural effusion.

C Paroxysmal nocturnal dyspnea is caused by the reabsorption of fluid from dependent body areas when the patient is sleeping and is characterized by waking up suddenly with the feeling of suffocation. Pulsus alternans is the alternation of strong and weak peripheral pulses during palpation. Orthopnea indicates that the patient is unable to lie flat because of dyspnea. Pleural effusions develop over a longer time period.

An advanced practice nurse is preparing to assess the external genitalia of a 25-year-old American woman of Chinese descent. Which action will the nurse do first? a. Place the patient in the lithotomy position. b. Drape the patient to enhance patient comfort. c. Assess the patient's feelings about the examination. d. Ask the patient if she would like her mother to be present in the room.

C Patients who are Chinese American often believe that examination of the external genitalia is offensive. Before proceeding with the examination, the nurse first determines how the patient feels about the procedure and explains the procedure to answer any questions and to help the patient feel comfortable with the assessment. Once the patient is ready to have her external genitalia examined, the nurse places the patient in the lithotomy position and drapes the patient appropriately. Typically, nurses ask adolescents if they want a parent present during the examination. The patient in this question is 25 years old; asking if she would like her mother to be present is inappropriate.

The nurse is caring for a surgical patient, when the family member asks what perioperative nursing means. How should the nurse respond? a. Perioperative nursing occurs in preadmission testing. b. Perioperative nursing occurs primarily in the postanesthesia care unit. c. Perioperative nursing includes activities before, during, and after surgery. d. Perioperative nursing includes activities only during the surgical procedure.

C Perioperative nursing care occurs before, during, and after surgery. Preadmission testing occurs before surgery and is considered preoperative. Nursing care provided during the surgical procedure is considered intraoperative, and in the postanesthesia care unit, it is considered postoperative. All of these are parts of the perioperative phase, but each individual phase does not explain the term completely.

After performing foot care, the nurse checks the medical record and discovers that the patient has a foot disorder caused by a virus. Which condition did the nurse most likely observe? a. Corns b. A callus c. Plantar warts d. Athlete's foot

C Plantar warts appear on the sole of the foot and are caused by the papillomavirus. Corns are caused by friction and pressure from ill-fitting or loose shoes. Athlete's foot (tinea pedis) is a fungal infection and can spread to other body parts. A callus is caused by local friction or pressure

Which topic is most important for the nurse to discuss preoperatively with a patient who is scheduled for abdominal surgery for an open cholecystectomy? a. Care for the surgical incision b. Medications used during surgery c. Deep breathing and coughing techniques d. Oral antibiotic therapy after discharge home

C Preoperative teaching, demonstration, and redemonstration of deep breathing and coughing are needed on patients having abdominal surgery to prevent postoperative atelectasis. Incisional care and the importance of completing antibiotics are better discussed after surgery, when the patient will be more likely to retain this information. The patient does not usually need information about medications that are used intraoperatively.

A nurse is reviewing a patient's care plan. Which information will the nurse identify as a nursing intervention? a. The patient will ambulate in the hallway twice this shift using crutches correctly. b. Impaired physical mobility related to inability to bear weight on right leg. c. Provide assistance while the patient walks in the hallway twice this shift with crutches. d. The patient is unable to bear weight on right lower extremity.

C Providing assistance to a patient who is ambulating is a nursing intervention. The statement, "The patient will ambulate in the hallway twice this shift using crutches correctly" is a patient outcome. Impaired physical mobility is a nursing diagnosis. The statement that the patient is unable to bear weight and ambulate can be included with assessment data and is a defining characteristic for the diagnosis of Impaired physical mobility

A confused patient with a urinary catheter, nasogastric tube, and intravenous line keeps touching these needed items for care. The nurse has tried to explain to the patient that these lines should not be touched, but the patient continues. Which is the best action by the nurse at this time? a. Apply restraints loosely on the patient's dominant wrist. b. Notify the health care provider that restraints are needed immediately. c. Try other approaches to prevent the patient from touching these care items. d. Allow the patient to pull out lines to prove that the patient needs to be restrained.

C Restraints can be used when less restrictive interventions are not successful. The nurse must try other approaches than just telling. The situation states that the patient is touching the items, not trying to pull them out. At this time, the patient's well-being is not at risk so restraints cannot be used at this time nor does the health care provider need to be notified. Allowing the patient to pull out any of these items to prove the patient needs to be restrained is not acceptable.

The nurse is providing perineal care to an uncircumcised male patient. Which action will the nurse take? a. Leave the foreskin alone because there is little chance of infection. b. Retract the foreskin for cleansing and allow it to return on its own. c. Retract the foreskin and return it to its natural position when done. d. Leave the foreskin retracted.

C Return the foreskin to its natural position. Keeping the foreskin retracted leads to tightening of the foreskin around the shaft of the penis, causing local edema and discomfort. The foreskin may not return to its natural position on its own. Patients at greatest risk for infection are uncircumcised males.

A patient undergoing an emergency appendectomy has been using St. Johns wort to prevent depression. Which complication would the nurse expect in the post-anesthesia care unit? a. Increased pain b. Hypertensive episodes c. Longer time to recover from anesthesia d. Increased risk for postoperative bleeding

C St. Johns wort may prolong the effects of anesthetic agents and increase the time to waken completely after surgery. It is not associated with increased bleeding risk, hypertension, or increased pain

A nurse is preparing to perform a complete physical examination on a weak, older-adult patient with bilateral basilar pneumonia. Which position will the nurse use? a. Prone b. Sims' c. Supine d. Lateral recumbent

C Supine is the most normally relaxed position. If the patient becomes short of breath easily, raise the head of the bed. Supine position would be easiest for a weak, older-adult person during the examination. Lateral recumbent and prone positions cause respiratory difficulty for any patient with respiratory difficulties. Sims' position is used for assessment of the rectum and the vagina

The nurse is caring for a patient who is reporting severe foot pain due to corns. The patient has been using oval corn pads to self-treat the corns, but they seem to be getting worse. Which information will the nurse share with the patient? a. Corn pads are an adequate treatment and should be continued. b. The patient should avoid soaking the feet before using a pumice stone. c. Depending on severity, surgery may be needed to remove the corns. d. Tighter shoes would help to compress the corns and make them smaller.

C Surgical removal is necessary, depending on severity of pain and the size of the corn. Oval corn pads should be avoided because they increase pressure on the toes and reduce circulation. Warm water soaks soften corns before gentle rubbing with a callus file or pumice stone. Wider and softer shoes, especially shoes with a wider toe box, are helpful.

The nurse working on the heart failure unit knows that teaching an older female patient with newly diagnosed heart failure is effective when the patient states that a. she will take furosemide (Lasix) every day at bedtime. b. the nitroglycerin patch is applied when any chest pain develops. c. she will call the clinic if her weight goes from 124 to 128 pounds in a week. d. an additional pillow can help her sleep if she is feeling short of breath at night.

C Teaching for a patient with heart failure includes information about the need to weigh daily and notify the health care provider about an increase of 3 pounds in 2 days or 3 to 5 pounds in a week. Nitroglycerin patches are used primarily to reduce preload (not to prevent chest pain) in patients with heart failure and should be used daily, not on an as needed basis. Diuretics should be taken earlier in the day to avoid nocturia and sleep disturbance. The patient should call the clinic if increased orthopnea develops, rather than just compensating by further elevating the head of the bed.

A patient with chronic heart failure who is taking a diuretic and an angiotensin-converting enzyme (ACE) inhibitor and who is on a low-sodium diet tells the home health nurse about a 5-pound weight gain in the last 3 days. The nurses priority action will be to a. have the patient recall the dietary intake for the last 3 days. b. ask the patient about the use of the prescribed medications. c. assess the patient for clinical manifestations of acute heart failure. d. teach the patient about the importance of restricting dietary sodium.

C The 5-pound weight gain over 3 days indicates that the patients chronic heart failure may be worsening. It is important that the patient be assessed immediately for other clinical manifestations of decompensation, such as lung crackles. A dietary recall to detect hidden sodium in the diet, reinforcement of sodium restrictions, and assessment of medication compliance may be appropriate interventions but are not the first nursing actions indicated.

The nurse notes the presence of white lesions that resemble milk curds in the back of a patients throat. Which question by the nurse is appropriate at this time? a. Do you have a productive cough? b. How often do you brush your teeth? c. Are you taking any medications at present? d. Have you ever had an oral herpes infection?

C The appearance of the lesions is consistent with an oral candidiasis (thrush) infection, which can occur in patients who are taking medications such as immunosuppressants or antibiotics. Candidiasis is not associated with poor oral hygiene or lower respiratory infections. The lesions do not look like an oral herpes infection.

A nurse is auscultating different areas on an adult patient. Which technique should the nurse use during an assessment? a. Uses the bell to listen for lung sounds b. Uses the diaphragm to listen for bruits c. Uses the diaphragm to listen for bowel sounds d. Uses the bell to listen for high-pitched murmurs

C The bell is best for hearing low-pitched sounds such as vascular (bruits) and certain heart sounds (low-pitched murmurs), and the diaphragm is best for listening to high-pitched sounds such as bowel and lung sounds and high-pitched murmurs.

Which topic will the nurse plan to include in discharge teaching for a patient with systolic heart failure and an ejection fraction of 33%? a. Need to begin an aerobic exercise program several times weekly b. Use of salt substitutes to replace table salt when cooking and at the table c. Benefits and side effects of angiotensin-converting enzyme (ACE) inhibitors d. Importance of making an annual appointment with the primary care provider

C The core measures for the treatment of heart failure established by The Joint Commission indicate that patients with an ejection fraction (EF) <40% receive an ACE inhibitor to decrease the progression of heart failure. Aerobic exercise may not be appropriate for a patient with this level of heart failure, salt substitutes are not usually recommended because of the risk of hyperkalemia, and the patient will need to see the primary care provider more frequently than annually.

While admitting an 82-year-old with acute decompensated heart failure to the hospital, the nurse learns that the patient lives alone and sometimes confuses the water pill with the heart pill. When planning for the patients discharge the nurse will facilitate a a. consult with a psychologist. b. transfer to a long-term care facility. c. referral to a home health care agency. d. arrangements for around-the-clock care.

C The data about the patient suggest that assistance in developing a system for taking medications correctly at home is needed. A home health nurse will assess the patients home situation and help the patient develop a method for taking the two medications as directed. There is no evidence that the patient requires services such as a psychologist consult, long-term care, or around-the-clock home care.

A patient is undergoing psoralen plus ultraviolet A light (PUVA) therapy for treatment of psoriasis. What action should the nurse take to prevent adverse effects from this procedure? a. Cleanse the skin carefully with an antiseptic soap. b. Shield any unaffected areas with lead-lined drapes. c. Have the patient use protective eyewear while receiving PUVA. d. Apply petroleum jelly to the areas surrounding the psoriatic lesions.

C The eyes should be shielded from UV light (UVL) during and after PUVA therapy to prevent the development of cataracts. The patient should be taught about the effects of UVL on unaffected skin, but lead-lined drapes, use of antiseptic soap, and petroleum jelly are not used to prevent skin damage.

While assessing a 68-year-old with ascites, the nurse also notes jugular venous distention (JVD) with the head of the patients bed elevated 45 degrees. The nurse knows this finding indicates a. decreased fluid volume. b. jugular vein atherosclerosis. c. increased right atrial pressure. d. incompetent jugular vein valves.

C The jugular veins empty into the superior vena cava and then into the right atrium, so JVD with the patient sitting at a 45-degree angle reflects increased right atrial pressure. JVD is an indicator of excessive fluid volume (increased preload), not decreased fluid volume. JVD is not caused by incompetent jugular vein valves or atherosclerosis.

A nurse is making initial rounds on patients. Which intervention for a patient with poor wound healing should the nurse perform first? a. Reinforce the wound dressing as needed with 4 × 4 gauze. b. Perform the ordered dressing change twice daily. c. Observe wound appearance and edges. d. Document wound characteristics.

C The most appropriate initial intervention is to assess the wound (observe wound appearance and edges). The nurse must assess the wound first before the findings can be documented, reinforcement of the dressing, and the actual skill of dressing changes.

During change-of-shift report, the nurse obtains the following information about a hypertensive patient who received the first dose of nadolol (Corgard) during the previous shift. Which information indicates that the patient needs immediate intervention? a. The patients most recent blood pressure (BP) reading is 158/91 mm Hg. b. The patients pulse has dropped from 68 to 57 beats/minute. c. The patient has developed wheezes throughout the lung fields. d. The patient complains that the fingers and toes feel quite cold.

C The most urgent concern for this patient is the wheezes, which indicate that bronchospasm (a common adverse effect of the noncardioselective b-blockers) is occurring. The nurse should immediately obtain an oxygen saturation measurement, apply supplemental oxygen, and notify the health care provider. The mild decrease in heart rate and complaint of cold fingers and toes are associated with b-receptor blockade but do not require any change in therapy. The BP reading may indicate that a change in medication type or dose may be indicated. However, this is not as urgently needed as addressing the bronchospasm.

The patient has been diagnosed with diabetes. When admitted, the patient is unkempt and is in need of a bath and foot care. When questioned about hygiene habits, the nurse learns the patient takes a bath once a week and a sponge bath every other day. To provide ultimate care for this patient, which principle should the nurse keep in mind? a. Patients who appear unkempt place little importance on hygiene practices. b. Personal preferences determine hygiene practices and are unchangeable. c. The patient's illness may require teaching of new hygiene practices. d. All cultures value cleanliness with the same degree of importance.

C The nurse must assist the patient in developing new hygiene practices when indicated by an illness or condition. For example, the nurse will need to teach a patient with diabetes proper foot hygiene. Patients who appear unkempt often need further assessment regarding their ability to participate in daily hygiene. Patients with certain types of physical limitations or disabilities often lack the physical energy and dexterity to perform hygienic care. Culturally, maintaining cleanliness does not hold the same importance for some ethnic groups as it does for others.

The nurse plans to provide preoperative teaching to an alert older man who has hearing and vision deficits. His wife usually answers most questions that are directed to the patient. Which action should the nurse take when doing the teaching? a. Use printed materials for instruction so that the patient will have more time to review the material. b. Direct the teaching toward the wife because she is the obvious support and caregiver for the patient. c. Provide additional time for the patient to understand preoperative instructions and carry out procedures. d. Ask the patients wife to wait in the hall in order to focus preoperative teaching with the patient himself.

C The nurse should allow more time when doing preoperative teaching and preparation for older patients with sensory deficits. Because the patient has visual deficits, he will not be able to use written material for learning. The teaching should be directed toward both the patient and the wife because both will need to understand preoperative procedures and teaching

When examining an older patient in the home, the home health nurse notices irregular patterns of bruising at different stages of healing on the patients body. Which action should the nurse take first? a. Discourage the use of throw rugs throughout the house. b. Ensure the patient has a pair of shoes with non-slip soles. c. Talk with the patient alone and ask about what caused the bruising. d. Notify the health care provider so that x-rays can be ordered as soon as possible.

C The nurse should note irregular patterns of bruising, especially in the shapes of hands or fingers, in different stages of resolution. These may be indications of other health problems or abuse, and should be further investigated. It is important that the nurse interview the patient alone because, if mistreatment is occurring, the patient may not disclose it in the presence of the person who may be the abuser. Throw rugs and shoes with slippery surfaces may contribute to falls. X-rays may be needed if the patient has fallen recently and also has complaints of pain or decreased mobility. However, the nurses first nursing action is to further assess the patient.

When assessing a new patient at the outpatient clinic, the nurse notes dry, scaly skin; thin hair; and thick, brittle nails. What is the nurses best action? a. Instruct the patient about the importance of nutrition in skin health. b. Make a referral to a podiatrist so that the nails can be safely trimmed. c. Consult with the health care provider about the need for further diagnostic testing. d. Teach the patient about using moisturizing creams and lotions to decrease dry skin.

C The patient has clinical manifestations that could be caused by systemic problems such as malnutrition or hypothyroidism, so further diagnostic evaluation is indicated. Patient teaching about nutrition, addressing the patients dry skin, and referral to a podiatrist may also be needed, but the priority is to rule out underlying disease that may be causing these manifestations.

A patient is taking Digoxin. Prior to administration you check the patient's apical pulse and find it to be 61 bpm. Morning lab values are the following: K+ 3.3 and Digoxin level of 5 ng/mL. Which of the following is the correct nursing action? A. Hold this dose and administer the second dose at 1800. B. Administer the dose as ordered. C. Hold the dose and notify the physician of the Digoxin level. D. Hold this dose until the patient's potassium level is normal.

C The patient is Digoxin toxic. A normal Digoxin level is <2 ng/mL. Therefore, the nurse should not administer the dose but hold it and notify the doctor for further orders.

A patient taking Digoxin is experiencing severe bradycardia, nausea, and vomiting. A lab draw shows that their Digoxin level is 4 ng/mL. What medication do you anticipate the physician to order for this patient? A. Narcan B. Aminophylline C. Digibind D. No medication because this is a normal Digoxin level.

C The patient is experiencing Digoxin toxicity...therefore the physician will order the antidote for Digoxin which is Digibind.

An experienced nurse orients a new nurse to the postanesthesia care unit (PACU). Which action by the new nurse, if observed by the experienced nurse, indicates that the orientation was successful? a. The new nurse assists a nauseated patient to a supine position. b. The new nurse positions an unconscious patient supine with the head elevated. c. The new nurse turns an unconscious patient to the side upon arrival in the PACU. d. The new nurse places a patient in the Trendelenburg position when the blood pressure drops.

C The patient should initially be positioned in the lateral recovery position to keep the airway open and avoid aspiration. The Trendelenburg position is avoided because it increases the work of breathing. The patient is placed supine with the head elevated after regaining consciousness.

An outpatient who has chronic heart failure returns to the clinic after 2 weeks of therapy with metoprolol (Toprol XL). Which assessment finding is most important for the nurse to report to the health care provider? a. 2+ pedal edema b. Heart rate of 56 beats/minute c. Blood pressure (BP) of 88/42 mm Hg d. Complaints of fatigue

C The patients BP indicates that the dose of metoprolol may need to be decreased because of hypotension. Bradycardia is a frequent adverse effect of b-adrenergic blockade, but the rate of 56 is not unusual with - adrenergic blocker therapy. b-Adrenergic blockade initially will worsen symptoms of heart failure in many patients, and patients should be taught that some increase in symptoms, such as fatigue and edema, is expected during the initiation of therapy with this class of drugs.

The nurse is caring for a patient with diabetes. Which task will the nurse assign to the nursing assistive personnel? a. Providing nail care b. Teaching foot care c. Making an occupied bed d. Determining aspiration risk

C The skill of making an occupied bed can be delegated to nursing assistive personnel. Nail care, teaching foot care, and assessing aspiration risk of a patient with diabetes must be performed by the RN; these skills cannot be delegated.

The patient has had a stroke that has affected the ability to speak. The patient becomes extremely frustrated when trying to speak. The patient responds correctly to questions and instructions but cannot form words coherently. Which type of aphasia is the patient experiencing? a. Sensory b. Receptive c. Expressive d. Combination

C The two types of aphasias are sensory (or receptive) and motor (or expressive). The patient cannot form words coherently, indicating expressive or motor aphasia is present. The patient responds correctly to questions and instructions, indicating receptive or sensory aphasia is not present. Patients sometimes suffer a combination of receptive and expressive aphasia, but this is not the case here.

On admission, a patient weighs 250 pounds. The weight is recorded as 256 pounds on the second inpatient day. Which condition will the nurse assess for in this patient? a. Anorexia b. Weight loss c. Fluid retention d. Increased nutritional intake

C This patient has gained 6 pounds in a 24-hour period. A weight gain of 5 pounds (2.3 kg) or more in a day indicates fluid retention problems, not nutritional intake. A weight loss is considered significant if the patient has lost more than 5% of body weight in a month or 10% in 6 months. A downward trend may indicate a reduction in nutritional reserves that may be caused by decreased intake such as anorexia.

A patient with a history of chronic heart failure is admitted to the emergency department (ED) with severe dyspnea and a dry, hacking cough. Which action should the nurse do first? a. Auscultate the abdomen. b. Check the capillary refill. c. Auscultate the breath sounds. d. Assess the level of orientation.

C This patients severe dyspnea and cough indicate that acute decompensated heart failure (ADHF) is occurring. ADHF usually manifests as pulmonary edema, which should be detected and treated immediately to prevent ongoing hypoxemia and cardiac/respiratory arrest. The other assessments will provide useful data about the patients volume status and also should be accomplished rapidly, but detection (and treatment) of pulmonary complications is the priority.

The nurse is assessing skin turgor. Which technique will the nurse use? a. Press lightly on the forearm. b. Press lightly on the fingertips. c. Grasp a fold of skin on the sternal area. d. Grasp a fold of skin on the back of the hand.

C To assess skin turgor, grasp a fold of skin on the back of the forearm or sternal area with the fingertips and release. Since the skin on the back of the hand is normally loose and thin, turgor is not reliably assessed at that site. Pressing lightly on the forearm can be used to assess for pitting edema or pain or sense of touch. Pressing lightly on the fingertips and observing nail color is assessing capillary refill.

The nurse is assessing the tympanic membranes of an infant. Which action by the nurse demonstrates proper technique? a. Pulls the auricle upward and backward. b. Holds handle of the otoscope between the thumb and little finger. c. Uses an inverted otoscope grip while pulling the auricle downward and back. d. Places the handle of the otoscope between the thumb and index finger while pulling the auricle upward.

C Using the inverted otoscope grip while pulling the auricle downward and back is a common approach with infant/child examinations because it prevents accidental movement of the otoscope deeper into the ear canal, as could occur with an unexpected pediatric reaction to the ear examination. The other techniques could result in injury to the infant's tympanic membrane. Insert the scope while pulling the auricle upward and backward in the adult and older child. Hold the handle of the otoscope in the space between the thumb and index finger, supported on the middle finger.

A nurse agrees with regulations for mandatory immunizations of children. The nurse believes that immunizations prevent diseases as well as prevent spread of the disease to others. Which ethical framework is the nurse using? a. Deontology b. Ethics of care c. Utilitarianism d. Feminist ethics

C Utilitarianism is a system of ethics that believes that value is determined by usefulness. This system of ethics focuses on the outcome of the greatest good for the greatest number of people. Deontology would not look to consequences of actions but on the "right-making characteristic" such as fidelity and justice. The ethics of care emphasizes the role of feelings. Relationships, which are an important component of feminist ethics, are not addressed in this case.

Which information will the nurse include when teaching an older patient about skin care? a. Dry the skin thoroughly before applying lotions. b. Bathe and wash hair daily with soap and shampoo. c. Use warm water and a moisturizing soap when bathing. d. Use antibacterial soaps when bathing to avoid infection.

C Warm water and moisturizing soap will avoid overdrying the skin. Because older patients have dryer skin, daily bathing and shampooing are not necessary and may dry the skin unnecessarily. Antibacterial soaps are not necessary. Lotions should be applied while the skin is still damp to seal moisture in.

The patient presents to the clinic with dysuria and hematuria. How does the nurse proceed to assess for kidney inflammation? a. Uses deep palpation posteriorly. b. Lightly palpates each abdominal quadrant. c. Percusses posteriorly the costovertebral angle at the scapular line. d. Inspects abdomen for abnormal movement or shadows using indirect lighting.

C With the patient sitting or standing erect, use direct or indirect percussion to assess for kidney inflammation. With the ulnar surface of the partially closed fist, percuss posteriorly the costovertebral angle at the scapular line. If the kidneys are inflamed, the patient feels tenderness during percussion. Use a systematic palpation approach for each quadrant of the abdomen to assess for muscular resistance, distention, abdominal tenderness, and superficial organs or masses. Light palpation would not detect kidney tenderness because the kidneys sit deep within the abdominal cavity. Posteriorly, the lower ribs and heavy back muscles protect the kidneys, so they cannot be palpated. Kidney inflammation will not cause abdominal movement. However, to inspect the abdomen for abnormal movement or shadows, the nurse should stand on the patient's right side and inspect from above the abdomen using direct light over the abdomen.

The nurse is caring for a dying patient. Which intervention is considered futile? a. Giving pain medication for pain b. Providing oral care every 5 hours c. Administering the influenza vaccine d. Supporting lower extremities with pillows

C Administering the influenza vaccine is futile. A vaccine is administered to prevent or lessen the likelihood of contracting an infectious disease at some time in the future. The term futile refers to something that is hopeless or serves no useful purpose. In health care discussions the term refers to interventions unlikely to produce benefit for a patient. Care delivered to a patient at the end of life that is focused on pain management, oral hygiene, and comfort measures is not futile.

An obstetric nurse comes across an automobile accident. The driver seems to have a crushed upper airway, and while waiting for emergency medical services to arrive, the nurse makes a cut in the trachea and inserts a straw from a purse to provide an airway. The patient survives and has a permanent problem with vocal cords, making it difficult to talk. Which statement is true regarding the nurse's performance? a. The nurse acted appropriately and saved the patient's life. b. The nurse stayed within the guidelines of the Good Samaritan Law. c. The nurse took actions beyond those that are standard and appropriate. d. The nurse should have just stayed with the patient and waited for help.

C An obstetric nurse would not have been trained in performing a tracheostomy (cut in the trachea), and doing so would be beyond what the nurse has been trained or educated to do. If you perform a procedure exceeding your scope of practice and for which you have no training, you are liable for injury that may result from that act. You should only provide care that is consistent with your level of expertise. The nurse did not act appropriately. The nurse is not protected by the Good Samaritan Law because the nurse acted outside the scope of practice and training. The nurse should have acted within what was trained and educated to do in this circumstance, not just stay with the patient.

A patient is scheduled for surgery. When getting ready to obtain the informed consent, the patient tells the nurse, "I have no idea what is going to happen. I couldn't ask any questions." The nurse does not allow the patient to sign the permit and notifies the health care provider of the situation. Which role is the nurse displaying? a. Manager b. Patient educator c. Patient advocate d. Clinical nurse specialist

C As a patient advocate, the nurse protects the patient's human and legal rights, including the right of the patient to understand procedures before signing permits. Although nurses can be educators, it is the responsibility of the surgeon to provide education for the patient in preparation for surgery, and it is the nurse's responsibility to notify the health care provider if the patient is not properly educated. Managers coordinate the activities of members of the nursing staff in delivering nursing care, and clinical nurse specialists are experts in a specialized area of nursing practice in a variety of settings

A nurse is providing perineal care to a female patient. Which washing technique will the nurse use? a. Back to front b. In a circular motion c. From pubic area to rectum d. Upward from rectum to pubic area

C Cleansing from pubic area to rectum (front to back) reduces the transfer of microorganisms to the urinary meatus and decreases the risk of urinary tract infection. Cleansing from rectum to pubic area or back to front increases the risk of urinary tract infection. Circular motions are used in male perineal care

A bill has been submitted to the State House of Representatives that is designed to reduce the cost of health care by increasing the patient-to-nurse ratio from a maximum of 2:1 in intensive care units to 3:1. What should the nurse realize? a. Legislation is politics beyond the nurse's control. b. National programs have no bearing on state politics. c. The individual nurse can influence legislative decisions. d. Focusing on nursing care provides the best patient benefit.

C Nurses can influence policy decisions at all governmental levels. One way is to get involved by participating in local and national efforts. This effort is critical in exerting nurses' influence early in the political process. Legislation is not beyond the nurse's control. National program can have bearing on state politics. The question is focusing on legislation and health care costs, not nursing care.

A nurse identifies gaps between local and best practices. Which Quality and Safety Education for Nurses (QSEN) competency is the nurse demonstrating? a. Safety b. Patient-centered care c. Quality improvement d. Teamwork and collaboration

C Quality improvement identifies gaps between local and best practices. Safety minimizes risk of harm to patients and providers through both system effectiveness and individual performance. Patient- centered care recognizes the patient or designee as the source of control and full partner in providing compassionate and coordinated care based on respect for patient's preferences, values, and needs. Teamwork and collaboration allows effective functioning within nursing and interprofessional teams, fostering open communication, mutual respect, and shared decision making

Which instruction will the nurse provide to the nursing assistive personnel when providing foot care for a patient with diabetes? a. Do not place slippers on the patient's feet. b. Trim the patient's toenails daily. c. Report sores on the patient's toes. d. Check the brachial artery.

C Report any changes that may indicate inflammation or injury to tissue. Do not allow the diabetic patient to go barefoot; injury can lead to amputations. Clipping toenails is not allowed. Patients with peripheral vascular disease or diabetes mellitus often require nail care from a specialist to reduce the risk of infection. When assessing the patient's feet, the nurse palpates the dorsalis pedis of the foot, not the brachial artery

The patient's son requests to view documentation in the medical record. What is the nurse's best response to this request? a. "I'll be happy to get that for you." b. "You are not allowed to look at it." c. "You will need your mother's permission." d. "I cannot let you see the chart without a doctor's order."

C The mother's permission is needed. The nurse understands that sharing health information is governed by HIPAA legislation, which defines rights and privileges of patients for protection of privacy. Private health information cannot be shared without the patient's specific permission. The nurse cannot obtain the records without permission. The son can look at it after approval from the patient. While talking to the physician or getting an order is appropriate, the patient still has to give consent.

A nurse is discussing quality of life issues with another colleague. Which topic will the nurse acknowledge for increased attention paid to quality of life concerns? a. Health care disparities b. Aging of the population c. Abilities of disabled persons d. Health care financial reform

C The population of disabled persons in the United States and elsewhere has reshaped the discussion about quality of life (QOL). Health care disparities, an aging population, and health care reform are components impacted by personal definitions of quality but are not the underlying reason why QOL discussions have arisen.

The patient reports to the nurse about a perceived decrease in hearing. When the nurse examines the patient's ear, a large amount of cerumen buildup at the entrance to the ear canal is observed. Which action will the nurse take next? a. Teach the patient how to use cotton-tipped applicators. b. Tell the patient to use a bobby pin to extract earwax. c. Apply gentle, downward retraction of the ear canal. d. Instill hot water into the ear canal to melt the wax.

C When cerumen is visible, gentle, downward retraction at the entrance to the ear canal causes the wax to loosen and slip out. Instruct the patient never to use sharp objects such as bobby pins or paper clips to remove earwax. Use of such objects can traumatize the ear canal and ruptures the tympanic membrane. Avoid the use of cotton-tipped applicators as well because they cause earwax to become impacted within the canal. Instilling cold or hot water causes nausea or vomiting.

You're making the patient assignments for the next shift. On your unit there are three LPNs, two RNs, and two nursing assistants. Which patients will you assign to the LPNs? (Select all that apply.) A. A 68 year-old male patient who is expected to be discharged home with IV antibiotic therapy. B. A 25 year-old female patient newly admitted with diabetic ketoacidosis. C. A 75 year-old male patient with dementia who has an ileostomy and scheduled tube feedings. D. A 65 year-old female patient who has an order to remove a Foley catheter.

C, D Options C and D are best for the LPNs: these are standard routine procedures the LPN can perform and these patient cases are stable. Option A: An RN is the best for this patient because the patient will need discharge teaching AND the nurse will need to teach the patient how to self-administer antibiotics. Option B: This is a new admission and the patient is UNSTABLE. Most patients with DKA (diabetic ketoacidosis) require insulin drips along with close monitoring of the blood glucose levels, which requires critical thinking and interpretation.

You're making the patient assignments for the next shift. On your unit there are three LPNs, two RNs, and two nursing assistants. Which patients will you assign to the LPNs? (Select all that apply.) A. A 68 year-old male patient who is expected to be discharged home with IV antibiotic therapy. B. A 25 year-old female patient newly admitted with diabetic ketoacidosis. C. A 75 year-old male patient with dementia who has an ileostomy and scheduled tube feedings. D. A 65 year-old female patient who has a order to remove the Foley catheter.

C, D Options C and D are best for the LPNs: these are standard routine procedures the LPN can perform and these patient cases are stable. Option A: An RN is the best for this patient because the patient will need discharge teaching AND the nurse will need to teach the patient how to self-administer antibiotics. Option B: This is a new admission and the patient is UNSTABLE. Most patients with DKA (diabetic ketoacidosis) require insulin drips along with close monitoring of the blood glucose levels, which requires critical thinking and interpretation.

A nurse is implementing interventions for a group of patients. Which actions are nursing interventions? (Select all that apply.) a. Order chest x-ray for suspected arm fracture. b. Prescribe antibiotics for a wound infection. c. Reposition a patient who is on bed rest. d. Teach a patient preoperative exercises. e. Transfer a patient to another hospital unit.

C, D, E A nursing intervention is any treatment based on clinical judgment and knowledge that a nurse performs to enhance patient outcomes. Repositioning, teaching, and transferring a patient are examples of nursing interventions. Ordering a chest x-ray and prescribing antibiotics are examples of medical interventions performed by a health care provider.

A nurse wants to become an advanced practice registered nurse. Which options should the nurse consider? (Select all that apply.) a. Patient advocate b. Nurse administrator c. Certified nurse-midwife d. Clinical nurse specialist e. Certified nurse practitioner

C, D, E Although all nurses should function as patient advocates, "advanced practice nurse" is an umbrella term for an advanced clinical nurse such as a certified nurse practitioner, clinical nurse specialist, certified registered nurse anesthetist, or certified nurse-midwife. A nurse administrator is not an example of advanced practice.

When caring for a preoperative patient on the day of surgery, which actions included in the plan of care can the nurse delegate to unlicensed assistive personnel (UAP)? (Select all that apply.) a. Teach incentive spirometer use b. Explain preoperative routine care. c. Obtain and document baseline vital signs. d. Remove nail polish and apply pulse oximeter. e. Transport the patient by stretcher to the operating room.

C, D, E Obtaining vital signs, removing nail polish, pulse oximeter placement, and transport of the patient are routine skills that are appropriate to delegate. Teaching patients about the preoperative routine and incentive spirometer use require critical thinking and should be done by the registered nurse.

The patient must stay in bed for a bed change. Which actions will the nurse implement? (Select all that apply.) a. Apply sterile gloves. b. Keep soiled linen close to uniform. c. Advise patient will feel a lump when rolling over. d. Turn clean pillowcase inside out over the hand holding it. e. Make a modified mitered corner with sheet, blanket, and spread.

C, D, E When making an occupied bed, advise patients they will feel a lump when turning, turn clean pillowcase inside out, and make a modified mitered corner. Clean gloves are used. Keep soiled linen away from uniform.

Select all the task you could delegate to a nursing assistant as the RN. (Select all that apply.) A. Wound dressing change B. IV flush C. Collecting vital signs D. Weighing a patient E. Mouth care F. Suctioning a patient G. Applying oxygen to a patient H. Connecting a patient to their IV fluids I. Assisting a patient with a bath J. Applying denture paste to dentures

C, D, E, I, J. These are all tasks the RN could delegate to a nursing assistant.

Select all the task you could delegate to a nursing assistant as the RN: A. Wound dressing change B. IV flush C. Collecting vital signs D. Weighing a patient E. Mouth care F. Suctioning a patient G. Applying oxygen to a patient H. Connecting a patient to their IV fluids I. Assisting a patient with a bath J. Applying denture paste to dentures

C, D, E, I, J. These are all tasks the RN could delegate to a nursing assistant.

You are completing the history on a patient who is scheduled to have surgery. What health history increases the risk for surgery for the patient? A. Urinary Tract infections B. History of Premature Ventricle Beats C. Abuse of street drugs D. Hyperthyroidism

C. If a patient has a history of street drug abuse this puts them at risk in surgery. This information is very important for the anesthesiologist due to the complications that can arise from the anesthesia. All of the other options are important to note but not a risk for surgery.

A patient reports he hasn't had a bowel movement or passed gas since surgery. On assessment, you note the abdomen is distended and no bowel sounds are noted in the four quadrants. You notify the MD. What non-invasive nursing interventions can you perform without a MD order? A. Insert a nasogastric attached to intermittent suction B. Administer IV fluids C. Encourage ambulation, maintain NPO status, and monitor intake & output D. Encourage at least 3000 ml of fluids per day

C. This patient is most likely experiencing a paralytic ileus which is failure for the bowels to move its contents. The only correct non-invasive option is to encourage ambulation, maintain NPO status, and monitor intake & output. Inserting a NG tube or administering IV fluids is invasive and requires a MD order. Patients with potential paralytic ileus are to be NPO (nothing by mouth) so encouraging fluid intake is incorrect.

A patient is now in the recovery room after having vaginal surgery. Due to the positioning of the procedure, you would want to assess for what while the patient is in recovery? A. Bowel Sounds B. Dysrhythmia C. Homan's Sign D. Hemoglobin Level

C. Vaginal surgeries require the patient to be in the lithotomy position. This position can put the patient at risk for a deep vein thrombosis. Therefore, the nurse would want to check for this by using Homan's Sign.

A nurse is completing an assessment of the patient. Which principle is a priority? a. Foot care will always be important. b. Daily bathing will always be important. c. Hygiene needs will always be important. d. Critical thinking will always be important.

D A patient's condition is always changing, requiring ongoing critical thinking and changing of nursing diagnoses. Apply the elements of critical thinking as you use the nursing process to meet patients' hygiene needs. Critical thinking will help you determine when foot care, daily bathing, and hygiene needs are important and when they are not.

A 56-year-old patient who has no previous history of hypertension or other health problems suddenly develops a blood pressure (BP) of 198/110 mm Hg. After reconfirming the BP, it is appropriate for the nurse to tell the patient that a. a BP recheck should be scheduled in a few weeks. b. dietary sodium and fat content should be decreased. c. there is an immediate danger of a stroke and hospitalization will be required. d. diagnosis of a possible cause, treatment, and ongoing monitoring will be needed.

D A sudden increase in BP in a patient over age 50 with no previous hypertension history or risk factors indicates that the hypertension may be secondary to some other problem. The BP will need treatment and ongoing monitoring. If the patient has no other risk factors, a stroke in the immediate future is unlikely. There is no indication that dietary salt or fat intake have contributed to this sudden increase in BP, and reducing intake of salt and fat alone will not be adequate to reduce this BP to an acceptable level.

Following an acute myocardial infarction, a previously healthy 63-year-old develops clinical manifestations of heart failure. The nurse anticipates discharge teaching will include information about a. digitalis preparations. b. b-adrenergic blockers. c. calcium channel blockers. d. angiotensin-converting enzyme (ACE) inhibitors.

D ACE inhibitor therapy is currently recommended to prevent the development of heart failure in patients who have had a myocardial infarction and as a first-line therapy for patients with chronic heart failure. Digoxin therapy for heart failure is no longer considered a first-line measure, and digoxin is added to the treatment protocol when therapy with other medications such as ACE-inhibitors, diuretics, and b-adrenergic blockers is insufficient. Calcium channel blockers are not generally used in the treatment of heart failure. The b- adrenergic blockers are not used as initial therapy for new onset heart failure.

A patient who has just been admitted with pulmonary edema is scheduled to receive the following medications. Which medication should the nurse question before giving? a. Furosemide (Lasix) 60 mg b. Captopril (Capoten) 25 mg c. Digoxin (Lanoxin) 0.125 mg d. Carvedilol (Coreg) 3.125 mg

D Although carvedilol (Coreg) is appropriate for the treatment of chronic heart failure, it is not used for patients with acute decompensated heart failure (ADHF) because of the risk of worsening the heart failure. The other medications are appropriate for the patient with ADHF.

The nurse is caring for a patient in the postanesthesia care unit. The patient has developed profuse bleeding from the surgical site, and the surgeon has determined the need to return to the operative area. How will the nurse classify this procedure? a. Major b. Urgent c. Elective d. Emergency

D An emergency procedure must be done immediately to save a life or preserve the function of a body part. An example would be repair of a perforated appendix, repair of a traumatic amputation, or control of internal hemorrhaging. An urgent procedure is necessary for a patient's health and often prevents additional problems from developing. An example would be excision of a cancerous tumor, removal of a gallbladder for stones, or vascular repair for an obstructed artery. An elective procedure is performed on the basis of the patient's choice; it is not essential and is not always necessary for health. An example would be a bunionectomy, plastic surgery, or hernia reconstruction. A major procedure involves extensive reconstruction or alteration in body parts; it poses great risks to well- being. An example would be a coronary artery bypass or colon resection.

A patient has received atropine before surgery and complains of dry mouth. Which action by the nurse is best? a. Check for skin tenting. b. Notify the health care provider. c. Ask the patient about any dizziness. d. Tell the patient dry mouth is an expected side effect.

D Anticholinergic medications decrease oral secretions, so the patient is taught that a dry mouth is an expected side effect. The dry mouth is not a symptom of dehydration in this case. Therefore there is no immediate need to check for skin tenting. The health care provider does not need to be notified about an expected side effect. Weakness, forgetfulness, and dizziness are side effects associated with other preoperative medications such as opioids and benzodiazepines.

An older adult patient with a squamous cell carcinoma (SCC) on the lower arm has a Mohs procedure in the dermatology clinic. Which nursing action will be included in the postoperative plan of care? a. Describe the use of topical fluorouracil on the incision. b. Teach how to use sterile technique to clean the suture line. c. Schedule daily appointments for wet-to-dry dressing changes. d. Teach about the use of cold packs to reduce bruising and swelling.

D Application of cold packs to the incision after the surgery will help decrease bruising and swelling at the site. Since the Mohs procedure results in complete excision of the lesion, topical fluorouracil is not needed after surgery. After the Mohs procedure the edges of the wound can be left open to heal or the edges can be approximated and sutured together. The suture line can be cleaned with tap water. No debridement with wet- to-dry dressings is indicated.

Which action should the nurse take first during the initial phase of implementation? a. Determine patient outcomes and goals. b. Prioritize patient's nursing diagnoses. c. Evaluate interventions. d. Reassess the patient.

D Assessment is a continuous process that occurs each time the nurse interacts with a patient. During the initial phase of implementation, reassess the patient. Determining the patient's goals and prioritizing diagnoses take place in the planning phase before choosing interventions. Evaluation is the last step of the nursing process.

The nurse values autonomy above all other principles. Which patient assignment will the nurse find most difficult to accept? a. Older-adult patient who requires dialysis b. Teenager in labor who requests epidural anesthesia c. Middle-aged father of three with an advance directive declining life support d. Family elder who is making the decisions for a young-adult female member

D Autonomy refers to freedom from external control. A person who values autonomy highly may find it difficult to accept situations where the patient is not the primary decision maker regarding his or her care. A teenager requesting an epidural, a father with an advance directive, and an elderly patient requiring dialysis all describe a patient or family who can make their own decisions and choices regarding care.

A nurse develops a teaching plan for a patient diagnosed with basal cell carcinoma (BCC). Which information should the nurse include in the teaching plan? a. Treatment plans include watchful waiting. b. Screening for metastasis will be important. c. Low dose systemic chemotherapy is used to treat BCC. d. Minimizing sun exposure will reduce risk for future BCC.

D BCC is frequently associated with sun exposure and preventive measures should be taken for future sun exposure. BCC spreads locally, and does not metastasize to distant tissues. Since BCC can cause local tissue destruction, treatment is indicated. Local (not systemic) chemotherapy may be used to treat BCC.

Having misplaced a stethoscope, a nurse borrows a colleague's stethoscope. The nurse next enters the patient's room and identifies self, washes hands with soap, and states the purpose of the visit. The nurse performs proper identification of the patient before auscultating the patient's lungs. Which critical health assessment step should the nurse have performed? a. Running warm water over stethoscope b. Draping stethoscope around the neck c. Rubbing stethoscope with betadine d. Cleaning stethoscope with alcohol

D Bacteria and viruses can be transferred from patient to patient when a stethoscope that is not clean is used. The stethoscope should be cleaned before use on each patient with isopropyl alcohol. Running water over the stethoscope does not kill bacteria. Betadine is an inappropriate cleaning solution and may damage the equipment. Draping the stethoscope around the neck is not advised.

A patient has the following risk factors for melanoma. Which risk factor should the nurse assign as the priority focus of patient teaching? a. The patient has multiple dysplastic nevi. b. The patient is fair-skinned and has blue eyes. c. The patients mother died of a malignant melanoma. d. The patient uses a tanning booth throughout the winter.

D Because the only risk factor that the patient can change is the use of a tanning booth, the nurse should focus teaching about melanoma prevention on this factor. The other factors also will contribute to increased risk for melanoma.

The nurse is caring for a patient who is receiving IV furosemide (Lasix) and morphine for the treatment of acute decompensated heart failure (ADHF) with severe orthopnea. Which clinical finding is the best indicator that the treatment has been effective? a. Weight loss of 2 pounds in 24 hours b. Hourly urine output greater than 60 mL c. Reduction in patient complaints of chest pain d. Reduced dyspnea with the head of bed at 30 degrees

D Because the patients major clinical manifestation of ADHF is orthopnea (caused by the presence of fluid in the alveoli), the best indicator that the medications are effective is a decrease in dyspnea with the head of the bed at 30 degrees. The other assessment data also may indicate that diuresis or improvement in cardiac output has occurred, but are not as specific to evaluating this patients response.

The nurse is caring for a 70-year-old who uses hydrochlorothiazide (HydroDIURIL) and enalapril (Norvasc), but whose self-monitored blood pressure (BP) continues to be elevated. Which patient information may indicate a need for a change? a. Patient takes a daily multivitamin tablet. b. Patient checks BP daily just after getting up. c. Patient drinks wine three to four times a week. d. Patient uses ibuprofen (Motrin) daily to treat osteoarthritis.

D Because use of nonsteroidal antiinflammatory drugs (NSAIDs) can prevent adequate BP control, the patient may need to avoid the use of ibuprofen. A multivitamin tablet will help supply vitamin D, which may help lower BP. BP decreases while sleeping, so self-monitoring early in the morning will result in obtaining pressures that are at their lowest. The patients alcohol intake is not excessive

A patient is diagnosed with left-sided systolic dysfunction heart failure. Which of the following are expected findings with this condition? A. Echocardiogram shows an ejection fraction of 38%. B. Heart catheterization shows an ejection fraction of 65%. C. Patient has frequent episodes of nocturnal paroxysmal dyspnea. D. Options A and C are both expected findings with left sided systolic dysfunction heart failure.

D Both Options A and C are correct. Option B is a finding expected in left-sided DIASTOLIC dysfunction heart failure because the issue is with the ability of the ventricle to FILL properly...therefore a patient usually has a normal ejection fraction. Remember a normal EF is >60% in a healthy heart.

The nurse obtains a health history from a patient who is scheduled for elective hip surgery in 1 week. The patient reports use of garlic and ginkgo biloba. Which action by the nurse is most appropriate? a. Ascertain that there will be no interactions with anesthetic agents. b. Teach the patient that these products may be continued preoperatively. c. Advise the patient to stop the use of all herbs and supplements at this time. d. Discuss the herb and supplement use with the patients health care provider.

D Both garlic and ginkgo biloba increase a patients risk for bleeding. The nurse should discuss the herb and supplement use with the patients health care provider. The nurse should not advise the patient to stop the supplements or to continue them without consulting with the health care provider. Determining the interactions between the supplements and anesthetics is not within the nurses scope of practice.

What is the best method to prevent the spread of infection when the nurse is changing the dressing over a wound infected with Staphylococcus aureus? a. Change the dressing using sterile gloves. b. Soak the dressing in sterile normal saline. c. Apply antibiotic ointment over the wound. d. Wash hands and properly dispose of soiled dressings.

D Careful hand washing and the safe disposal of soiled dressings are the best means of preventing the spread of skin problems. Sterile glove and sterile saline use during wound care will not necessarily prevent spread of infection. Applying antibiotic ointment will treat the bacteria but not necessarily prevent the spread of infection.

The nurse is caring for a patient diagnosed with furunculosis. Which nursing action could the nurse delegate to unlicensed assistive personnel (UAP)? a. Applying antibiotic cream to the groin. b. Obtaining cultures from ruptured lesions. c. Evaluating the patients personal hygiene. d. Cleaning the skin with antimicrobial soap.

D Cleaning the skin is within the education and scope of practice for UAP. Administration of medication, obtaining cultures, and evaluation are higher-level skills that require the education and scope of practice of licensed nursing personnel.

The nurse is encouraging a reluctant postoperative patient to deep breathe and cough. Which explanation can the nurse provide that may encourage the patient to comply? a. "If you don't deep breathe and cough, you will get pneumonia." b. "You will need to cough only a few times during this shift." c. "Let's try clearing the throat because that will work just as well." d. "Deep breathing and coughing will clear out the anesthesia."

D Deep breathing and coughing expel retained anesthetic gases and facilitate a patient's return to consciousness. Although it is correct that a patient may experience atelectasis and pneumonia if deep breathing and coughing are not performed, the way this is worded sounds threatening and could be communicated in a more therapeutic manner. Deep breathing and coughing are encouraged every 2 hours while the patient is awake. Just clearing the throat does not remove mucus from deeper airways

Which action by the nurse will be the most important for preventing skin impairment in a mobile patient with local nerve damage? a. Insert an indwelling urinary catheter. b. Limit caloric and protein intake. c. Turn the patient every 2 hours. d. Assess for pain during a bath.

D During a bath, assess the status of sensory nerve function by checking for touch, pain, heat, cold, and pressure. When restricted from moving freely, dependent body parts are exposed to pressure that reduces circulation. However, this patient is mobile and therefore is able to change positions. Limiting caloric and protein intake may result in impaired or delayed wound healing. A mobile patient can use bathroom facilities or a urinal and does not need a urinary catheter.

The nurse is providing education about the importance of proper foot care to a patient who has diabetes mellitus. Which primary goal is the nurse trying to achieve? a. Prevention of plantar warts b. Prevention of foot fungus c. Prevention of neuropathy d. Prevention of amputation

D Foot ulceration is the most common single precursor to lower extremity amputations among persons with diabetes. Prevention of plantar warts and foot fungus are important but not the primary goal. Neuropathy is a degeneration of the peripheral nerves usually due to poor control of blood glucose levels; it is not a direct result of foot care.

After the nurse teaches the patient with stage 1 hypertension about diet modifications that should be implemented, which diet choice indicates that the teaching has been effective? a. The patient avoids eating nuts or nut butters. b. The patient restricts intake of chicken and fish. c. The patient has two cups of coffee in the morning. d. The patient has a glass of low-fat milk with each meal.

D For the prevention of hypertension, the Dietary Approaches to Stop Hypertension (DASH) recommendations include increasing the intake of calcium-rich foods. Caffeine restriction and decreased protein intake are not included in the recommendations. Nuts are high in beneficial nutrients and 4 to 5 servings weekly are recommended in the DASH diet.

Which action will be included in the plan of care when the nurse is caring for a patient who is receiving nicardipine (Cardene) to treat a hypertensive emergency? a. Keep the patient NPO to prevent aspiration caused by nausea and possible vomiting. b. Organize nursing activities so that the patient has undisturbed sleep for 6 to 8 hours at night. c. Assist the patient up in the chair for meals to avoid complications associated with immobility. d. Use an automated noninvasive blood pressure machine to obtain frequent blood pressure (BP) measurements.

D Frequent monitoring of BP is needed when the patient is receiving rapid-acting IV antihypertensive medications. This can be most easily accomplished with an automated BP machine or arterial line. The patient will require frequent assessments, so allowing 6 to 8 hours of undisturbed sleep is not appropriate. When patients are receiving IV vasodilators, bed rest is maintained to prevent decreased cerebral perfusion and fainting. There is no indication that this patient is nauseated or at risk for aspiration, so an NPO status is unnecessary.

During a routine pediatric history and physical, the parents report that their child was a very small, premature infant that had to stay in the neonatal intensive care unit longer than usual. They state that the infant was yellow when born and developed an infection that required "every antibiotic under the sun" to reach a cure. Which exam is a priority for the nurse to conduct on the child? a. Cardiac b. Respiratory c. Ophthalmic d. Hearing acuity

D Hearing is the priority. Risk factors for hearing problems include low birth weight, nonbacterial intrauterine infection, and excessively high bilirubin levels. Hearing loss due to ototoxicity (injury to auditory nerves) can result from high maintenance doses of antibiotics. Cardiac, respiratory, and eye examinations are important assessments but are not relevant to this child's condition.

Which information should the nurse include when teaching a patient with newly diagnosed hypertension? a. Increasing physical activity will control blood pressure (BP) for most patients. b. Most patients are able to control BP through dietary changes. c. Annual BP checks are needed to monitor treatment effectiveness. d. Hypertension is usually asymptomatic until target organ damage occurs.

D Hypertension is usually asymptomatic until target organ damage has occurred. Lifestyle changes (e.g., physical activity, dietary changes) are used to help manage blood pressure, but drugs are needed for most patients. Home BP monitoring should be taught to the patient and findings checked by the health care provider frequently when starting treatment for hypertension and then every 3 months once stable.

The nurse assesses that the oxygen saturation is 89% in an unconscious patient who was transferred from surgery to the postanesthesia care unit (PACU) 15 minutes ago. Which action should the nurse take first? a. Elevate the patients head. b. Suction the patients mouth. c. Increase the oxygen flow rate. d. Perform the jaw-thrust maneuver.

D In an unconscious postoperative patient, a likely cause of hypoxemia is airway obstruction by the tongue, and the first action is to clear the airway by maneuvers such as the jaw thrust or chin lift. Increasing the oxygen flow rate and suctioning are not helpful when the airway is obstructed by the tongue. Elevating the patients head will not be effective in correcting the obstruction but may help with oxygenation after the patient is awake.

The nurse is urgently called to the gymnasium regarding an injured student. The student is crying in severe pain with a malformed fractured lower leg. Which proper sequence will the nurse follow to perform the initial assessment? a. Light palpation, deep palpation, and inspection b. Inspection, light palpation, and deep palpation c. Auscultation and light palpation d. Inspection and light palpation

D Inspection is the use of vision and hearing to distinguish normal from abnormal findings. Light palpation determines areas of tenderness and skin temperature, moisture, and texture. Deep palpation is used to examine the condition of organs, such as those in the abdomen. Caution is the rule with deep palpation. Deep palpation is performed after light palpation; however, deep palpation is not performed on a fractured leg. Auscultation is used to evaluate sound and is not used to assess a fractured leg.

The nurse and the nursing assistive personnel are assisting a postoperative patient to turn in bed. To assist in minimizing discomfort, which instruction should the nurse provide to the patient? a. "Close your eyes and think about something pleasant." b. "Hold your breath and count to three." c. "Grab my shoulders with your hands." d. "Place your hand over your incision."

D Instruct the patient to place the right hand over the incisional area to splint it, providing support and minimizing pulling during turning. Closing one's eyes, holding one's breath, and holding the nurse's shoulders do not help support the incision during a turn.

As the nurse prepares a patient the morning of surgery, the patient refuses to remove a wedding ring, saying, I have never taken it off since the day I was married. Which response by the nurse is best? a. Have the patient sign a release and leave the ring on. b. Tape the wedding ring securely to the patients finger. c. Tell the patient that the hospital is not liable for loss of the ring. d. Suggest that the patient give the ring to a family member to keep

D Jewelry is not allowed to be worn by the patient, especially if electrocautery will be used. There is no need for a release form or to discuss liability with the patient

The physician's order says to administered Lasix 40 mg IV twice a day. The patient has the following morning labs: Na+ 148, BNP 900, K+ 2.0, and BUN 10. Which of the following is a nursing priority? A. Administer the Lasix as ordered B. Notify the physician of the BNP level C. Assess the patient for edema D. Hold the dose and notify the physician about the potassium level

D Lasix is a diuretic that wastes potassium. A normal potassium level is 3.5-5.1. The nurse should hold the dose and notify the physician who will order a potassium supplement to replace the potassium deficient.

When teaching the patient with newly diagnosed heart failure about a 2000-mg sodium diet, the nurse explains that foods to be restricted include a. canned and frozen fruits. b. fresh or frozen vegetables. c. eggs and other high-protein foods. d. milk, yogurt, and other milk products.

D Milk and yogurt naturally contain a significant amount of sodium, and intake of these should be limited for patients on a diet that limits sodium to 2000 mg daily. Other milk products, such as processed cheeses, have very high levels of sodium and are not appropriate for a 2000-mg sodium diet. The other foods listed have minimal levels of sodium and can be eaten without restriction.

A school nurse recognizes a belt buckle-shaped ecchymosis on a 7-year-old student. When privately asked about how the injury occurred, the student described falling on the playground. Which action will the nurse take next? a. Talk to the principal about how to proceed. b. Disregard the finding based upon child's response. c. Interview the patient in the presence of the teacher. d. Contact social services and report suspected abuse.

D Most states mandate a report to a social service center if nurses suspect abuse or neglect. When abuse is suspected, the nurse interviews the patient in private, not with a teacher. Observe the behavior of the individual for any signs of frustration, explanations that do not fit his or her physical presentation, or signs of injury. The nurse knows how to proceed and does not need to talk to the principal about what to do. Disregarding the finding is not advised because victims often will not complain or report that they are in an abusive situation.

A young woman who is pregnant with a fetus exposed to multiple teratogens consents to have her fetus undergo serial PUBS (percutaneous umbilical blood sampling) to examine how exposure affects the fetus over time. Although these tests will not improve the fetus's outcomes and will expose it to some risks, the information gathered may help infants in the future. Which ethical principle is at greatest risk? a. Fidelity b. Autonomy c. Beneficence d. Nonmaleficence

D Nonmaleficence is the ethical principle that focuses on avoidance of harm or hurt. Repeated PUBS may expose the mother and fetus to some risks. Fidelity refers to the agreement to keep promises (obtain serial PUBS). Autonomy refers to freedom from external control (mother consented), and beneficence refers to taking positive actions to help others (may help infants in the future).

Which of the following tests/procedures are NOT used to diagnose heart failure? A. Echocardiogram B. Brain natriuretic peptide blood test C. Nuclear stress test D. Holter monitoring

D Options A, B, and C are all used to diagnose heart failure...however a holter monitor is not. A holter monitor is used to monitor a patient's heart rate and rhythm.

The nurse assesses a patient who has just arrived in the postanesthesia recovery area (PACU) after a blepharoplasty. Which assessment data should be reported to the surgeon immediately? a. The patient complains of incisional pain. b. The patients heart rate is 110 beats/minute. c. The patient is unable to detect when the eyelids are touched. d. The skin around the incision is pale and cold when palpated.

D Pale, cool skin indicates a possible decrease in circulation, so the surgeon should be notified immediately. The other assessment data indicate a need for ongoing assessment or nursing action. A heart rate of 110 beats/minute may be related to the stress associated with surgery. Assessment of other vital signs and continued monitoring are appropriate. Because local anesthesia would be used for the procedure, numbness of the incisional area is expected immediately after surgery. The nurse should monitor for return of feeling.

You're providing diet discharge teaching to a patient with a history of heart failure. Which of the following statements made by the patient represents they understood the diet teaching? A. "I will limit my sodium intake to 5-6 grams a day." B. "I will be sure to incorporate canned vegetables and fish into my diet." C. "I'm glad I can still eat sandwiches because I love bologna and cheese sandwiches." D. "I will limit my consumption of frozen meals."

D Patients with heart failure should limit sodium intake to 2 to 3 grams per day (not 5-6 grams), avoid canned vegetable/fish, and avoid sandwich meats and cheeses because of their high sodium content. Frozen meals are high in sodium, therefore the patient is correct in saying they should limit their consumption of them.

Which abnormality on the skin of an older patient is the priority to discuss immediately with the health care provider? a. Several dry, scaly patches on the face b. Numerous varicosities noted on both legs c. Dilation of small blood vessels on the face d. Petechiae present on the chest and abdomen

D Petechiae are caused by pinpoint hemorrhages and are associated with a variety of serious disorders such as meningitis and coagulopathies. The nurse should contact the patients health care provider about this finding for further diagnostic follow-up. The other skin changes are associated with aging. Although the other changes will also require ongoing monitoring or intervention by the nurse, they do not indicate a need for urgent action.

Which of the following is a late sign of heart failure? A. Shortness of breath B. Orthopnea C. Edema D. Frothy-blood tinged sputum

D Shortness of breath, orthopnea, and edema are EARLY signs and symptoms. Frothy-blood tinged sputum is a late sign.

The nurse is caring for a patient supported with a ventilator who has been unresponsive since arrival via ambulance 8 days ago. The patient has not been identified, & no family members have been found. The nurse is concerned about the plan of care regarding maintenance or withdrawal of life support measures. Place the steps the nurse will use to resolve this ethical dilemma in the correct order. 1. The nurse identifies possible solutions or actions to resolve the dilemma. 2. The nurse reviews the medical record, including entries by all health care disciplines, to gather information relevant to this patient's situation. 3. Health care providers use negotiation to redefine the patient's plan of care. 4. The nurse evaluates the plan & revises it with input from other health care providers as necessary. 5. The nurse examines the issue to clarify opinions, values, & facts. 6. The nurse states the problem. a. 6, 1, 2, 5, 4, 3 b. 5, 6, 2, 3, 4, 1 c. 1, 2, 5, 4, 3, 6 d. 2, 5, 6, 1, 3, 4

D Step 1. Gather as much information as possible that is relevant to the case. Step 2. Examine and determine your values about the issues. Step 3. Verbalize the problem. Step 4. Consider possible courses of action. Step 5. Negotiate the outcome. Step 6. Evaluate the action

The outpatient surgery nurse reviews the complete blood cell (CBC) count results for a patient who is scheduled for surgery in a few days. The results are white blood cell (WBC) count 10.2 (103/L); hemoglobin 15 g/dL; hematocrit 45%; platelets 150 (103/L). Which action should the nurse take? a. Call the surgeon and anesthesiologist immediately. b. Ask the patient about any symptoms of a recent infection. c. Discuss the possibility of blood transfusion with the patient. d. Send the patient to the holding area when the operating room calls.

D The CBC count results are normal. With normal results, the patient can go to the holding area when the operating room is ready for the patient. There is no need to notify the surgeon or anesthesiologist, discuss blood transfusion, or ask about recent infection

A newly hired experienced nurse is preparing to change a patient's abdominal dressing and hasn't done it before at this hospital. Which action by the nurse is best? a. Have another nurse do it so the correct method can be viewed. b. Change the dressing using the method taught in nursing school. c. Ask the patient how the dressing change has been recently done. d. Check the policy and procedure manual for the facility's method.

D The Joint Commission requires accredited hospitals to have written nursing policies and procedures. These internal standards of care are specific and need to be accessible on all nursing units. For example, a policy/procedure outlining the steps to follow when changing a dressing or administering medication provides specific information about how nurses are to perform. The nurse being observed may not be doing the procedure according to the facility's policy or procedure. The procedure taught in nursing school may not be consistent with the policy or procedure for this facility. The patient is not responsible for maintaining the standards of practice. Patient input is important, but it's not what directs nursing practice.

A patient has just been diagnosed with hypertension and has been started on captopril (Capoten). Which information is important to include when teaching the patient about this medication? a. Check blood pressure (BP) in both arms before taking the medication. b. Increase fluid intake if dryness of the mouth is a problem. c. Include high-potassium foods such as bananas in the diet. d. Change position slowly to help prevent dizziness and falls.

D The angiotensin-converting enzyme (ACE) inhibitors frequently cause orthostatic hypotension, and patients should be taught to change position slowly to allow the vascular system time to compensate for the position change. Increasing fluid intake may counteract the effect of the medication, and the patient is taught to use gum or hard candy to relieve dry mouth. The BP should be taken in the nondominant arm by newly diagnosed patients in the morning, before taking the medication, and in the evening. Because ACE inhibitors cause potassium retention, increased intake of high-potassium foods is inappropriate.

During assessment of the patients skin, the nurse observes a similar pattern of small, raised lesions on the left and right upper back areas. Which term should the nurse use to document these lesions? a. Confluent b. Zosteriform c. Generalized d. Symmetric

D The description of the lesions indicates that they are grouped. The other terms are inconsistent with the description of the lesions.

A patient with an enlarging, irregular mole that is 7 mm in diameter is scheduled for outpatient treatment. The nurse should plan to prepare the patient for which procedure? a. Curettage b. Cryosurgery c. Punch biopsy d. Surgical excision

D The description of the mole is consistent with malignancy, so excision and biopsy are indicated. Curettage and cryosurgery are not used if malignancy is suspected. A punch biopsy would not be done for a lesion greater than 5 mm in diameter.

The nurse interviews a patient scheduled to undergo general anesthesia for a hernia repair. Which information is most important to communicate to the surgeon and anesthesiologist before surgery? a. The patient drinks 3 or 4 cups of coffee every morning before going to work. b. The patient takes a baby aspirin daily but stopped taking aspirin 10 days ago. c. The patient drank 4 ounces of apple juice 3 hours before coming to the hospital. d. The patients father died after receiving general anesthesia for abdominal surgery.

D The information about the patients father suggests that there may be a family history of malignant hyperthermia and that precautions may need to be taken to prevent this complication. Current research indicates that having clear liquids 3 hours before surgery does not increase the risk for aspiration in most patients. Patients are instructed to discontinue aspirin 1 to 2 weeks before surgery. The patient should be offered caffeinated beverages postoperatively to prevent a caffeine-withdrawal headache, but this does not have preoperative implications

A patient who takes a diuretic and a b-blocker to control blood pressure is scheduled for breast reconstruction surgery. Which patient information is most important to communicate to the health care provider before surgery? a. Hematocrit 36% b. Blood pressure 144/82 c. Pulse rate 58 beats/minute d. Serum potassium 3.2 mEq/L

D The low potassium level may increase the risk for intraoperative complications such as dysrhythmias. Slightly elevated blood pressure is common before surgery because of anxiety. The lower heart rate would be expected in a patient taking a b-blocker. The hematocrit is in the low normal range but does not require any intervention before surgery.

The nurse and the nursing assistive personnel (NAP) are caring for a group of postoperative patients who need turning, coughing, deep breathing, incentive spirometer, and leg exercises. Which task will the nurse assign to the NAP? a. Teach postoperative exercises. b. Do nothing associated with postoperative exercises. c. Document in the medical record when exercises are completed. d. Inform the nurse if the patient is unwilling to perform exercises.

D The nurse can delegate to the NAP to encourage patients to practice postoperative exercises regularly after instruction and to inform the nurse if the patient is unwilling to perform these exercises. The skills of demonstrating and teaching postoperative exercises and documenting are not within the scope of practice for the nursing assistant. Doing nothing is not appropriate.

A nursing assistive personnel (NAP) is providing AM care to patients. Which action by the NAP will require the nurse to intervene? a. Not offering a backrub to a patient with fractured ribs b. Not offering to wash the hair of a patient with neck trauma c. Turning off the television while giving a backrub to the patient d. Turning patient's head with neck injury to side when giving oral care

D The nurse must intervene if the NAP turns the patient's head with a neck injury; this is contraindicated and must be stopped to prevent further injury. All the other actions are appropriate and do not need follow-up. Consult the medical record for any contraindications to a massage (e.g., fractured ribs, burns, and heart surgery). Before washing a patient's hair, determine that there are no contraindications to procedure (e.g., neck injury). When providing a backrub, enhance relaxation by reducing noise (turning off the television) and ensuring that the patient is comfortable.

A patient who has severe refractory psoriasis on the face, neck, and extremities is socially withdrawn because of the appearance of the lesions. Which action should the nurse take first? a. Discuss the possibility of enrolling in a worker-retraining program. b. Encourage the patient to volunteer to work on community projects. c. Suggest that the patient use cosmetics to cover the psoriatic lesions. d. Ask the patient to describe the impact of psoriasis on quality of life.

D The nurses initial actions should be to assess the impact of the disease on the patients life and to allow the patient to verbalize feelings about the psoriasis. Depending on the assessment findings, other actions may be appropriate.

A patient in the emergency department is reporting left lower abdominal pain. Which proper order will the nurse follow to perform the comprehensive abdominal examination? a. Percussion, palpation, auscultation b. Percussion, auscultation, palpation c. Inspection, palpation, auscultation d. Inspection, auscultation, palpation

D The order of an abdominal examination differs slightly from that of other assessments. Begin with inspection and follow with auscultation. By using auscultation before palpation, the chance of altering the frequency and character of bowel sounds is lessened.

A patient with atopic dermatitis has a new prescription for pimecrolimus (Elidel). After teaching the patient about the medication, which statement by the patient indicates that further teaching is needed? a. After I apply the medication, I can go ahead and get dressed as usual. b. I will need to minimize my time in the sun while I am using the Elidel. c. I will rub the medication gently onto the skin every morning and night. d. If the medication burns when I apply it, I will wipe it off and call the doctor.

D The patient should be taught that transient burning at the application site is an expected effect of pimecrolimus and that the medication should be left in place. The other statements by the patient are accurate and indicate that patient teaching has been effective.

A patient recovering from a leg fracture after a fall reports having dull pain in the affected leg and rates it as a 7 on a 0 to 10 scale. The patient is not able to walk around in the room with crutches because of leg discomfort. Which nursing intervention is priority? a. Assist the patient to walk in the room with crutches. b. Obtain a walker for the patient. c. Consult physical therapy. d. Administer pain medication.

D The patient's pain is a 7, indicating the priority is pain relief (administer pain medication). Acute pain is the priority because the nurse can address the problem of immobility after the patient receives adequate pain relief. Assisting the patient to walk or obtaining a walker will not address the pain the patient is experiencing.

The nurse notes darker skin pigmentation in the skinfolds of a middle-aged patient who has a body mass index of 40 kg/m2. What is the nurses best action? a. Teach the patient about the treatment of fungal infection. b. Discuss the use of drying agents to minimize infection risk. c. Instruct the patient about the use of mild soap to clean skinfolds. d. Ask the patient about type 2 diabetes or if there is a family history of it.

D The presence of acanthosis nigricans in skinfolds suggests either having type 2 diabetes or being at an increased risk for it. The description of the patients skin does not indicate problems with fungal infection, poor hygiene, or the need to dry the skinfolds better.

Which information should the nurse include when teaching patients about decreasing the risk for sun damage to the skin? a. Use a sunscreen with an SPF of at least 8 to 10 for adequate protection. b. Water resistant sunscreens will provide good protection when swimming. c. Increase sun exposure by no more than 10 minutes a day to avoid skin damage. d. Try to stay out of the sun between the hours of 10 AM and 2 PM (regular time).

D The risk for skin damage from the sun is highest with exposure between 10 AM and 2 PM. No sunscreen is completely water resistant. Sunscreens classified as water resistant sunscreens still need to be reapplied after swimming. Sunscreen with an SPF of at least 15 is recommended for people at normal risk for skin cancer. Although gradually increasing sun exposure may decrease the risk for burning, the risk for skin cancer is not decreased.

The nurse is caring for a patient in the preoperative holding area of an ambulatory surgery center. Which nursing action will be most appropriate for this area? a. Count the sterile surgical instruments. b. Empty the urinary drainage bag. c. Check the surgical dressing. d. Apply a warm blanket.

D The temperature in the preoperative holding area and in adjacent operating suites is usually cold. Offer the patient an extra warm blanket. Counts are taken by the circulating and scrub nurses in the operating room. Emptying a urinary drainage bag and checking the surgical dressing occur in the postanesthesia care unit, not in the holding area.

These drugs are used as first-line treatment of heart failure. They work by allowing more blood to flow to the heart which decreases the work load of the heart and allows the kidneys to secrete sodium. However, some patients can develop a nagging cough with these types of drugs. This description describes? A. Beta-blockers B. Vasodilators C. Angiotensin II receptor blockers D. Angiotensin-converting-enzyme inhibitors

D This is a description of ACE inhibitors (option D).

A patient is being discharged home after hospitalization of left ventricular systolic dysfunction. As the nurse providing discharge teaching to the patient, which statement is NOT a correct statement about this condition? A. "Signs and symptoms of this type of heart failure can include: dyspnea, persistent cough, difficulty breathing while lying down, and weight gain." B. "It is important to monitor your daily weights, fluid and salt intake." C. "Left-sided heart failure can lead to right-sided heart failure, if left untreated." D. "This type of heart failure can build up pressure in the hepatic veins and cause them to become congested with fluid which leads to peripheral edema."

D This is a description of right-sided heart failure NOT left ventricular systolic dysfunction. Left-sided systolic dysfunction is where the left side of the heart is unable to CONTRACT efficiently which causes blood to back-up into the lungs...leading to pulmonary edema.

What type of heart failure does this statement describe? The ventricle is unable to properly fill with blood because it is too stiff. Therefore, blood backs up into the lungs causing the patient to experience shortness of breath. A. Left ventricular systolic dysfunction B. Left ventricular ride-sided dysfunction C. Right ventricular diastolic dysfunction D. Left ventricular diastolic dysfunction

D This statement describes left ventricular DIASTOLIC dysfunction.

A head and neck physical examination is completed on a 50-year-old female patient. All physical findings are normal except for fine brittle hair. Which laboratory test will the nurse expect to be ordered, based upon the physical findings? a. Oxygen saturation b. Liver function test c. Carbon monoxide d. Thyroid-stimulating hormone test

D Thyroid disease can make hair thin and brittle. Liver function testing is indicated for a patient who has jaundice. Oxygen saturation will be used for cyanosis. Cherry-colored lips indicate carbon monoxide poisoning.

The health care provider prescribes topical 5-FU for a patient with actinic keratosis on the left cheek. The nurse should include which statement in the patients instructions? a. 5-FU will shrink the lesion so that less scarring occurs once the lesion is excised. b. You may develop nausea and anorexia, but good nutrition is important during treatment. c. You will need to avoid crowds because of the risk for infection caused by chemotherapy. d. Your cheek area will be painful and develop eroded areas that will take weeks to heal.

D Topical 5-FU causes an initial reaction of erythema, itching, and erosion that lasts 4 weeks after application of the medication is stopped. The medication is topical, so there are no systemic effects such as increased infection risk, anorexia, or nausea.

Upon assessment, the patient is breathing normally and has normal vesicular lung sounds. Which expected inspiratory-to-expiratory breath sounds will the nurse hear? a. The expiration phase is longer than the inspiration phase. b. The inspiratory phase lasts exactly as long as the expiratory phase. c. The expiration phase is 2 times longer than the inspiration phase. d. The inspiratory phase is 3 times longer than the expiratory phase.

D Vesicular breath sounds are normal breath sounds; the inspiratory phase is 3 times longer than the expiratory phase. Bronchovesicular breath sounds have an inspiratory phase equal to the expiratory phase. Bronchial breath sounds have an expiration phase longer than the inspiration phase at a 3:2 ratio.

A female nursing student in the final term of nursing school is overheard by a nursing faculty member telling another student that she got to insert a nasogastric tube in the emergency department while working as a nursing assistant. Which advice is best for the nursing faculty member to give to the nursing student? a. "Just be careful when you are doing new procedures and make sure you are following directions by the nurse." b. "Review your procedures before you go to work, so you will be prepared to do them if you have a chance." c. "The nurse should not have allowed you to insert the nasogastric tube because something bad could have happened." d. "You are not allowed to perform any procedures other than those in your job description even with the nurse's permission."

D When nursing students work as nursing assistants or nurse's aides when not attending classes, they should not perform tasks that do not appear in a job description for a nurse's aide or assistant. The nursing student should always follow the directions of the nurse, unless doing so violates the institution's guidelines or job description under which the nursing student was hired, such as inserting a nasogastric tube or giving an intramuscular medication. The nursing student should be able to safely complete the procedures delegated as a nursing assistant, and reviewing those not done recently is a good idea, but it has nothing to do with the situation. The focus of the discussion between the nursing faculty member and the nursing student should be on following the job description under which the nursing student is working.

The nurse is providing oral care to an unconscious patient. Which action should the nurse take? a. Moisten the mouth using lemon-glycerin sponges. b. Hold the patient's mouth open with gloved fingers. c. Use foam swabs to help remove plaque. d. Suction the oral cavity.

D When providing oral hygiene to an unconscious patient, the nurse needs to protect the, from choking & aspiration. Have 2 nurses provide care; 1 nurse does the actual cleaning, & the other caregiver removes secretions with suction equipment. The nurse can delegate nursing assistive personnel to participate. Some agencies use equipment that combines a mouth swab with the suction device. This device can be used safely by 1 nurse to provide oral care. Commercially made foam swabs are ineffective in removing plaque. Do not use lemon-glycerin sponges because they dry mucous membranes & erode tooth enamel. While cleansing the oral cavity, use a small oral airway or a padded tongue blade to hold the mouth open. Never use your fingers to hold the patient's mouth open. A human bite contains multiple pathogenic microorganisms.

The nurse assesses a patient on the second postoperative day after abdominal surgery to repair a perforated duodenal ulcer. Which finding is most important for the nurse to report to the surgeon? a. Tympanic temperature 99.2 F (37.3 C) b. Fine crackles audible at both lung bases c. Redness and swelling along the suture line d. 200 mL sanguineous fluid in the wound drain

D Wound drainage should decrease and change in color from sanguineous to serosanguineous by the second postoperative day. The color and amount of drainage for this patient are abnormal and should be reported. Redness and swelling along the suture line and a slightly elevated temperature are normal signs of postoperative inflammation. Atelectasis is common after surgery. The nurse should have the patient cough and deep breathe, but there is no urgent need to notify the surgeon.

During your morning assessment of a patient with heart failure, the patient complains of sudden vision changes that include seeing yellowish-green halos around the lights. Which of the following medications do you suspect is causing this issue? A. Lisinopril B. Losartan C. Lasix D. Digoxin

D Yellowish-green halos/vision changes are classic signs of Digoxin toxicity.

A nurse must make an ethical decision concerning vulnerable patient populations. Which philosophy of health care ethics would be particularly useful for this nurse? a. Teleology b. Deontology c. Utilitarianism d. Feminist ethics

D Feminist ethics particularly focuses on the nature of relationships, especially those where there is a power imbalance or a point of view that is ignored or invisible. Deontology refers to making decisions or "right-making characteristics," bioethics focuses on consensus building, while utilitarianism and teleology speak to the greatest good for the greatest number.

The patient is being treated for cancer with weekly radiation therapy to the head and chemotherapy treatments. Which assessment is the priority? a. Feet b. Nail beds c. Perineum d. Oral cavity

D The oral cavity is the priority. Radiation to the head reduces salivary flow and lowers pH of saliva, leading to stomatitis and tooth decay, while chemotherapy drugs kill the normal cells lining the oral cavity, leading to ulcers and inflammation. While the feet, nail beds, and perineum are important, they are not as affected as the oral cavity with head or neck radiation and chemotherapy.

When taking the health history of an older adult, the nurse discovers that the patient has worked in the landscaping business for 40 years. The nurse will plan to teach the patient about how to self-assess for which clinical manifestations (select all that apply)? a. Vitiligo b. Alopecia c. Intertrigo d. Erythema e. Actinic keratosis

D, E A patient who has worked as a landscaper is at risk for skin lesions caused by sun exposure such as erythema and actinic keratosis. Vitiligo, alopecia, and intertrigo are not associated with excessive sun exposure.

An RN has a critical patient that needs constant monitoring. However, the RN also has other patients in need of care. Which tasks below could the RN delegate to the LPN to help continue the process of patient care? (Select all that apply.) A. Admitting and assessing the new admission B. Completing the discharge teaching to a patient going home C. Updating and evaluating the patient's plan of care D. Administering subcutaneous Heparin E. Obtaining a routine 12-lead EKG F. Collecting a stool specimen G. Flushing a central line with normal saline

D, E, F these are all tasks an LPN can perform. They are routine procedures that usually have predictable outcomes. RNs are responsible for performing assessments on new admissions, teaching, evaluating, flushing and maintain central lines, and updating the patient's plan of care.

The nurse is caring for a group of postoperative patients on the surgical unit. Which patient assessments indicate the nurse needs to follow up? (Select all that apply.) a. Patient with abdominal surgery has patent airway. b. Patient with knee surgery has approximated incision. c. Patient with femoral artery surgery has strong pedal pulse. d. Patient with lung surgery has 20 mL/hr of urine output via catheter. e. Patient with bladder surgery has bloody urine within the first 12 hours. f. Patient with appendix surgery has thready pulse and blood pressure is 90/60.

D, F Thready pulse, low blood pressure, and urine output of 20 mL/hr need to have follow-up by the nurse. Hemorrhage results in a fall in blood pressure; elevated heart and respiratory rates; thready pulse; cool, clammy, pale skin; and restlessness. Notify the surgeon if these changes occur. If the patient has a urinary catheter, there should be a continuous flow of urine of approximately 30 to 50 mL/hr in adults; this patient requires follow-up since the output is 20 mL/hr. All the rest are normal findings. A patent airway, a strong distal pulse, and approximated incision are all normal findings. Surgery involving portions of the urinary tract normally causes bloody urine for at least 12 to 24 hours, depending on the type of surgery.

A patient is scheduled to take Captopril. When is the best time to administer this medication? a. 30 minutes after a meal b. At bedtime c. In the morning d. 1 hour before a meal

D.

A nurse is developing a care plan for a patient who is at risk for developing pneumonia after surgery. Which of the following is not an appropriate nursing intervention? A. Encourage patient intake of 3000 ml/day of fluids if not contraindicated B. Encourage patient to use the incentive spirometer device 10 times every 1-2 hours while awake C. Encourage early ambulation and patient to eat meals in beside chair D. Repositioning every 3-4 hours

D. All options are correct expect for repositioning every 3-4 hours. If the patient is unable to reposition themselves or ambulate, they must be repositioned every 1 to 2 hours minimally.

A patient is taking Aspirin 325 mg PO by mouth daily. The patient is scheduled for surgery in a week. What education do you provide the patient with before surgery? A. Educate the patient to take the scheduled dose of Aspirin the day of surgery to help prevent blood clots B. To hold his morning dose of Aspirin because the nurse will give it to him before surgery C. None of the above are correct D. The medication should be discontinued for 48 hours prior to the scheduled surgery date

D. Aspirin alters the normal clotting factors and increases the patient's chances of hemorrhaging. Therefore, it should be held for at least 48 hours prior to surgery as specified by the surgeon.

After surgery your patient is semicomatose with vital signs within normal limits. As the nurse, what position would be best for this patient? A. Semi-Fowlers B. Prone C. Low-Fowlers D. Side positioning preferably on the left side

D. patients who are semicomatose are at risk for aspiration (due to secretions pooling in the mouth or vomiting which is a common side effect of sedation). Placing the patient onto their side preferably the left will help decrease the risk of aspiration and help promote cardiovascular circulation.

A patient has a history of heart failure. Which of the following statements by the patient indicates the patient may be experiencing heart failure exacerbation? A. "I've noticed that I've gain 6 lbs in one week." B. "While I sleep I have to prop myself up with a pillow so I can breathe." C. "I haven't noticed any swelling in my feet or hands lately." D. Options B and C are correct. E. Options A and B are correct. F. Options A, B, and C are all correct.

E Options A and B are classic signs and symptoms a patient may experience with heart failure exacerbation.

True or False: An RN delegates to the LPN to administer a scheduled tube feeding to a patient. The RN has now transferred full accountability to the LPN for the task getting done, and the RN is no longer accountable for the task.

FALSE: The RN can delegate this task to the LPN BUT the RN is still ACCOUNTABLE for the task getting done even though the RN is not the one performing it.

Abandonment., - is when the Nurse gets angry at patient and nurse leaves the hospital.

False imprisonment., - is when the Nurse immediately applies restraints to make patient stay in bed.

Arterial problems., - is when the Lower extremity is pale and cool with decreased pulse

Jugular vein distention., - is when the Neck vein is visible when sitting

Invasion of privacy., - is when the Nurse posts about patient's loud and unruly family members.

Malpractice., - is when the Nurse leaves bed in high position, causing patient to fall and break hip.

Lordosis.,- is Swayback

Melena., - is Black, tarry stools

True or False: Most patients with hypertension are asymptomatic.

True

True or False: Patients with left-sided diastolic dysfunction heart failure usually have a normal ejection fraction.

True Patient with left-sided DIASTOLIC dysfunction heart failure normally have a normal ejection fraction. However, patients with left-sided SYSTOLIC dysfunction heart failure usually do not because the heart is unable to CONTRACT efficiently rather than fill properly as with diastolic dysfunction.

Koilonychia., - is Spoon nails

Venous problems., - is when the Lower extremity is swollen and warm with normal pulse

A patient is being discharged home on Hydrochlorothiazide (HCTZ) for treatment of hypertension. Which of the following statements by the patient indicates they understood your discharge teaching about this medication? a. I will make sure I consume foods high in potassium. b. I will only take this medication if my blood pressure is high. c. I understand a dry cough is a common side effect with this medication. d. I will monitor my glucose levels closely because this medication may mask symptoms of hypoglycemia.

a

Which of the following systems of the body are affected by hypertension? a. Cardiovascular, brain, kidney, eyes b. Cardiovascular, gastrointestinal, reproductive, and kidney c. Brain, respiratory, kidney, cardiovascular d. None of the options are correct

a

Which of the following drugs is NOT considered an Angiotension Receptor Blocker (ARBs) medication used in hypertension? a. Catapres b. Losartan c. Benicar d. Valsartan

a.

When performing a skin assessment, the nurse notes several angiomas on the chest of an older patient. Which action should the nurse take next? a. Assess the patient for evidence of liver disease. b. Discuss the adverse effects of sun exposure on the skin. c. Teach the patient about possible skin changes with aging. d. Suggest that the patient make an appointment with a dermatologist.

a. Angiomas are a common occurrence as patients get older, but they may occur with systemic problems such as liver disease. The patient may want to see a dermatologist to have the angiomas removed, but this is not the initial action by the nurse. The nurse may need to teach the patient about the effects of aging on the skin and about the effects of sun exposure, but the initial action should be further assessment.

Which family of drugs are the following medications considered: Amlodipine, Verapamil, Diltiazem? a. Beta blockers (BB) b. ACE Inhibitors (ACEI) c. Angiotension Receptor Blockers (ARBs) d. Calcium Channel Blockers (CCBs)

c.


Conjuntos de estudio relacionados

Quality & Safety and Infrastructure

View Set

Biology - Evolution, Natural Selection, History of Life

View Set

Impact of Modern Technology on Society Final Exam

View Set

Chapter 10 Quiz Questions: (Quiz #5)

View Set